Tensorflow: рдкрд╛рдЗрдк рдХреЗ рд╕рд╛рде Python3.7 рдкрд░ TensorFlow рд╕реНрдерд╛рдкрд┐рдд рдХрд░рдиреЗ рдореЗрдВ рдЕрд╕рдорд░реНрде

рдХреЛ рдирд┐рд░реНрдорд┐рдд 30 рдЬреВрди 2018  ┬╖  148рдЯрд┐рдкреНрдкрдгрд┐рдпрд╛рдБ  ┬╖  рд╕реНрд░реЛрдд: tensorflow/tensorflow

рдкреНрд░рдгрд╛рд▓реА рдХреА рдЬрд╛рдирдХрд╛рд░реА

  • рдХреНрдпрд╛ рдореИрдВрдиреЗ рдХрд╕реНрдЯрдо рдХреЛрдб рд▓рд┐рдЦрд╛ рд╣реИ (рдЬреИрд╕рд╛ рдХрд┐ TensorFlow рдореЗрдВ рдкреНрд░рджрд╛рди рдХреА рдЧрдИ рд╕реНрдЯреЙрдХ рдЙрджрд╛рд╣рд░рдг рд╕реНрдХреНрд░рд┐рдкреНрдЯ рдХрд╛ рдЙрдкрдпреЛрдЧ рдХрд░рдиреЗ рдХреЗ рд╡рд┐рдкрд░реАрдд) : N/A
  • OS рдкреНрд▓реЗрдЯрдлреЙрд░реНрдо рдФрд░ рд╡рд┐рддрд░рдг (рдЬреИрд╕реЗ, Linux Ubuntu 16.04) : macOS 10.13
  • TensorFlow (рд╕реНрд░реЛрдд рдпрд╛ рдмрд╛рдЗрдирд░реА) рд╕реЗ рд╕реНрдерд╛рдкрд┐рдд : рдмрд╛рдЗрдирд░реА
  • TensorFlow рд╕рдВрд╕реНрдХрд░рдг (рдиреАрдЪреЗ рдХрдорд╛рдВрдб рдХрд╛ рдЙрдкрдпреЛрдЧ рдХрд░реЗрдВ) : 1.8
  • рдкрд╛рдпрдерди рд╕рдВрд╕реНрдХрд░рдг : 3.7
  • рдмреЗрдЬрд╝рд▓ рд╕рдВрд╕реНрдХрд░рдг (рдпрджрд┐ рд╕реНрд░реЛрдд рд╕реЗ рд╕рдВрдХрд▓рд┐рдд рдХрд┐рдпрд╛ рдЬрд╛ рд░рд╣рд╛ рд╣реИ) : рдПрди/рдП
  • рдЬреАрд╕реАрд╕реА/рдХрдВрдкрд╛рдЗрд▓рд░ рд╕рдВрд╕реНрдХрд░рдг (рдпрджрд┐ рд╕реНрд░реЛрдд рд╕реЗ рд╕рдВрдХрд▓рд┐рдд рдХрд┐рдпрд╛ рдЬрд╛ рд░рд╣рд╛ рд╣реИ) : рд▓рд╛рдЧреВ рдирд╣реАрдВ
  • CUDA/cuDNN рд╕рдВрд╕реНрдХрд░рдг : рдПрди/рдП
  • GPU рдореЙрдбрд▓ рдФрд░ рдореЗрдореЛрд░реА : N/A
  • рдкреБрди: рдкреЗрд╢ рдХрд░рдиреЗ рдХреЗ рд▓рд┐рдП рд╕рдЯреАрдХ рдЖрджреЗрд╢ : pip install tensorflow

рд╕рдорд╕реНрдпрд╛ рд╡рд┐рд╕реНрддрд╛рд░ рд╕реЗ рд╕рдордЭрд╛рдЗрдпреЗ

рдкрд╛рдЗрдк рдХреЗ рд╕рд╛рде Python3.7 рдкрд░ TensorFlow рд╕реНрдерд╛рдкрд┐рдд рдХрд░рдирд╛ рд╡рд┐рдлрд▓ рд░рд╣рд╛ред рдХреГрдкрдпрд╛ рдиреАрдЪреЗ рд╡рд┐рдлрд▓рддрд╛ рд▓реЙрдЧ рджреЗрдЦреЗрдВред

рд╕реНрд░реЛрдд рдХреЛрдб / рд▓реЙрдЧ

рдРрд╕рд╛ рд╕рдВрд╕реНрдХрд░рдг рдирд╣реАрдВ рдорд┐рд▓рд╛ рдЬреЛ рдЯреЗрдВрд╕рд░рдлрд╝реНрд▓реЛ рдХреА рдЖрд╡рд╢реНрдпрдХрддрд╛ рдХреЛ рдкреВрд░рд╛ рдХрд░рддрд╛ рд╣реЛ (рд╕рдВрд╕реНрдХрд░рдгреЛрдВ рд╕реЗ:)
рдЯреЗрдВрд╕рд░рдлрд╝реНрд▓реЛ рдХреЗ рд▓рд┐рдП рдХреЛрдИ рдорд┐рд▓рд╛рди рд╡рд┐рддрд░рдг рдирд╣реАрдВ рдорд┐рд▓рд╛

community support builinstall

рд╕рдмрд╕реЗ рдЙрдкрдпреЛрдЧреА рдЯрд┐рдкреНрдкрдгреА

рдореИрдВрдиреЗ рдЗрд╕реЗ рд╕рдлрд▓рддрд╛рдкреВрд░реНрд╡рдХ рд╕реНрдерд╛рдкрд┐рдд рдХрд┐рдпрд╛ pip install https://storage.googleapis.com/tensorflow/mac/cpu/tensorflow-1.8.0-py3-none-any.whl ред рд╣рд╛рд▓рд╛рдБрдХрд┐, рдРрд╕рд╛ рд▓рдЧрддрд╛ рд╣реИ рдХрд┐ рдпрд╣ рдкрд╣рд┐рдпрд╛ Python 3.7 рд╕рдВрдЧрдд рдирд╣реАрдВ рд╣реИред рд╕рдорд╕реНрдпрд╛ рдпрд╣ рд╣реИ рдХрд┐ tensorflow/python/pywrap_tensorflow_internal.py рдореЗрдВ рдЪрд░ рдирд╛рдо рдХреЗ рд░реВрдк рдореЗрдВ async рд╣реИ рд▓реЗрдХрд┐рди async python3.7 рдореЗрдВ рдПрдХ рдХреАрд╡рд░реНрдб рдмрди рдЬрд╛рддрд╛ рд╣реИ, рдЗрд╕рд▓рд┐рдП рдпрд╣ рдЕрдм рдЙрдкрд▓рдмреНрдз рдирд╣реАрдВ рд╣реИред

рд╕рд╛рде рд╣реА, рдореИрдВрдиреЗ рджреЗрдЦрд╛ рдХрд┐ рдпреВрдЖрд░рдПрд▓ рдХреЗрд╡рд▓ рдкрд╛рдпрдерди 3.4, 3.5, 3.6 рдХреЗ рд▓рд┐рдП рд╣реИ рд▓реЗрдХрд┐рди рдореБрдЭреЗ рдХреЛрдИ рдЕрдиреНрдп рдпреВрдЖрд░рдПрд▓ рдирд╣реАрдВ рдорд┐рд▓ рд░рд╣рд╛ рд╣реИред рддреЛ рд╢рд╛рдпрдж рд╣рдореЗрдВ рдЖрдзрд┐рдХрд╛рд░рд┐рдХ рдкрд╛рдпрдерди 3.7-рд╕рдВрдЧрдд рдкреИрдХреЗрдЬ рдХреА рдкреНрд░рддреАрдХреНрд╖рд╛ рдХрд░рдиреЗ рдХреА рдЖрд╡рд╢реНрдпрдХрддрд╛ рд╣реИред

рд╕рднреА 148 рдЯрд┐рдкреНрдкрдгрд┐рдпрд╛рдБ

рдореИрдВрдиреЗ рдЗрд╕реЗ рд╕рдлрд▓рддрд╛рдкреВрд░реНрд╡рдХ рд╕реНрдерд╛рдкрд┐рдд рдХрд┐рдпрд╛ pip install https://storage.googleapis.com/tensorflow/mac/cpu/tensorflow-1.8.0-py3-none-any.whl ред рд╣рд╛рд▓рд╛рдБрдХрд┐, рдРрд╕рд╛ рд▓рдЧрддрд╛ рд╣реИ рдХрд┐ рдпрд╣ рдкрд╣рд┐рдпрд╛ Python 3.7 рд╕рдВрдЧрдд рдирд╣реАрдВ рд╣реИред рд╕рдорд╕реНрдпрд╛ рдпрд╣ рд╣реИ рдХрд┐ tensorflow/python/pywrap_tensorflow_internal.py рдореЗрдВ рдЪрд░ рдирд╛рдо рдХреЗ рд░реВрдк рдореЗрдВ async рд╣реИ рд▓реЗрдХрд┐рди async python3.7 рдореЗрдВ рдПрдХ рдХреАрд╡рд░реНрдб рдмрди рдЬрд╛рддрд╛ рд╣реИ, рдЗрд╕рд▓рд┐рдП рдпрд╣ рдЕрдм рдЙрдкрд▓рдмреНрдз рдирд╣реАрдВ рд╣реИред

рд╕рд╛рде рд╣реА, рдореИрдВрдиреЗ рджреЗрдЦрд╛ рдХрд┐ рдпреВрдЖрд░рдПрд▓ рдХреЗрд╡рд▓ рдкрд╛рдпрдерди 3.4, 3.5, 3.6 рдХреЗ рд▓рд┐рдП рд╣реИ рд▓реЗрдХрд┐рди рдореБрдЭреЗ рдХреЛрдИ рдЕрдиреНрдп рдпреВрдЖрд░рдПрд▓ рдирд╣реАрдВ рдорд┐рд▓ рд░рд╣рд╛ рд╣реИред рддреЛ рд╢рд╛рдпрдж рд╣рдореЗрдВ рдЖрдзрд┐рдХрд╛рд░рд┐рдХ рдкрд╛рдпрдерди 3.7-рд╕рдВрдЧрдд рдкреИрдХреЗрдЬ рдХреА рдкреНрд░рддреАрдХреНрд╖рд╛ рдХрд░рдиреЗ рдХреА рдЖрд╡рд╢реНрдпрдХрддрд╛ рд╣реИред

рдХрд╛рд░рдг рдХреА рдкрд╣рдЪрд╛рди рдХрд░рдиреЗ рдХреЗ рд▓рд┐рдП рдзрдиреНрдпрд╡рд╛рдж!

рдЕрднреА рднреА рдЗрд╕реЗ python3.7 . рдХреЗ рд╕рд╛рде рд╕реНрдерд╛рдкрд┐рдд рдирд╣реАрдВ рдХрд░ рд╕рдХрддрд╛

(env) mcbk:trainer todor$ pip3 install tensorflow
Collecting tensorflow
  Could not find a version that satisfies the requirement tensorflow (from versions: )
No matching distribution found for tensorflow
(env) mcbk:trainer todor$ which python3.7
/Users/todor/xor/projects/trainer/env/bin/python3.7
(env) mcbk:trainer todor$ which pip3
/Users/todor/xor/projects/trainer/env/bin/pip3

рдореЗрд░рд╛ рднреА рдпрд╣реА рд╡рд┐рдЪрд╛рд░ рд╣реИред

+1

рдХрд┐рд╕реА рднреА рдЕрджреНрдпрддрди?

рдХреНрд░рд┐рд╕-рдПрдордмреАрдкреА:~ рдХреНрд░рд┐рд╕ $ pip3 tensorflow рд╕реНрдерд╛рдкрд┐рдд рдХрд░реЗрдВ
рдЯреЗрдВрд╕рд░рдлрд╝реНрд▓реЛ рдПрдХрддреНрд░рд┐рдд рдХрд░рдирд╛
рдРрд╕рд╛ рд╕рдВрд╕реНрдХрд░рдг рдирд╣реАрдВ рдорд┐рд▓рд╛ рдЬреЛ рдЯреЗрдВрд╕рд░рдлрд╝реНрд▓реЛ рдХреА рдЖрд╡рд╢реНрдпрдХрддрд╛ рдХреЛ рдкреВрд░рд╛ рдХрд░рддрд╛ рд╣реЛ (рд╕рдВрд╕реНрдХрд░рдгреЛрдВ рд╕реЗ:)
рдЯреЗрдВрд╕рд░рдлрд╝реНрд▓реЛ рдХреЗ рд▓рд┐рдП рдХреЛрдИ рдорд┐рд▓рд╛рди рд╡рд┐рддрд░рдг рдирд╣реАрдВ рдорд┐рд▓рд╛

рдЗрд╕рдХреЗ рд▓рд┐рдП рдлрд┐рдХреНрд╕ рдкрд░ рдХреЛрдИ рдЕрдкрдбреЗрдЯ? рдзрдиреНрдпрд╡рд╛рджред

рдпрд╣ рдореБрджреНрджрд╛ рдХреНрдпреЛрдВ рдмрдВрдж рдХрд┐рдпрд╛ рдЧрдпрд╛?

рдХреНрдпрд╛ рдЗрд╕ рдкрд╣рд┐рдпреЗ рдХреА рд╕рдорд╕реНрдпрд╛ https://storage.googleapis.com/tensorflow/mac/cpu/tensorflow-1.8.0-py3-none-any.whl рдХреЛ рдареАрдХ рдХрд░ рджрд┐рдпрд╛ рдЧрдпрд╛ рд╣реИ? рдпрд╛ рдпреЛрдВ рдХрд╣реЗрдВ рдХрд┐ рдирдпрд╛ рд▓рд┐рдВрдХ/рд╕рдВрд╕реНрдХрд░рдг рдХреНрдпрд╛ рд╣реИ?

рдкрд╛рдпрдерди 3.7 рдкреВрд░реА рддрд░рд╣ рд╕реЗ рд░рд┐рд▓реАрдЬ рд╣реЛрдиреЗ рдХреЗ рдХрд╛рд░рдг, рд╕рднреА рдЙрддреНрдкрд╛рджрди рдУрдкрдирд╕реАрд╡реА рдореМрдд рд╣реИрдВ рдХреНрдпреЛрдВрдХрд┐ рдЯреЗрдВрд╕рд░рдлреНрд▓реЛ рдЕрд╕рдВрдЧрдд рд╣реИрдВред рдХреГрдкрдпрд╛ рдорджрдж рдХрд░реЗрдВ !!!!!!

рдпрд╣рд╛рдВ рдПрдХ рд╣реА рд╕рдорд╕реНрдпрд╛ рд╣реИ .. рдпрд╣ рдЬрд╛рдирдирд╛ рдмрд╣реБрдд рдЕрдЪреНрдЫрд╛ рд╣реЛрдЧрд╛ рдХрд┐ рдЗрд╕ рдореБрджреНрджреЗ рдХреЗ рд▓рд┐рдП рдХреЛрдИ рдлрд┐рдХреНрд╕ рдХрдм рдЙрдкрд▓рдмреНрдз рд╣реЛрдЧрд╛ :)

рдпрд╣ рдореБрдЭреЗ рдХрдИ рдкрд░рд┐рдпреЛрдЬрдирд╛рдУрдВ рдХреЛ 3.7 рдореЗрдВ рдЕрдкрдЧреНрд░реЗрдб рдХрд░рдиреЗ рд╕реЗ рд░реЛрдХ рд░рд╣рд╛ рд╣реИ, рдлрд┐рдХреНрд╕ рдИрдЯреАрдП рдмрд╣реБрдд рдЕрдЪреНрдЫрд╛ рд╣реЛрдЧрд╛ред

@richardARPANET рдореБрджреНрджрд╛ рдпрд╣ рд╣реИ рдХрд┐ рд╣рдо рдпрд╣ рдирд╣реАрдВ рдЬрд╛рди рдкрд╛рдПрдВрдЧреЗ рдХрд┐ рдЕрдЬрдЧрд░ 3.7 рдХреЛ рдЕрдкрдЧреНрд░реЗрдб рдХрд░рдиреЗ рд╕реЗ рдкрд╣рд▓реЗ рдХреНрдпрд╛ рд╣реЛрдЧрд╛, рдХреНрдпреЛрдВрдХрд┐ рдХреЛрдИ рднреА рдирд╣реАрдВ рдЬрд╛рдПрдЧрд╛ рдФрд░ рдкреНрд░рддреНрдпреЗрдХ рдореЙрдбреНрдпреВрд▓ рдХреА рдЬрд╛рдВрдЪ рдХрд░реЗрдЧрд╛ рдХрд┐ рдпрд╣ рд╕рдВрдЧрдд рд╣реИ рдпрд╛ рдирд╣реАрдВред

рдФрд░ рдЙрд╕рдХреЗ рдХрд╛рд░рдг, рдЕрдЬрдЧрд░ 3.7 рдореЗрдВ рдЕрдкрдЧреНрд░реЗрдб рдХрд░рдиреЗ рдХреЗ рдмрд╛рдж, рд╕рдм рдХреБрдЫ рджреБрдГрд╕реНрд╡рдкреНрди рдмрди рдЬрд╛рддрд╛ рд╣реИред

@thuyen рдРрд╕рд╛ рдирд╣реАрдВ рд╣реИ рдХрд┐ рд╕реНрд╡рдЪрд╛рд▓рд┐рдд рдкрд░реАрдХреНрд╖рдг рдХреНрдпреЛрдВ рдореМрдЬреВрдж рд╣реИрдВ?

рдореЗрд░рд╛ рднреА рдпрд╣реА рд╡рд┐рдЪрд╛рд░ рд╣реИред

рдпрд╣рд╛рдВ рднреА рд╡рд╣реА рдореБрджреНрджрд╛ред

рдпрд╣рд╛рдБ рд╡рд╣реА рдореБрджреНрджрд╛ред рдЗрддрдиреА рд╢рд░реНрдо рдХреА рдмрд╛рдд рд╣реИ рдХреНрдпреЛрдВрдХрд┐ рдореИрдВ рдЗрд╕реЗ рдЬреАрд╕реАрдкреА рдХреЗ рдХреНрд▓рд╛рдЙрдб рдлрд╝рдВрдХреНрд╢рдВрд╕ рдореЗрдВ рдЗрд╕реНрддреЗрдорд╛рд▓ рдХрд░рдирд╛ рдЪрд╛рд╣рддрд╛ рд╣реВрдВ, рдЬреЛ рдХреЗрд╡рд▓ 3.7 рдПрдЯреАрдПрдо рдХрд╛ рд╕рдорд░реНрдерди рдХрд░рддрд╛ рд╣реИ ...

@christiaanleysen 3.7 рдкрд░ рдХреБрдЫ рдорд╛рдореВрд▓реА рдмрджрд▓рд╛рд╡ рдФрд░ рдкреИрдЪ рдХреЗ рд╕рд╛рде рдареАрдХ рдХрд╛рдо рдХрд░рддрд╛ рд╣реИ рдпрджрд┐ рдЖрдк рд╕реНрд╡рдпрдВ рдХреЛ рдмрдирд╛рдиреЗ рдХреЗ рдЗрдЪреНрдЫреБрдХ рд╣реИрдВ: https://github.com/tensorflow/tensorflow/pull/21202

рдпрд╣ рдореЗрд░реЗ рд▓рд┐рдП рдХрд╛рдо рд╣реИ
рдмрд╕ рдЗрд╕реЗ рдиреАрдЪреЗ рдЙрдкрдпреЛрдЧ рдХрд░рдиреЗ рдХрд╛ рдкреНрд░рдпрд╛рд╕ рдХрд░реЗрдВ
рдкрд╛рдЗрдк рд╕реНрдерд╛рдкрд┐рдд рдХрд░реЗрдВ https://storage.googleapis.com/tensorflow/mac/cpu/tensorflow-1.8.0-py3-none-any.whl

рдЕрдЬрдЧрд░ 3.7 рдХреЗ рд▓рд┐рдП рдлрд┐рдХреНрд╕ рдХреЗ рд╕рд╛рде рдЯреЗрдВрдбрд░рдлреНрд▓реЛ-рдЬреАрдкреАрдпреВ рдХреИрд╕реЗ рдмрдирд╛рдПрдВ? @bstriner

@Mahamatnour
рдпрд╣ рдореЗрд░реЗ рд▓рд┐рдП рдХрд╛рдо рдирд╣реАрдВ рдХрд░рддрд╛ рд╣реИред

рдпрд╣ рдХрдорд╛рдВрдб рдХрд╛ рдЙрдкрдпреЛрдЧ рдХрд░рдХреЗ рдореЗрд░реЗ рд▓рд┐рдП рдареАрдХ рдХрд╛рдо рдХрд░рддрд╛ рд╣реИ:
pip3 install https://storage.googleapis.com/tensorflow/mac/cpu/tensorflow-1.10.1-py3-none-any.whl

рдХреГрдкрдпрд╛ https://www.tensorflow.org/install/install_mac#the_url_of_the_tensorflow_python_package рд╕реЗ рджрд╕реНрддрд╛рд╡реЗрдЬрд╝ рджреЗрдЦреЗрдВ

рд╕рдорд╕реНрдпрд╛ рдореИрдХ рдкреНрд▓реЗрдЯрдлреЙрд░реНрдо рдкрд░ рдирд╣реАрдВ рд╣реИ !!!

рд╕рднреА рдореМрдЬреВрджрд╛ рд╕рдорд╕реНрдпрд╛рдПрдВ рд▓рд┐рдирдХреНрд╕ рдкреНрд▓реЗрдЯрдлреЙрд░реНрдо рдХреЗ рд▓рд┐рдП рд╣реИрдВ, рдпрджрд┐ рдЖрдк рдореИрдХ рдХрд╛ рдЙрдкрдпреЛрдЧ рдХрд░ рд░рд╣реЗ рд╣реИрдВ, рддреЛ рдЗрд╕реЗ рдпрд╣рд╛рдВ рд░рд┐рдкреЛрд░реНрдЯ рдХрд░рдиреЗ рдХреА рдЖрд╡рд╢реНрдпрдХрддрд╛ рдирд╣реАрдВ рд╣реИ, рдХреНрдпреЛрдВрдХрд┐ рдореИрдХ рдкреНрд▓реЗрдЯрдлреЙрд░реНрдо рдХреЛ рдХреЛрдИ рд╕рдорд╕реНрдпрд╛ рдирд╣реАрдВ рдорд┐рд▓реА рд╣реИ

@thyeun рдХреГрдкрдпрд╛, рдирд╛рддреНрд╕реБрдХрд┐рдпрдо рдХреЗ рдкреНрд▓реЗрдЯрдлреЙрд░реНрдо рдХреЛ рджреЗрдЦреЗрдВ ... рдореИрдВрдиреЗ рдЙрд╕реЗ рд╕рд┐рд░реНрдл рдЗрд╕рд▓рд┐рдП рдЙрддреНрддрд░ рджрд┐рдпрд╛ рдХреНрдпреЛрдВрдХрд┐ рдореЗрд░реЗ рдкрд╛рд╕ рдЖрдЙрдЯрдкреБрдЯ рдореЗрдВ рдПрдХ рд╣реА рддреНрд░реБрдЯрд┐ рдереА ... (рдФрд░ рдЙрд╕реА рдкреНрд▓реЗрдЯрдлреЙрд░реНрдо рдХреЗ рд╕рд╛рде ...)

@ рдмреНрд░рд╛рдЙрдиреА 75 рдХреГрдкрдпрд╛, рдореИрдВрдиреЗ рдпрд╣ рдирд╣реАрдВ рдХрд╣рд╛ рд╣реИ рдХрд┐ рдЖрдк рдкрд░, рдпрд╣ рд╕реБрдирд┐рд╢реНрдЪрд┐рдд рдХрд░рдиреЗ рдХреЗ рд▓рд┐рдП рдХрд┐ рдореИрдХ рдЙрдкрдпреЛрдЧрдХрд░реНрддрд╛рдУрдВ рдХреЛ рдкрд╛рдпрдерди 3.7 рдкрд░ рдХреЛрдИ рд╕рдорд╕реНрдпрд╛ рдирд╣реАрдВ рд╣реИ, рдХреЗрд╡рд▓ рд╕рдорд╕реНрдпрд╛ рд▓рд┐рдирдХреНрд╕ рдЙрдкрдпреЛрдЧрдХрд░реНрддрд╛рдУрдВ рдХреЗ рд▓рд┐рдП рд╣реИ !!!, рдореЗрд░реА рдЯрд┐рдкреНрдкрдгреА рдХреЛ рдЧрд▓рдд рди рд╕рдордЭреЗрдВред

@thyeun рд╕рдордЭрдиреЗ рдпрд╛ рдЧрд▓рдд рд╕рдордЭрдиреЗ рдЬреИрд╕реА рдХреЛрдИ рдмрд╛рдд рдирд╣реАрдВ рд╣реИ, cu, рдХреЛрдИ рднреА рд╕реЛрдЪрдирд╛ рдареАрдХ рд╣реИ

рдпрд╣рд╛рдВ рдЬреЛ рдЗрд╕рдХреЗ рд▓рд╛рдпрдХ рд╣реИ, рд╡рд╣ 3.7 рдФрд░ Tensorflow рдХреЗ рд╕рд╛рде рдбреЙрдХрд░ рдЫрд╡рд┐ рдмрдирд╛рдиреЗ рдХреЗ рд▓рд┐рдП рд░реЗрдкреЛ рд╣реИ рдЬреЛ рдХрд╛рдо рдХрд░рддрд╛ рд╣реИ рд▓реЗрдХрд┐рди рдЗрд╕реЗ рдмрдирд╛рдиреЗ рдореЗрдВ рдмрд╣реБрдд рд╕рдордп рд▓рдЧрддрд╛ рд╣реИ: https://github.com/Guillemdb/docker-base

@ SukeshP1995 рд▓рд┐рдВрдХ рдореЗрдВ рд╡рд┐рд╕реНрддреГрдд рд╣реЛрдирд╛ рдЪрд╛рд╣рд┐рдП, рд▓реЗрдХрд┐рди рдЕрдЧрд░ рдЖрдкрдХреЛ рдХреЛрдИ рд╕рдорд╕реНрдпрд╛ рд╣реИ рддреЛ рдореБрдЭреЗ рдмрддрд╛рдПрдВред рдореБрдЦреНрдп рдкрд░рд┐рд╡рд░реНрддрди рдЕрдм рд░рд┐рдкреЙрдЬрд┐рдЯрд░реА рдореЗрдВ рд╣реИрдВ, рд▓реЗрдХрд┐рди рдЖрдкрдХреЛ рдЕрднреА рднреА workspace.bzl рдХреЛ рдЕрдкрдбреЗрдЯ рдХрд░рдиреЗ рдФрд░ eigen рдФрд░ protobuf рдХреЛ рдкреИрдЪ рдХрд░рдиреЗ рдХреЗ рд▓рд┐рдП рд╕рдВрд╢реЛрдзрд┐рдд рдХрд░рдирд╛ рд╣реЛрдЧрд╛ред

@vmarkovtsev рд╢рд╛рдпрдж рдЬрд╛рдВрдЪреЗрдВ рдХрд┐ рдХрд┐рддрдиреЗ рдзрд╛рдЧреЗ рдЪрд▓ рд░рд╣реЗ рд╣реИрдВред рдХрднреА-рдХрднреА рдбреЙрдХрд░ рдореЗрдВ рд░рд╛рд╕реНрддрд╛ рдмрдВрдж рд╣реЛ рдЬрд╛рддрд╛ рд╣реИред 8 рд╕реАрдкреАрдпреВ рдкрд░ 1k рдзрд╛рдЧреЗ рдЪрд▓рд╛рдиреЗ рдХреА рдХреЛрд╢рд┐рд╢ рдХрд░рддрд╛ рд╣реИ...

@bstriner рдореИрдВ рд╕рдлрд▓рддрд╛рдкреВрд░реНрд╡рдХ рд╕реНрдерд╛рдиреАрдп рд░реВрдк рд╕реЗ рдирд┐рд░реНрдорд╛рдг рдХрд░рддрд╛ рд╣реВрдВ, Dockerhub рдкрд░ рдирд┐рд░реНрдорд╛рдг рдХрд░рдиреЗ рдореЗрдВ рдЕрдзрд┐рдХ рд╕рдордп рд▓рдЧрддрд╛ рд╣реИ рдФрд░ рдПрдХ рдХрдард┐рди рд╕реАрдорд╛ рд╣реЛрддреА рд╣реИред

рдпрджрд┐ рдЖрдк dockerhub рдмрдирд╛рдирд╛ рдЪрд╛рд╣рддреЗ рд╣реИрдВ, рддреЛ рдХрдИ рдЫрд╡рд┐рдпреЛрдВ рдХреЛ рд╢реНрд░реГрдВрдЦрд▓рд╛рдмрджреНрдз рдХрд░реЗрдВ

рдЧреБрд░реБ, рдЕрдЧрд╕реНрдд 30, 2018, 3:01 рдЕрдкрд░рд╛рд╣реНрди рдкрд░ рд╡рд╛рджрд┐рдо рдорд╛рд░реНрдХреЛрд╡рддреНрд╕реЗрд╡ рдЕрдзрд┐рд╕реВрдЪрдирд╛рдПрдВ @github.com
рд▓рд┐рдЦрд╛ рдерд╛:

@bstriner https://github.com/bstriner рдореИрдВ рд╕рдлрд▓рддрд╛рдкреВрд░реНрд╡рдХ рд╕реНрдерд╛рдиреАрдп рд░реВрдк рд╕реЗ рдирд┐рд░реНрдорд╛рдг рдХрд░рддрд╛ рд╣реВрдВ, it
Dockerhub рдкрд░ рдирд┐рд░реНрдорд╛рдг рдХрд░рдиреЗ рдореЗрдВ рдЕрдзрд┐рдХ рд╕рдордп рд▓рдЧрддрд╛ рд╣реИ рдФрд░ рдЗрд╕рдХреА рдПрдХ рдХрдард┐рди рд╕реАрдорд╛ рд╣реЛрддреА рд╣реИред

-
рдЖрдк рдЗрд╕реЗ рдкреНрд░рд╛рдкреНрдд рдХрд░ рд░рд╣реЗ рд╣реИрдВ рдХреНрдпреЛрдВрдХрд┐ рдЖрдкрдХрд╛ рдЙрд▓реНрд▓реЗрдЦ рдХрд┐рдпрд╛ рдЧрдпрд╛ рдерд╛ред
рдЗрд╕ рдИрдореЗрд▓ рдХрд╛ рд╕реАрдзреЗ рдЙрддреНрддрд░ рджреЗрдВ, рдЗрд╕реЗ GitHub рдкрд░ рджреЗрдЦреЗрдВ
https://github.com/tensorflow/tensorflow/issues/20444#issuecomment-417430029 ,
рдпрд╛ рдереНрд░реЗрдб рдХреЛ рдореНрдпреВрдЯ рдХрд░реЗрдВ
https://github.com/notifications/unsubscribe-auth/AL4rbPXGZMGkGX-ETM718jmMSogkWiJsks5uWDZvgaJpZM4U-EUx
.

90% рд╕рдордп bazel build @bstriner . рджреНрд╡рд╛рд░рд╛ рд▓рд┐рдпрд╛ рдЬрд╛рддрд╛ рд╣реИ

рд╣рд╛рдЗрдкреЛрдереЗрдЯрд┐рдХ рд░реВрдк рд╕реЗ, рдХреБрдЫ рдЙрдк-рдкрд░рд┐рдпреЛрдЬрдирд╛рдУрдВ рдХреЛ рд╡реНрдпрдХреНрддрд┐рдЧрдд рд░реВрдк рд╕реЗ рдкрд╣рд▓реЗ рд╕реЗ рдПрдХ рдореЗрдВ рдЪрд▓рд╛рдПрдВ
рдЫрд╡рд┐, рдлрд┐рд░ рдкреИрдХреЗрдЬ рдХреЛ рджреВрд╕рд░реЗ рдореЗрдВ рдмрдирд╛рдПрдВ рдФрд░ рд╡реЗ рдирд┐рд░реНрднрд░рддрд╛рдПрдВ рдирд╣реАрдВ рд╣реЛрдВрдЧреА
рдкреБрдирд░реНрдирд┐рд░реНрдорд╛рдг, рд▓реЗрдХрд┐рди рд╢рд╛рдпрдж рдЗрд╕рдХреЗ рд▓рд╛рдпрдХ рд╕реЗ рдЕрдзрд┐рдХ рджрд░реНрджред TF рдЯреАрдо рдирд╣реАрдВ рдХрд░рдирд╛ рдЪрд╛рд╣рддреА
рдЕрдиреМрдкрдЪрд╛рд░рд┐рдХ рд░рд┐рд▓реАрдЬ рдкрд░ рдирд┐рд░реНрднрд░рддрд╛ рдорд░реНрдЬ рдХрд░реЗрдВ, рдЗрд╕рд▓рд┐рдП рд╕рдмрд╕реЗ рдмрдбрд╝реА рдмрд╛рдзрд╛
рдЖрдзрд┐рдХрд╛рд░рд┐рдХ 3.7 рд╕рднреА рдУрдПрд╕ рдХреЗ рд▓рд┐рдП рдмрдирд╛рддрд╛ рд╣реИ рдкреНрд░реЛрдЯреЛрдмрдл рдФрд░ рдИрдЬреЗрди рдХреЛ рдбреАрдмрдЧ рдХрд░рдирд╛ рд╣реИред рдкрд░ рдЕрдЧрд░ рддреБрдо
3.7 рдХреЗ рдмрд╛рд░реЗ рдореЗрдВ рдкрд░рд╡рд╛рд╣ рд╣реИ, рдЖрдк рд╢рд╛рдпрдж рд╡реИрд╕реЗ рднреА рд╕реНрд░реЛрдд рд╕реЗ рдирд┐рд░реНрдорд╛рдг рдХрд░рдирд╛ рдЪрд╛рд╣рддреЗ рд╣реИрдВ
рд╕рд╣реА рдЕрдиреБрдХреВрд▓рдиред

рдЧреБрд░реБ, рдЕрдЧрд╕реНрдд 30, 2018 рдЕрдкрд░рд╛рд╣реНрди 4:10 рдмрдЬреЗ рд╡рд╛рджрд┐рдо рдорд╛рд░реНрдХреЛрд╡рддреНрд╕реЗрд╡ рд╕реВрдЪрдирд╛рдПрдВ @github.com
рд▓рд┐рдЦрд╛ рдерд╛:

90% рд╕рдордп bazel рдмрд┐рд▓реНрдб @bstriner . рджреНрд╡рд╛рд░рд╛ рд▓рд┐рдпрд╛ рдЬрд╛рддрд╛ рд╣реИ
https://github.com/bstriner

-
рдЖрдк рдЗрд╕реЗ рдкреНрд░рд╛рдкреНрдд рдХрд░ рд░рд╣реЗ рд╣реИрдВ рдХреНрдпреЛрдВрдХрд┐ рдЖрдкрдХрд╛ рдЙрд▓реНрд▓реЗрдЦ рдХрд┐рдпрд╛ рдЧрдпрд╛ рдерд╛ред
рдЗрд╕ рдИрдореЗрд▓ рдХрд╛ рд╕реАрдзреЗ рдЙрддреНрддрд░ рджреЗрдВ, рдЗрд╕реЗ GitHub рдкрд░ рджреЗрдЦреЗрдВ
https://github.com/tensorflow/tensorflow/issues/20444#issuecomment-417450405 ,
рдпрд╛ рдереНрд░реЗрдб рдХреЛ рдореНрдпреВрдЯ рдХрд░реЗрдВ
https://github.com/notifications/unsubscribe-auth/AL4rbLxpR-fNjtL58nLw3AeBd0R_VVrhks5uWEawgaJpZM4U-EUx
.

рдореИрдВ рдпрд╣ рдирд╣реАрдВ рдХрд╣рдирд╛ рдЪрд╛рд╣рддрд╛ рдХрд┐ рдбреЙрдХрд╣рдм рдореЗрдВ рдЕрдЪреНрдЫреЗ рд▓реЛрдЧреЛрдВ рдХрд╛ рд╢реЛрд╖рдг рдХрд░реЗрдВ, рд▓реЗрдХрд┐рди рдореЗрд░реЗ рдкрд╛рд╕ рд╣реИ
рдкрд░рд┐рдпреЛрдЬрдирд╛рдУрдВ рдХреЗ рд▓рд┐рдП рдореБрдЭреЗ pytorch рдФрд░ рдПрдХ рдЯрди рдХрд╛ рдирд┐рд░реНрдорд╛рдг рдХрд░рдиреЗ рдХреЗ рд▓рд┐рдП 3-4 рдЫрд╡рд┐рдпреЛрдВ рдХреА рдПрдХ рд╢реНрд░реГрдВрдЦрд▓рд╛ рдХреА рдЖрд╡рд╢реНрдпрдХрддрд╛ рдереА
рд╕рдВрдмрдВрдзрд┐рдд рд╕реЙрдлреНрдЯрд╡реЗрдпрд░ред рдпрд╣ рдЬрдирддрд╛ рдХреА рднрд▓рд╛рдИ рдХреЗ рд▓рд┐рдП рдерд╛ (рдПрдХ рд╡рд░реНрдЧ рдХреЗ рд▓рд┐рдП)ред рдЬрдм рддрдХ
рдЖрдк рд▓рдЧрд╛рддрд╛рд░ рдкреБрдирд░реНрдирд┐рд░реНрдорд╛рдг рдирд╣реАрдВ рдХрд░ рд░рд╣реЗ рд╣реИрдВ рдФрд░ рдЖрдк рдЗрд╕рдХреЗ рд╕рд╛рде рдХреБрдЫ рдЕрдЪреНрдЫрд╛ рдХрд░ рд░рд╣реЗ рд╣реИрдВ,
рдЖрдкрдХреЛ рдЬрд┐рддрдиреЗ рдХрджрдо рдЪрд╛рд╣рд┐рдП рдЙрддрдиреЗ рдХрджрдо рдЙрдард╛рдПрдВред

рдЧреБрд░реБ, рдЕрдЧрд╕реНрдд 30, 2018 рдЕрдкрд░рд╛рд╣реНрди 4:22 рдмрдЬреЗ рдмреЗрдВрдЬрд╛рдорд┐рди рд╕реНрдЯреНрд░рд┐рдирд░ [email protected] рдиреЗ рд▓рд┐рдЦрд╛:

рд╣рд╛рдЗрдкреЛрдереЗрдЯрд┐рдХ рд░реВрдк рд╕реЗ, рдХреБрдЫ рдЙрдк-рдкрд░рд┐рдпреЛрдЬрдирд╛рдУрдВ рдХреЛ рд╡реНрдпрдХреНрддрд┐рдЧрдд рд░реВрдк рд╕реЗ рдкрд╣рд▓реЗ рд╕реЗ рдПрдХ рдореЗрдВ рдЪрд▓рд╛рдПрдВ
рдЫрд╡рд┐, рдлрд┐рд░ рдкреИрдХреЗрдЬ рдХреЛ рджреВрд╕рд░реЗ рдореЗрдВ рдмрдирд╛рдПрдВ рдФрд░ рд╡реЗ рдирд┐рд░реНрднрд░рддрд╛рдПрдВ рдирд╣реАрдВ рд╣реЛрдВрдЧреА
рдкреБрдирд░реНрдирд┐рд░реНрдорд╛рдг, рд▓реЗрдХрд┐рди рд╢рд╛рдпрдж рдЗрд╕рдХреЗ рд▓рд╛рдпрдХ рд╕реЗ рдЕрдзрд┐рдХ рджрд░реНрджред TF рдЯреАрдо рдирд╣реАрдВ рдХрд░рдирд╛ рдЪрд╛рд╣рддреА
рдЕрдиреМрдкрдЪрд╛рд░рд┐рдХ рд░рд┐рд▓реАрдЬ рдкрд░ рдирд┐рд░реНрднрд░рддрд╛ рдорд░реНрдЬ рдХрд░реЗрдВ, рдЗрд╕рд▓рд┐рдП рд╕рдмрд╕реЗ рдмрдбрд╝реА рдмрд╛рдзрд╛
рдЖрдзрд┐рдХрд╛рд░рд┐рдХ 3.7 рд╕рднреА рдУрдПрд╕ рдХреЗ рд▓рд┐рдП рдмрдирд╛рддрд╛ рд╣реИ рдкреНрд░реЛрдЯреЛрдмрдл рдФрд░ рдИрдЬреЗрди рдХреЛ рдбреАрдмрдЧ рдХрд░рдирд╛ рд╣реИред рдкрд░ рдЕрдЧрд░ рддреБрдо
3.7 рдХреЗ рдмрд╛рд░реЗ рдореЗрдВ рдкрд░рд╡рд╛рд╣ рд╣реИ, рдЖрдк рд╢рд╛рдпрдж рд╡реИрд╕реЗ рднреА рд╕реНрд░реЛрдд рд╕реЗ рдирд┐рд░реНрдорд╛рдг рдХрд░рдирд╛ рдЪрд╛рд╣рддреЗ рд╣реИрдВ
рд╕рд╣реА рдЕрдиреБрдХреВрд▓рдиред

рдЧреБрд░реБ, рдЕрдЧрд╕реНрдд 30, 2018 рдЕрдкрд░рд╛рд╣реНрди 4:10 рдмрдЬреЗ рд╡рд╛рджрд┐рдо рдорд╛рд░реНрдХреЛрд╡рддреНрд╕реЗрд╡ рд╕реВрдЪрдирд╛рдПрдВ @github.com
рд▓рд┐рдЦрд╛ рдерд╛:

90% рд╕рдордп bazel рдмрд┐рд▓реНрдб @bstriner . рджреНрд╡рд╛рд░рд╛ рд▓рд┐рдпрд╛ рдЬрд╛рддрд╛ рд╣реИ
https://github.com/bstriner

-
рдЖрдк рдЗрд╕реЗ рдкреНрд░рд╛рдкреНрдд рдХрд░ рд░рд╣реЗ рд╣реИрдВ рдХреНрдпреЛрдВрдХрд┐ рдЖрдкрдХрд╛ рдЙрд▓реНрд▓реЗрдЦ рдХрд┐рдпрд╛ рдЧрдпрд╛ рдерд╛ред
рдЗрд╕ рдИрдореЗрд▓ рдХрд╛ рд╕реАрдзреЗ рдЙрддреНрддрд░ рджреЗрдВ, рдЗрд╕реЗ GitHub рдкрд░ рджреЗрдЦреЗрдВ
https://github.com/tensorflow/tensorflow/issues/20444#issuecomment-417450405 ,
рдпрд╛ рдереНрд░реЗрдб рдХреЛ рдореНрдпреВрдЯ рдХрд░реЗрдВ
https://github.com/notifications/unsubscribe-auth/AL4rbLxpR-fNjtL58nLw3AeBd0R_VVrhks5uWEawgaJpZM4U-EUx
.

рдореИрдВрдиреЗ рдереЛрдбрд╝реА рджреЗрд░ рдХреА рдЦреЛрдЬ рдХреЗ рдмрд╛рдж рдЗрд╕реЗ рдХрд╛рдо рдХрд░рдиреЗ рдХреЗ рд▓рд┐рдП рдкрд╛рдпрд╛ред рдореИрдВ рдЗрд╕реЗ рдпрд╣рд╛рдБ рдЫреЛрдбрд╝ рджреВрдБрдЧрд╛ рдХреНрдпреЛрдВрдХрд┐ рдпрд╣ рдХрд┐рд╕реА рдХреЗ рд▓рд┐рдП рдЙрдкрдпреЛрдЧреА рд╣реЛ рд╕рдХрддрд╛ рд╣реИред

Tensorflow рдЖрдЬ рддрдХ рдХреЗрд╡рд▓ Python 3.6 рдХрд╛ рд╕рдорд░реНрдерди рдХрд░рддрд╛ рд╣реИред рдЖрдк рдЕрдкрдиреЗ рдорд╛рдирдХ рдХреЗ рд╕рд╛рде рдЕрдЬрдЧрд░ рдХрд╛ рдПрдХ рдЕрд▓рдЧ рд╕рдВрд╕реНрдХрд░рдг рд╕реНрдерд╛рдкрд┐рдд рдХрд░ рд╕рдХрддреЗ рд╣реИрдВред рдЖрдкрдХреЛ:

  • рдЖрдзрд┐рдХрд╛рд░рд┐рдХ рд╡реЗрдмрд╕рд╛рдЗрдЯ рд╕реЗ Python3.6 tgz рдлрд╝рд╛рдЗрд▓ рдбрд╛рдЙрдирд▓реЛрдб рдХрд░реЗрдВ (рдЙрджрд╛рд╣рд░рдг рдХреЗ рд▓рд┐рдП Python-3.6.6.tgz)
  • рдЗрд╕реЗ tar -xvzf Python-3.6.6.tgz рдХреЗ рд╕рд╛рде рдЕрдирдкреИрдХ рдХрд░реЗрдВ
  • cd Python-3.6.6
  • ./configure . рдЪрд▓рд╛рдПрдВ
  • рдЗрд╕реЗ рд╕реНрдерд╛рдкрд┐рдд рдХрд░рдиреЗ рдХреЗ рд▓рд┐рдП make altinstall рдЪрд▓рд╛рдПрдВ ( install рдмрдирд╛рдо altinstall рд╕реНрдкрд╖реНрдЯреАрдХрд░рдг рдпрд╣рд╛рдВ https://stackoverflow.com/questions/16018463/difference-in-details-between-make-install- рдФрд░-рдореЗрдХ-рдСрд▓реНрдЯрдЗрдВрд╕реНрдЯреЙрд▓)

рдЖрдк рдЖрдорддреМрд░ рдкрд░ /usr/local/bin рдХреЗ рддрд╣рдд рдЕрдкрдирд╛ рдирдпрд╛ рдкрд╛рдпрдерди рдЗрдВрд╕реНрдЯреЙрд▓ рдкрд╛рдПрдВрдЧреЗред рдЕрдм рдЖрдк рдЕрдЬрдЧрд░ рд╕рдВрд╕реНрдХрд░рдг рдХреЛ рдирд┐рд░реНрджрд┐рд╖реНрдЯ рдХрд░рддреЗ рд╣реБрдП рдПрдХ рдирдпрд╛ рд╡рд░реНрдЪреБрдЕрд▓рдПрдиреНрд╡ рдмрдирд╛ рд╕рдХрддреЗ рд╣реИрдВ:

  • virtualenv --python=python3.6 env3.6
  • рд╡рд░реНрдЪреБрдЕрд▓рдПрдиреНрд╡ рдореЗрдВ source env3.6/source/bin/activate рдХрдорд╛рдВрдб рдЪрд▓рд╛рдХрд░ рдкреНрд░рд╡реЗрд╢ рдХрд░реЗрдВред
  • рдХреНрд▓рд╛рд╕рд┐рдХ pip install tensorflow . рдХреЗ рд╕рд╛рде рдЯреЗрдВрд╕рд░рдлрд╝реНрд▓реЛ рд╕реНрдерд╛рдкрд┐рдд рдХрд░реЗрдВ
  • рдлрд╛рдпрджрд╛

рдпрд╣ рдХрдорд╛рдВрдб рдХрд╛ рдЙрдкрдпреЛрдЧ рдХрд░рдХреЗ рдореЗрд░реЗ рд▓рд┐рдП рдареАрдХ рдХрд╛рдо рдХрд░рддрд╛ рд╣реИ:
pip3 install https://storage.googleapis.com/tensorflow/mac/cpu/tensorflow-1.10.1-py3-none-any.whl

рдХреГрдкрдпрд╛ https://www.tensorflow.org/install/install_mac#the_url_of_the_tensorflow_python_package рд╕реЗ рджрд╕реНрддрд╛рд╡реЗрдЬрд╝ рджреЗрдЦреЗрдВ

рдореЗрд░реЗ рд▓рд┐рдпреЗ рдХрд╛рд░реНрдп рдХрд░рддрд╛ рд╣реИ!

рд╕реНрдерд╛рдкрдирд╛ рдКрдкрд░ рд╡рд░реНрдгрд┐рдд @liutingchen рджреНрд╡рд╛рд░рд╛ рдХрд╛рдо рдХрд░рддреА рд╣реИ, рд▓реЗрдХрд┐рди рдЬрдм рдореИрдВ import tensorflow рдЪрд▓рд╛рддрд╛ рд╣реВрдВ рддрдм рднреА рдореБрдЭреЗ рддреНрд░реБрдЯрд┐рдпрд╛рдВ рдорд┐рд▓рддреА рд╣реИрдВ (... рдореЗрд░рд╛ рдШрд░ рдкрде рд╣реИ):

import tensorflow

Traceback (most recent call last): File "<stdin>", line 1, in <module> File "/.../python3_env/lib/python3.7/site-packages/tensorflow/__init__.py", line 22, in <module> from tensorflow.python import pywrap_tensorflow # pylint: disable=unused-import File "/.../python3_env/lib/python3.7/site-packages/tensorflow/python/__init__.py", line 49, in <module> from tensorflow.python import pywrap_tensorflow File "/.../python3_env/lib/python3.7/site-packages/tensorflow/python/pywrap_tensorflow.py", line 58, in <module> from tensorflow.python.pywrap_tensorflow_internal import * File "/.../python3_env/lib/python3.7/site-packages/tensorflow/python/pywrap_tensorflow_internal.py", line 114 def TFE_ContextOptionsSetAsync(arg1, async): ^ SyntaxError: invalid syntax

рдореИрдВрдиреЗ рдЗрд╕рдХрд╛ рдЙрдкрдпреЛрдЧ рдХрд░рдХреЗ рд╕реНрдерд╛рдкрд┐рдд рдХрд┐рдпрд╛:
pip3 https://storage.googleapis.com/tensorflow/mac/cpu/tensorflow-1.8.0-py3-none-any.whl рдЗрдВрд╕реНрдЯреЙрд▓ рдХрд░реЗрдВ

рд╣рд╛рд▓рд╛рдВрдХрд┐, рдЪрд▓ рд░рд╣реА рд╕реНрдХреНрд░рд┐рдкреНрдЯ рд╣реЛ рд░рд╣реА рд╣реИ: "TFE_ContextOptionsSetAsync"

рдЯреНрд░реЗрд╕рдмреИрдХ (рд╕рдмрд╕реЗ рд╣рд╛рд▓рд┐рдпрд╛ рдХреЙрд▓ рдЕрдВрддрд┐рдо):
рдлрд╝рд╛рдЗрд▓ "yolo.py", рдкрдВрдХреНрддрд┐ 11, in
рдХреЗрд░рд╕ рдЖрдпрд╛рдд рдмреИрдХрдПрдВрдб рд╕реЗ K . рдХреЗ рд░реВрдк рдореЗрдВ
рдлрд╝рд╛рдЗрд▓ "/usr/local/lib/python3.7/site-packages/keras/__init__.py", рдкрдВрдХреНрддрд┐ 3, рдореЗрдВ
рд╕реЗ ред рдЖрдпрд╛рдд рдХреЗ рдмрд░реНрддрди
рдлрд╝рд╛рдЗрд▓ "/usr/local/lib/python3.7/site-packages/keras/utils/__init__.py", рд▓рд╛рдЗрди 6, рдореЗрдВ
рд╕реЗ ред рдЖрдпрд╛рдд рдХрд░реЗрдВ
рдлрд╝рд╛рдЗрд▓ "/usr/local/lib/python3.7/site-packages/keras/utils/conv_utils.py", рд▓рд╛рдЗрди 9, рдореЗрдВ
рд╕реЗ .. K . рдХреЗ рд░реВрдк рдореЗрдВ рдмреИрдХрдПрдВрдб рдЖрдпрд╛рдд рдХрд░реЗрдВ
рдлрд╝рд╛рдЗрд▓ "/usr/local/lib/python3.7/site-packages/keras/backend/__init__.py", рд▓рд╛рдЗрди 89, рдореЗрдВ
.tensorflow_backend рдЖрдпрд╛рдд рд╕реЗ *
рдлрд╝рд╛рдЗрд▓ "/usr/local/lib/python3.7/site-packages/keras/backend/tensorflow_backend.py", рд▓рд╛рдЗрди 5, рдореЗрдВ
tf . рдХреЗ рд░реВрдк рдореЗрдВ рдЯреЗрдВрд╕рд░рдлрд╝реНрд▓реЛ рдЖрдпрд╛рдд рдХрд░реЗрдВ
рдлрд╝рд╛рдЗрд▓ "/usr/local/lib/python3.7/site-packages/tensorflow/__init__.py", рд▓рд╛рдЗрди 24, рдореЗрдВ
tensorflow.python рд╕реЗ рдЖрдпрд╛рдд pywrap_tensorflow # рдкрд╛рдЗрд▓рд┐рдВрдЯ: рдЕрдХреНрд╖рдо = рдЕрдкреНрд░рдпреБрдХреНрдд-рдЖрдпрд╛рдд
рдлрд╝рд╛рдЗрд▓ "/usr/local/lib/python3.7/site-packages/tensorflow/python/__init__.py", рд▓рд╛рдЗрди 49, рдореЗрдВ
tensorflow.python рд╕реЗ рдЖрдпрд╛рдд pywrap_tensorflow
рдлрд╝рд╛рдЗрд▓ "/usr/local/lib/python3.7/site-packages/tensorflow/python/pywrap_tensorflow.py", рд▓рд╛рдЗрди 58, рдореЗрдВ
tensorflow.python.pywrap_tensorflow_internal рдЖрдпрд╛рдд рд╕реЗ *
рдлрд╝рд╛рдЗрд▓ "/usr/local/lib/python3.7/site-packages/tensorflow/python/pywrap_tensorflow_internal.py", рдкрдВрдХреНрддрд┐ 114
def TFE_ContextOptionsSetAsync(arg1, async):

рдпрд╣ рдореЗрд░реЗ рд▓рд┐рдП рдХрд╛рдо рд╣реИ
рдмрд╕ рдЗрд╕реЗ рдиреАрдЪреЗ рдЙрдкрдпреЛрдЧ рдХрд░рдиреЗ рдХрд╛ рдкреНрд░рдпрд╛рд╕ рдХрд░реЗрдВ
рдкрд╛рдЗрдк рд╕реНрдерд╛рдкрд┐рдд рдХрд░реЗрдВ https://storage.googleapis.com/tensorflow/mac/cpu/tensorflow-1.8.0-py3-none-any.whl

рдпрд╣ рдЕрдЪреНрдЫрд╛ рд╣реИ, рд▓реЗрдХрд┐рди pip install tensorflow рдЕрдЬрдЧрд░ рд╕рдВрд╕реНрдХрд░рдг 3.7 рдХреЗ рдорд╛рдзреНрдпрдо рд╕реЗ рд╡рд┐рдлрд▓ рд╣реЛ рдЧрдпрд╛ рд╣реИ, рдРрд╕рд╛ рд▓рдЧрддрд╛ рд╣реИ рдХрд┐ рдпрд╣ рдкрд╣рд┐рдпрд╛ рдкрд╛рдпрдерди 3.7 рд╕рдВрдЧрдд рдирд╣реАрдВ рд╣реИред рд╕рдорд╕реНрдпрд╛ рдпрд╣ рд╣реИ рдХрд┐ tensorflow/python/pywrap_tensorflow_internal.py рдореЗрдВ рдЪрд░ рдирд╛рдореЛрдВ рдХреЗ рд░реВрдк рдореЗрдВ async рд╢рд╛рдорд┐рд▓ рд╣реИ, рд▓реЗрдХрд┐рди async python3.7 рдореЗрдВ рдПрдХ рдХреАрд╡рд░реНрдб рдмрди рдЬрд╛рддрд╛ рд╣реИ, рдЗрд╕рд▓рд┐рдП рдпрд╣ рдЕрдм рдЙрдкрд▓рдмреНрдз рдирд╣реАрдВ рд╣реИред

рдореИрдХ рдУрдПрд╕рдПрдХреНрд╕ рдФрд░ рдкрд╛рдпрдерди 3.7 (рдПрдирд╛рдХреЛрдВрдбрд╛) рдкрд░ рдпрд╣рд╛рдВ рд╡рд╣реА рд╕рдорд╕реНрдпрд╛
def TFE_ContextOptionsSetAsync(arg1, async): ^ SyntaxError: invalid syntax

Tensorflow рдЕрдиреБрд░рдХреНрд╖рдХ рд╕реЗ рдХреЛрдИ рдЕрдкрдбреЗрдЯ? рдХреНрдпрд╛ рдЖрдк рдХреГрдкрдпрд╛ рдЗрд╕ рдореБрджреНрджреЗ рдХреЛ рдлрд┐рд░ рд╕реЗ рдЦреЛрд▓ рд╕рдХрддреЗ рд╣реИрдВ?

рдмреАрдЯреАрдбрдмреНрд▓реНрдпреВ, рдореИрдВ рд▓реЛрдЧреЛрдВ рдХреЛ рдХреЛрдВрдбрд╛ рдкрд░ рдЕрдЬрдЧрд░ рд╕рдВрд╕реНрдХрд░рдг рдХреЛ 3.5/.6 рдкрд░ рдбрд╛рдЙрдирдЧреНрд░реЗрдб рдХрд░рдиреЗ рдХреА рд╕рд▓рд╛рд╣ рджреЗрддрд╛ рд╣реВрдВ:
conda install python=3.5

рдЗрд╕реЗ #21202 рддрдХ рдареАрдХ рдХрд┐рдпрд╛ рдЬрд╛рдирд╛ рдЪрд╛рд╣рд┐рдПред

рдкреВрд░реЗ рдЙрдкрд░реЛрдХреНрдд рдХреЗ рд╕рдорд╛рди рдкреНрд░рд╢реНрди ~ рдХреНрдпрд╛ рдЗрд╕реЗ рдареАрдХ рдпрд╛ рдЕрджреНрдпрддрди рдХрд┐рдпрд╛ рдЧрдпрд╛ рд╣реИ?

TL; DR рдпрджрд┐ рдЖрдк рдЪрд╛рд╣рддреЗ рд╣реИрдВ рдХрд┐ TF рдЖрдзрд┐рдХрд╛рд░рд┐рдХ рдкреИрдХреЗрдЬ рдХрд╛рдо рдХрд░реЗрдВ, рддреЛ рдмрдЧ eigen рдФрд░ protobuf рдирдИ рд░рд┐рд▓реАрдЬрд╝ рдХрд░реЗрдВред

рдереЛрдбрд╝реА рджреЗрд░ рдХреЗ рд▓рд┐рдП рдорд╛рд╕реНрдЯрд░ рдкрд░ рддрдп рдХрд┐рдпрд╛ рдЧрдпрд╛ рд╣реИ, рд▓реЗрдХрд┐рди рдпрджрд┐ рдЖрдк py3.7 рдХрд╛ рд╕рдорд░реНрдерди рдХрд░рдиреЗ рд╡рд╛рд▓реЗ рд░рд┐рд▓реАрдЬ рдХреА рдкреНрд░рддреАрдХреНрд╖рд╛ рдХрд░ рд░рд╣реЗ рд╣реИрдВ рддреЛ рдпрд╣ рдереЛрдбрд╝рд╛ рд╕рд╛ рд╣реЛ рд╕рдХрддрд╛ рд╣реИред рддреЛ рдореВрд▓ рд░реВрдк рд╕реЗ, рдЬрдм рддрдХ рдЖрдк рдпрд╣рд╛рдВ рдПрдХ рдкреЛрд╕реНрдЯ рдирд╣реАрдВ рджреЗрдЦрддреЗ рд╣реИрдВ рдХрд┐ рдРрд╕рд╛ рд╣реБрдЖ рд╣реИ, рдЖрдзрд┐рдХрд╛рд░рд┐рдХ рдкрд╛рдЗрдк рдЗрдВрд╕реНрдЯреЙрд▓ рдХрд╛рдо рдирд╣реАрдВ рдХрд░реЗрдЧрд╛, рд▓реЗрдХрд┐рди рдЖрдк рдпрд╣ рдХрд░ рд╕рдХрддреЗ рд╣реИрдВ:

  • рд╕реНрд░реЛрдд рд╕реЗ рдирд┐рд░реНрдорд╛рдг (#21202)
  • рдПрдХ рдЕрдиреМрдкрдЪрд╛рд░рд┐рдХ рдкрд╣рд┐рдпрд╛ рдХрд╛ рдЙрдкрдпреЛрдЧ рдХрд░реЗрдВ (рдХреБрдЫ рд▓рд┐рдВрдХ рдЗрд╕ рдзрд╛рдЧреЗ рдореЗрдВ рдереЗ)

рдЕрдВрддрд┐рдо рдИрдЬреЗрди рд░рд┐рд▓реАрдЬрд╝ 23.07.2018 рдереА рдФрд░ рдЗрд╕ рдкреЛрд╕реНрдЯрд┐рдВрдЧ (AFAIK) рдХреЗ рдЕрдиреБрд╕рд╛рд░ рдЕрдВрддрд┐рдо рдкреНрд░реЛрдЯреЛрдмрдлрд╝ рд░рд┐рд▓реАрдЬрд╝ 31 рдЬреБрд▓рд╛рдИ рдереАред рджреЛрдиреЛрдВ рдХреЛ py3.7 рдХрд╛ рд╕рдорд░реНрдерди рдХрд░рдиреЗ рдХреЗ рд▓рд┐рдП рд╕реБрдзрд╛рд░ рдХреА рдЖрд╡рд╢реНрдпрдХрддрд╛ рд╣реИред рдореБрдЭреЗ рд▓рдЧрддрд╛ рд╣реИ рдХрд┐ рдЕрднреА рддрдХ рд╡реЗ рдлрд┐рдХреНрд╕ рдорд╛рд╕реНрдЯрд░ рдореЗрдВ рд╣реИрдВ рд▓реЗрдХрд┐рди рд░рд┐рд▓реАрдЬ рдореЗрдВ рдирд╣реАрдВ, рд▓реЗрдХрд┐рди рдореИрдВ рдЧрд▓рдд рд╣реЛ рд╕рдХрддрд╛ рдерд╛ред TF рдПрдХ рдЖрдзрд┐рдХрд╛рд░рд┐рдХ рдкреИрдХреЗрдЬ рдЬрд╛рд░реА рдирд╣реАрдВ рдХрд░рдиреЗ рдЬрд╛ рд░рд╣рд╛ рд╣реИ рдЬреЛ рдЕрдкреНрд░рдХрд╛рд╢рд┐рдд рдпрд╛ рдкреИрдЪ рдХрд┐рдП рдЧрдП рддреГрддреАрдп-рдкрдХреНрд╖ рд╕рд╛рдорд╛рди рдкрд░ рдирд┐рд░реНрднрд░ рдХрд░рддрд╛ рд╣реИ рдпрджрд┐ рдЗрд╕реЗ рдЯрд╛рд▓рд╛ рдЬрд╛ рд╕рдХрддрд╛ рд╣реИред

рдореИрдВ рд╡рд╛рд╕реНрддрд╡ рдореЗрдВ рдЙрди рджреЛ рдкреБрд╕реНрддрдХрд╛рд▓рдпреЛрдВ рдореЗрдВ рд╕реЗ рдХрд┐рд╕реА рдПрдХ рдореЗрдВ рд╢рд╛рдорд┐рд▓ рдирд╣реАрдВ рд╣реВрдВ рдЗрд╕рд▓рд┐рдП рдореИрдВ рдЙрди рдореБрджреНрджреЛрдВ рдХреА рд╕реНрдерд┐рддрд┐ рдХреЗ рдмрд╛рд░реЗ рдореЗрдВ рднреА рдЙрддреНрд╕реБрдХ рд╣реВрдВред TF рд╕рдорд╕реНрдпрд╛рдУрдВ рдХреЛ рдареАрдХ рдХрд░рдирд╛ рдФрд░ рдЬрд╛рд░реА рдХрд░рдирд╛ рдЖрд╕рд╛рди рд╣реИред рддреГрддреАрдп-рдкрдХреНрд╖ рдкреБрд╕реНрддрдХрд╛рд▓рдп-рдЗрддрдирд╛ рдирд╣реАрдВред рдпрджрд┐ рдпрд╣ рд╕рд┐рд░реНрдл TF рд╕рдорд╕реНрдпрд╛ рдереА, рддреЛ рдХреБрдЫ рд╣реА рд╕рдордп рдореЗрдВ Python 3.7 рдХреЛ рдЖрдзрд┐рдХрд╛рд░рд┐рдХ рд░реВрдк рд╕реЗ рд╕рдорд░реНрдерди рджрд┐рдпрд╛ рдЧрдпрд╛ рд╣реЛрддрд╛ред

рдХреБрдЫ рд╕рдВрдмрдВрдзрд┐рдд рд▓рд┐рдВрдХ рдорд┐рд▓реЗ:
https://github.com/protocolbuffers/protobuf/pull/4940
https://github.com/protocolbuffers/protobuf/pull/4862

рдкрддрд╛ рдирд╣реАрдВ рдЯреАрдмреАрдПрдЪ рдХреА рдИрдЬрд┐рди рд╕реНрдерд┐рддрд┐ рдХреНрдпрд╛ рд╣реИред

рдЕрднреА рднреА рдкрд╛рдЗрдк рдХреЗ рд╕рд╛рде Python3.7 рдкрд░ TensorFlow рд╕реНрдерд╛рдкрд┐рдд рдХрд░рдиреЗ рдореЗрдВ рдЕрд╕рдорд░реНрдеред рд╢рд╛рдпрдж рдЗрд╕ рдореБрджреНрджреЗ рдХреЛ рдЦреБрд▓рд╛ рд░рдЦрдирд╛ рдЪрд╛рд╣рд┐рдП рдЬрдм рддрдХ рдХрд┐ рдПрдХ рдкрд╣рд┐рдпрд╛ рдЬрд╛рд░реА рди рд╣реЛ рдЬрд╛рдПред

рдореИрдХреЛ (рдореЗрд░реЗ рдЬреИрд╕реЗ) рдкрд░ рдЙрди рд▓реЛрдЧреЛрдВ рдХреЗ рд▓рд┐рдП, рдЖрдк brew to (рдЕрд╕реНрдерд╛рдпреА рд░реВрдк рд╕реЗ) 3.6.5: рд▓рд┐рдВрдХ рдкрд░ рд╡рд╛рдкрд╕ рд╕реНрд╡рд┐рдЪ рдХрд╛ рдЙрдкрдпреЛрдЧ рдХрд░ рд╕рдХрддреЗ рд╣реИрдВред рдореБрдЭреЗ рдХрд░рдирд╛ рдерд╛
brew link --overwrite python рдЗрд╕рдХреЗ рдЕрд▓рд╛рд╡рд╛ рдФрд░ рдлрд┐рд░
pip install --upgrade tensorflow рдареАрдХ рдХрд╛рдо рдХрд░рддрд╛ рд╣реИ

рдЕрдЪреНрдЫрд╛ рдХрд┐рдпрд╛ рдзрдиреНрдпрд╡рд╛рдж, рдЯреЗрдВрд╕рд░рдлрд╝реНрд▓реЛ рдЕрдм рдЕрдЬрдЧрд░ 3.7 рдореЗрдВ рдЙрдкрд▓рдмреНрдз рдирд╣реАрдВ рд╣реИред рд░рд┐рд▓реАрдЬ рдХрд╛ рдЗрдВрддрдЬрд╛рд░ рдХрд░ рд░рд╣реЗ рд╣реИрдВред

рдЕрдиреНрдп рд▓реЛрдЧреЛрдВ рдХреЗ рд▓рд┐рдП рдЬреЛ рдЗрд╕ рд╕рдорд╕реНрдпрд╛ рдХрд╛ рд╕рд╛рдордирд╛ рдХрд░ рд╕рдХрддреЗ рд╣реИрдВ: рдЕрдЬрдЧрд░ 3.6 :-) рдХреЗ рд╕рд╛рде рдПрдХ рдирдпрд╛ рдПрдирд╡реА рдмрдирд╛рдПрдВред

рдореИрдВ tensorflow рдЪрд▓рд╛рдиреЗ рдХреЗ рд▓рд┐рдП рдбреЙрдХрд░ рдХрд╛ рдЙрдкрдпреЛрдЧ рдХрд░рддрд╛ рд╣реВрдВ, рдЗрд╕рд▓рд┐рдП рдпрд╣ рдореЗрд░реЗ рд▓рд┐рдП рдХреЛрдИ рд╕рдорд╕реНрдпрд╛ рдирд╣реАрдВ рд╣реИред рдХреЗрд╡рд▓ рдЗрд╕ рдУрд░ рдЗрд╢рд╛рд░рд╛ рдХрд░рддреЗ рд╣реБрдП рдХрд┐ рд╣рдореЗрдВ рд╣рдореЗрд╢рд╛ рдХрд┐рд╕реА рднрд╛рд╖рд╛ рдХреЗ рдирд╡реАрдирддрдо рдорд╛рдирдХ рдХрд╛ рдкрд╛рд▓рди рдХрд░рдирд╛ рдЪрд╛рд╣рд┐рдП, рдпрд╣рд╛рдВ рддрдХ тАЛтАЛрдХрд┐ рд╡рд╣ рддреЗрдЬреА рд╕реЗ рдмрджрд▓рддреА рднреА рд╣реИред

рдкрд┐рдк рдЗрдВрд╕реНрдЯрд╛рд▓ рдЯреЗрдВрд╕рд░рдлреНрд▓реЛ рдХреЗ рдмрд╛рд░реЗ рдореЗрдВ рдХреЛрдИ рдЕрдкрдбреЗрдЯ?

рдЪреАрдЬреЛрдВ рдХреЛ 3.7 рдХреЗ рд╕рд╛рде рдХрд╛рдо рдХрд░рдиреЗ рдХреЗ рд▓рд┐рдП рдпрд╣рд╛рдВ рдПрдХ рдЖрд╕рд╛рди рдЪрд╛рд▓ рд╣реИ, рд▓реЗрдХрд┐рди рдХреЗрд╡рд▓ рдЕрдЧрд░ рдЖрдк рдПрдХ рдХрдард┐рди рдХреАрдЯ * рдЖрд░ рд╣реИрдВ рдЬреЛ рдЪреАрдЬреЗрдВ рдЯреВрдЯрдиреЗ рдкрд░ рдмрдХрд╡рд╛рд╕ рдирд╣реАрдВ рдХрд░рддреЗ рд╣реИрдВ:

wget https://storage.googleapis.com/tensorflow/linux/cpu/tensorflow-1.11.0-cp36-cp36m-linux_x86_64.whl

(рдпрд╛ рдЬреЛ рднреА рд╕рд╣реА рдкрд╣рд┐рдпрд╛ рд╣реИ)
рддрдерд╛:

cp tensorflow-1.11.0-cp36-cp36m-linux_x86_64.whl tensorflow-1.11.0-cp37-cp37m-linux_x86_64.whl
pip install tensorflow-1.11.0-cp37-cp37m-linux_x86_64.whl

рдЪреАрдЬреЛрдВ рдХреЛ 3.7 рдХреЗ рд╕рд╛рде рдХрд╛рдо рдХрд░рдиреЗ рдХреЗ рд▓рд┐рдП рдпрд╣рд╛рдВ рдПрдХ рдЖрд╕рд╛рди рдЪрд╛рд▓ рд╣реИ, рд▓реЗрдХрд┐рди рдХреЗрд╡рд▓ рдЕрдЧрд░ рдЖрдк рдПрдХ рдХрдард┐рди рдХреАрдЯ * рдЖрд░ рд╣реИрдВ рдЬреЛ рдЪреАрдЬреЗрдВ рдЯреВрдЯрдиреЗ рдкрд░ рдмрдХрд╡рд╛рд╕ рдирд╣реАрдВ рдХрд░рддреЗ рд╣реИрдВ:

wget https://storage.googleapis.com/tensorflow/linux/cpu/tensorflow-1.11.0-cp36-cp36m-linux_x86_64.whl

(рдпрд╛ рдЬреЛ рднреА рд╕рд╣реА рдкрд╣рд┐рдпрд╛ рд╣реИ)
рддрдерд╛:

cp tensorflow-1.11.0-cp36-cp36m-linux_x86_64.whl tensorflow-1.11.0-cp37-cp37m-linux_x86_64.whl
pip install tensorflow-1.11.0-cp37-cp37m-linux_x86_64.whl

рд╣рд╛рдп, рд╣рд╛рд░реНрдкреЛрдиред

рдореИрдВрдиреЗ рдЖрдкрдХреА рдЪрд╛рд▓ рдХреА рдХреЛрд╢рд┐рд╢ рдХреА, рд▓реЗрдХрд┐рди рдореБрдЭреЗ рдпрд╣ рд╕рдВрджреЗрд╢ рдорд┐рд▓рд╛:

/home/adriano/anaconda3/lib/python3.7/importlib/_bootstrap.py:219: рд░рдирдЯрд╛рдЗрдо рдЪреЗрддрд╛рд╡рдиреА: рдореЙрдбреНрдпреВрд▓ 'tensorflow.python.framework.fast_tensor_util' рдХрд╛ рд╕рдВрдХрд▓рди рд╕рдордп рд╕рдВрд╕реНрдХрд░рдг 3.6 рд░рдирдЯрд╛рдЗрдо рд╕рдВрд╕реНрдХрд░рдг 3.7 рд╕реЗ рдореЗрд▓ рдирд╣реАрдВ рдЦрд╛рддрд╛
рд╡рд╛рдкрд╕реА рдПрдл (рдЖрд░реНрдЧ , * рдХрд┐рд▓реЛрд╡рд╛рдЯ)

рд▓реЗрдХрд┐рди... рдореИрдВрдиреЗ рджреЗрдЦрд╛ рдХрд┐ рдЬрдм рдореИрдВ рдЕрдкрдиреА рд╕реНрдХреНрд░рд┐рдкреНрдЯ рдореЗрдВ рдЯреЗрдВрд╕рд░рдлрд╝реНрд▓реЛ рдореЙрдбреНрдпреВрд▓ рдЖрдпрд╛рдд рдХрд░рддрд╛ рд╣реВрдБ рддреЛ рдореБрдЭреЗ рдХреЛрдИ рддреНрд░реБрдЯрд┐ рд╕рдВрджреЗрд╢ рдирд╣реАрдВ рдорд┐рд▓рддрд╛ рд╣реИред

рдкрд╛рдЗрдк рдЗрдВрд╕реНрдЯрд╛рд▓ --рдЕрдкрдЧреНрд░реЗрдб https://storage.googleapis.com/tensorflow/mac/cpu/tensorflow-0.12.0-py3-none-any.whl
рдЬреЛ рдореИрдВ рдЙрдкрдпреЛрдЧ рдХрд░рддрд╛ рд╣реВрдВ, рд╡рд┐рдВрдбреЛрдЬрд╝ рдкрд░ рдкрд╛рдпрдерди 3.7 рдФрд░ рдпрд╣ рдХрд╛рдо рдХрд░рддрд╛ рд╣реИред

рдкрд╛рдЗрдк рдЗрдВрд╕реНрдЯрд╛рд▓ --рдЕрдкрдЧреНрд░реЗрдб https://storage.googleapis.com/tensorflow/mac/cpu/tensorflow-0.12.0-py3-none-any.whl
рдЬреЛ рдореИрдВ рдЙрдкрдпреЛрдЧ рдХрд░рддрд╛ рд╣реВрдВ, рд╡рд┐рдВрдбреЛрдЬрд╝ рдкрд░ рдкрд╛рдпрдерди 3.7 рдФрд░ рдпрд╣ рдХрд╛рдо рдХрд░рддрд╛ рд╣реИред

рдореЗрд░реЗ рд▓рд┐рдП рдХрд╛рдо рдХрд┐рдпрд╛, рдореИрдХ рдкрд░ рдЕрдЬрдЧрд░ 3.7ред

рдкрд╛рдЗрдк рдЗрдВрд╕реНрдЯрд╛рд▓ --рдЕрдкрдЧреНрд░реЗрдб https://storage.googleapis.com/tensorflow/mac/cpu/tensorflow-0.12.0-py3-none-any.whl
рдЬреЛ рдореИрдВ рдЙрдкрдпреЛрдЧ рдХрд░рддрд╛ рд╣реВрдВ, рд╡рд┐рдВрдбреЛрдЬрд╝ рдкрд░ рдкрд╛рдпрдерди 3.7 рдФрд░ рдпрд╣ рдХрд╛рдо рдХрд░рддрд╛ рд╣реИред

рдореИрдХ рдкрд░ рднреА рдореЗрд░реЗ рд▓рд┐рдП рдХрд╛рдо рдХрд┐рдпрд╛, рдзрдиреНрдпрд╡рд╛рдж!

рдмрдВрдж рдХреНрдпреЛрдВ рд╣реИ?
pip install tensorflow рдЕрднреА рднреА рдХрд╛рдо рдирд╣реАрдВ рдХрд░ рд░рд╣рд╛ рд╣реИред

pip install --upgrade https://storage.googleapis.com/tensorflow/mac/cpu/tensorflow-0.12.0-py3-none-any.whl рд╕рдлрд▓, рд▓реЗрдХрд┐рди рд╕рд░рд▓ n00b-рдЕрдиреБрдХреВрд▓ рдЗрдВрд╕реНрдЯреЙрд▓ рдирд╣реАрдВред

рдПрдХ рддрд░рдл: [ tf.enable_eager_execution() рд╣рдЯрд╛ рджрд┐рдпрд╛ рдЧрдпрд╛ рд╣реИ, рдЕрдЧрд░ рд╡рд╣ рдЖрдкрдХреА рдЪреЗрдХрд╕реНрдХреНрд░рд┐рдкреНрдЯ рдореЗрдВ рд╣реИ;
AttributeError: module 'tensorflow' has no attribute 'enable_eager_execution' ]

рдкрд╛рдЗрдк рдЗрдВрд╕реНрдЯрд╛рд▓ --рдЕрдкрдЧреНрд░реЗрдб https://storage.googleapis.com/tensorflow/mac/cpu/tensorflow-0.12.0-py3-none-any.whl

рдХреНрдпрд╛ рдпрд╣ рд╕рдВрд╕реНрдХрд░рдг 0.12 рдпрд╛ 1.12 рд╣реИ?

рдУрдПрд╕ рдПрдХреНрд╕ рдЕрдЬрдЧрд░ 3.7 рдкрд░ рдХрд╛рдо рдирд╣реАрдВ рдХрд░ рд░рд╣рд╛ рдкрд╛рдЗрдк рд╕реНрдерд╛рдкрд┐рдд рдЯреЗрдВрд╕рд░рдлреНрд▓реЛ
рдЗрд╕ рдереНрд░реЗрдб рд╕реЗ рдлрд┐рдХреНрд╕ рд╕рдВрд╕реНрдХрд░рдг 0.12.0 рджрд┐рдЦрд╛рддрд╛ рд╣реИ?

tf . рдХреЗ рд░реВрдк рдореЗрдВ рдЯреЗрдВрд╕рд░рдлрд╝реНрд▓реЛ рдЖрдпрд╛рдд рдХрд░реЗрдВ
tf.__рд╕рдВрд╕реНрдХрд░рдг__
'0.12.0' ??

рдмрдВрдж рдХреНрдпреЛрдВ рд╣реИ?
pip install tensorflow рдЕрднреА рднреА рдХрд╛рдо рдирд╣реАрдВ рдХрд░ рд░рд╣рд╛ рд╣реИред

pip install --upgrade https://storage.googleapis.com/tensorflow/mac/cpu/tensorflow-0.12.0-py3-none-any.whl рд╕рдлрд▓, рд▓реЗрдХрд┐рди рд╕рд░рд▓ n00b-рдЕрдиреБрдХреВрд▓ рдЗрдВрд╕реНрдЯреЙрд▓ рдирд╣реАрдВред

рдПрдХ рддрд░рдл: [ tf.enable_eager_execution() рд╣рдЯрд╛ рджрд┐рдпрд╛ рдЧрдпрд╛ рд╣реИ, рдЕрдЧрд░ рд╡рд╣ рдЖрдкрдХреА рдЪреЗрдХрд╕реНрдХреНрд░рд┐рдкреНрдЯ рдореЗрдВ рд╣реИ;
AttributeError: module 'tensorflow' has no attribute 'enable_eager_execution' ]

рдореИрдВ рдЙрд╕реЗ рд╕рдорд░реНрдерди рджреЗрддрд╛ рд╣реВрдБред рдпрд╣ 0.12. рд╕рдВрд╕реНрдХрд░рдг рднреА рдХреЗрд░рд╕ рдХреЗ рд╕рд╛рде рд╕рдВрдЧрддрддрд╛ рдХреЛ рддреЛрдбрд╝рддрд╛ рд╣реИред рдпрд╣рд╛рдВ рддрдХ тАЛтАЛрдХрд┐ рдЬрд╣рд╛рдВ рдХреЗрд░рд╕ рдХреЛрдИ рдЪреЗрддрд╛рд╡рдиреА рдпрд╛ рддреНрд░реБрдЯрд┐ рд╕рдВрджреЗрд╢ рдирд╣реАрдВ рдлреЗрдВрдХрддрд╛ рд╣реИ, рд▓реЗрдХрд┐рди рд╕реАрдЦрдиреЗ рд╕реЗ рдЗрдирдХрд╛рд░ рдХрд░рддрд╛ рд╣реИред

рдореИрдВрдиреЗ рдХреЛрдВрдбрд╛ (рдорд┐рдирд┐рдХреЛрдВрдбрд╛ 3 рд╕рдЯреАрдХ рд╣реЛрдиреЗ рдХреЗ рд▓рд┐рдП) рд╕реНрдерд╛рдкрд┐рдд рдХрд░рдХреЗ рдЕрдкрдиреА рд╕рдорд╕реНрдпрд╛ рд╣рд▓ рдХреА рд╣реИ, рдлрд┐рд░ рдЗрд╕рдХрд╛ рдЙрдкрдпреЛрдЧ рдкрд╛рдЗрдерди 3.6.5 рдХреЗ рд╕рд╛рде рд╡рд░реНрдЪреБрдЕрд▓рдПрдиреНрд╡ рд╕реНрдерд╛рдкрд┐рдд рдХрд░рдиреЗ рдХреЗ рд▓рд┐рдП рдХрд┐рдпрд╛ рд╣реИ, рдФрд░ рдЙрд╕рдореЗрдВ рдЯреАрдПрдл рдФрд░ рдХреЗрд░рд╕ рджреЛрдиреЛрдВ рд╕реНрдерд╛рдкрд┐рдд рдХрд┐рдП рдЧрдП рд╣реИрдВред рдЕрдм рдореЗрд░рд╛ рдореЙрдбрд▓ рд╕реАрдЦрддрд╛ рд╣реИред

рдореИрдВ рд▓рдЧрднрдЧ рдирд┐рд╢реНрдЪрд┐рдд рд╣реВрдВ рдХрд┐ 0.12 рдореВрд▓ рдЯрд┐рдкреНрдкрдгреА рдореЗрдВ рдЯрд╛рдЗрдкреЛ рдХреЗ рдХрд╛рд░рдг рд╣реИред https://storage.googleapis.com/tensorflow/mac/cpu/tensorflow-1.12.0-py3-none-any.whl рдЗрдВрд╕реНрдЯреЙрд▓ рдХрд░рдирд╛ рдореЗрд░реЗ рд▓рд┐рдП рдХрд╛рдо рдХрд░рддрд╛ рд╣реИ:

Python 3.7.1 (v3.7.1:260ec2c36a, Oct 20 2018, 03:13:28) 
[Clang 6.0 (clang-600.0.57)] on darwin
Type "help", "copyright", "credits" or "license" for more information.
>>> import tensorflow
/Library/Frameworks/Python.framework/Versions/3.7/lib/python3.7/importlib/_bootstrap.py:219: RuntimeWarning: compiletime version 3.6 of module 'tensorflow.python.framework.fast_tensor_util' does not match runtime version 3.7
  return f(*args, **kwds)
>>> tensorflow.__version__
'1.12.0'
>>> 

рдзреНрдпрд╛рди рджреЗрдВ рдХрд┐ RuntimeWarning рд╕рд┐рд░реНрдл рдПрдХ рдЪреЗрддрд╛рд╡рдиреА рд╣реИред

рдореИрдВ рд▓рдЧрднрдЧ рдирд┐рд╢реНрдЪрд┐рдд рд╣реВрдВ рдХрд┐ 0.12 рдореВрд▓ рдЯрд┐рдкреНрдкрдгреА рдореЗрдВ рдЯрд╛рдЗрдкреЛ рдХреЗ рдХрд╛рд░рдг рд╣реИред https://storage.googleapis.com/tensorflow/mac/cpu/tensorflow-1.12.0-py3-none-any.whl рдЗрдВрд╕реНрдЯреЙрд▓ рдХрд░рдирд╛ рдореЗрд░реЗ рд▓рд┐рдП рдХрд╛рдо рдХрд░рддрд╛ рд╣реИ:

Python 3.7.1 (v3.7.1:260ec2c36a, Oct 20 2018, 03:13:28) 
[Clang 6.0 (clang-600.0.57)] on darwin
Type "help", "copyright", "credits" or "license" for more information.
>>> import tensorflow
/Library/Frameworks/Python.framework/Versions/3.7/lib/python3.7/importlib/_bootstrap.py:219: RuntimeWarning: compiletime version 3.6 of module 'tensorflow.python.framework.fast_tensor_util' does not match runtime version 3.7
  return f(*args, **kwds)
>>> tensorflow.__version__
'1.12.0'
>>> 

рдзреНрдпрд╛рди рджреЗрдВ рдХрд┐ RuntimeWarning рд╕рд┐рд░реНрдл рдПрдХ рдЪреЗрддрд╛рд╡рдиреА рд╣реИред

рд╕рдордЭ рдЧрдпрд╛, thx! рдореЗрд░реЗ рд▓рд┐рдП рдХрд╛рдо рдХрд┐рдпрд╛, рдмрд┐рдирд╛ рдХрд┐рд╕реА рдЕрдВрддрд░ рдХреЗ рд╡рд╣реА рдЪреЗрддрд╛рд╡рдиреАред

$ pip --version
pip 18.1 

$ python --version
Python 3.7.0

$ pip install tensorflow
Collecting tensorflow
  Could not find a version that satisfies the requirement tensorflow (from versions: )
No matching distribution found for tensorflow

$ pip search tensorflow
tensorflow (1.12.0)       - TensorFlow is an open source machine learning framework for everyone.

$ pip install --upgrade https://storage.googleapis.com/tensorflow/mac/cpu/tensorflow-1.12.0-py3-none-any.whl

$ python
Python 3.7.0 (default, Oct 18 2018, 15:08:12)
[Clang 10.0.0 (clang-1000.10.44.2)] on darwin
Type "help", "copyright", "credits" or "license" for more information.
>>> import tensorflow
/lib/python3.7/importlib/_bootstrap.py:219: RuntimeWarning: compiletime version 3.6 of module 'tensorflow.python.framework.fast_tensor_util' does not match runtime version 3.7
  return f(*args, **kwds)
>>> tensorflow.__version__
'1.12.0'

рдкрд┐рдк рдЗрдВрд╕реНрдЯреЙрд▓ --рдЕрдкрдЧреНрд░реЗрдб https://storage.googleapis.com/tensorflow/mac/cpu/tensorflow-1.12.0-py3-none-any.whl
рдЕрдЬрдЧрд░ 3.7.1 . рдХреЗ рд▓рд┐рдП рдХрд╛рдо рдирд╣реАрдВ рдХрд░рддрд╛

рдореИрдВрдиреЗ рдереЛрдбрд╝реА рджреЗрд░ рдХреА рдЦреЛрдЬ рдХреЗ рдмрд╛рдж рдЗрд╕реЗ рдХрд╛рдо рдХрд░рдиреЗ рдХреЗ рд▓рд┐рдП рдкрд╛рдпрд╛ред рдореИрдВ рдЗрд╕реЗ рдпрд╣рд╛рдБ рдЫреЛрдбрд╝ рджреВрдБрдЧрд╛ рдХреНрдпреЛрдВрдХрд┐ рдпрд╣ рдХрд┐рд╕реА рдХреЗ рд▓рд┐рдП рдЙрдкрдпреЛрдЧреА рд╣реЛ рд╕рдХрддрд╛ рд╣реИред

Tensorflow рдЖрдЬ рддрдХ рдХреЗрд╡рд▓ Python 3.6 рдХрд╛ рд╕рдорд░реНрдерди рдХрд░рддрд╛ рд╣реИред рдЖрдк рдЕрдкрдиреЗ рдорд╛рдирдХ рдХреЗ рд╕рд╛рде рдЕрдЬрдЧрд░ рдХрд╛ рдПрдХ рдЕрд▓рдЧ рд╕рдВрд╕реНрдХрд░рдг рд╕реНрдерд╛рдкрд┐рдд рдХрд░ рд╕рдХрддреЗ рд╣реИрдВред рдЖрдкрдХреЛ:

* Download the Python3.6 tgz file from the official website (eg. Python-3.6.6.tgz)

* Unpack it with `tar -xvzf Python-3.6.6.tgz`

* `cd Python-3.6.6`

* run `./configure`

* run `make altinstall` to install it (`install` vs `altinstall` explanation here https://stackoverflow.com/questions/16018463/difference-in-details-between-make-install-and-make-altinstall)

рдЖрдк рдЖрдорддреМрд░ рдкрд░ /usr/local/bin рдХреЗ рддрд╣рдд рдЕрдкрдирд╛ рдирдпрд╛ рдкрд╛рдпрдерди рдЗрдВрд╕реНрдЯреЙрд▓ рдкрд╛рдПрдВрдЧреЗред рдЕрдм рдЖрдк рдЕрдЬрдЧрд░ рд╕рдВрд╕реНрдХрд░рдг рдХреЛ рдирд┐рд░реНрджрд┐рд╖реНрдЯ рдХрд░рддреЗ рд╣реБрдП рдПрдХ рдирдпрд╛ рд╡рд░реНрдЪреБрдЕрд▓рдПрдиреНрд╡ рдмрдирд╛ рд╕рдХрддреЗ рд╣реИрдВ:

* `virtualenv --python=python3.6 env3.6`

* Get into the virtualenv running the command `source env3.6/source/bin/activate`.

* Install tensorflow with the classic `pip install tensorflow`

* Profit

рд╡рд░реНрдЪреБрдЕрд▓ рдПрдирд╡реА рдХрдорд╛рдВрдб рдЪрд▓рд╛рдиреЗ рдореЗрдВ рдПрдХ рдЯрд╛рдЗрдкреЛ рд╣реИред рдпрд╣ source env3.6/bin/activate . рд╣реЛрдирд╛ рдЪрд╛рд╣рд┐рдП

рдореЗрд░реЗ рдкрд╛рд╕ рдПрдХ рд╣реА рдореБрджреНрджрд╛ рдерд╛ред рдЗрд╕реЗ рдЗрд╕реНрддреЗрдорд╛рд▓ рдХрд░реЗ:

  • рдЕрдкрдиреЗ рдЕрдЬрдЧрд░ рдХреЛ 3.7 рд╕реЗ 3.6 рддрдХ рдбрд╛рдЙрдирдЧреНрд░реЗрдб рдХрд░реЗрдВ (рдпрджрд┐ conda virtual env рдХрд╛ рдЙрдкрдпреЛрдЧ рдХрд░ рд░рд╣реЗ рд╣реИрдВ, рддреЛ 'conda create -n yourenvname python==3.6 anaconda' рдХреЗ рд╕рд╛рде рдПрдХ рдирдпрд╛ рд╡рд░реНрдЪреБрдЕрд▓ env рдмрдирд╛рдПрдВ)
  • рдлрд┐рд░ 'рдкрд╛рдЗрдк рдЗрдВрд╕реНрдЯреЙрд▓ рдЯреЗрдВрд╕рд░рдлрд╝реНрд▓реЛ-рдЬреАрдкреАрдпреВ'

рдХрд╛рдо рдХрд┐рдпрд╛ рдЬрд╛рдирд╛ рдЪрд╛рд╣рд┐рдП..

рдпрджрд┐ рдЖрдк x86_64 рдФрд░ рдХрд┐рд╕реА рднреА Linux рдбрд┐рд╕реНрдЯреНрд░реЛ рдХрд╛ рдЙрдкрдпреЛрдЧ рдХрд░ рд░рд╣реЗ рд╣реИрдВ, рддреЛ рдЖрдк рдЙрдкрдпреЛрдЧреА рд╣реЛрдиреЗ рдХреЗ рд▓рд┐рдП GitHub рд░реЗрдкреЛ рдХреЗ рдиреАрдЪреЗ рдкрд╛ рд╕рдХрддреЗ рд╣реИрдВред

https://github.com/evdcush/TensorFlow-wheels

рдореИрдВрдиреЗ рдЕрдкрдиреА рдорд╢реАрди рдореЗрдВ рдХреЛрд╢рд┐рд╢ рдХреА рдФрд░ рд╕рдлрд▓рддрд╛рдкреВрд░реНрд╡рдХ рд╕реНрдерд╛рдкрд┐рдд рдХрд┐рдпрд╛ рд╣реИред

$ wget https://github.com/evdcush/TensorFlow-wheels/releases/download/tf-1.12.0-py37-cpu-ivybridge/tensorflow-1.12.0-cp37-cp37m-linux_x86_64.whl
$ pip install --user tensorflow-1.12.0-cp37-cp37m-linux_x86_64.whl  
$ python
Python 3.7.1 (default, Oct 22 2018, 10:41:28) 
[GCC 8.2.1 20180831] on linux
Type "help", "copyright", "credits" or "license" for more information.
>>> import tensorflow
>>> print(tensorflow.__version__)
1.12.0
>>>

рдпрджрд┐ рдЖрдк Tensorflow + GPU рдХрд╛ рдирд┐рд░реНрдорд╛рдг рдХрд░рдирд╛ рдЪрд╛рд╣рддреЗ рд╣реИрдВ, рддреЛ рдЖрдк рдЪреБрди рд╕рдХрддреЗ рд╣реИрдВ рдХрд┐ рдЖрдк рдХреМрди рд╕рд╛ Github рд░реЗрдкреЛ рдкреЗрдЬ рдХреЗ рдКрдкрд░ рдЙрдкрдпреЛрдЧ рдХрд░рдирд╛ рдЪрд╛рд╣рддреЗ рд╣реИрдВред

рдЪреАрдпрд░реНрд╕ред

рдкрд╛рдЗрдк рдЗрдВрд╕реНрдЯрд╛рд▓ --рдЕрдкрдЧреНрд░реЗрдб https://storage.googleapis.com/tensorflow/mac/cpu/tensorflow-0.12.0-py3-none-any.whl
рдЬреЛ рдореИрдВ рдЙрдкрдпреЛрдЧ рдХрд░рддрд╛ рд╣реВрдВ, рд╡рд┐рдВрдбреЛрдЬрд╝ рдкрд░ рдкрд╛рдпрдерди 3.7 рдФрд░ рдпрд╣ рдХрд╛рдо рдХрд░рддрд╛ рд╣реИред

рдмрд╕ рдЖрдк рдпрд╣ рдЬрд╛рдирдирд╛ рдЪрд╛рд╣рддреЗ рд╣реИрдВ рдХрд┐ рдЗрд╕рдиреЗ рдХрд╛рд▓реНрдкрдирд┐рдХ рд░реВрдк рд╕реЗ рдХрд╛рдо рдХрд┐рдпрд╛ред рдзрдиреНрдпрд╡рд╛рдж!

рд▓рд┐рдирдХреНрд╕ рдХреЗ рд▓рд┐рдП рдХреЛрдИ рд╕реБрдЭрд╛рд╡?

pip install --upgrade https://storage.googleapis.com/tensorflow/linux/gpu/tensorflow_gpu-1.12.0-cp36-cp36m-linux_x86_64.whl

tensorflow-1.12.0-cp36-cp36m-linux_x86_64.whl рдЗрд╕ рдкреНрд▓реЗрдЯрдлреЙрд░реНрдо рдкрд░ рд╕рдорд░реНрдерд┐рдд рд╡реНрд╣реАрд▓ рдирд╣реАрдВ рд╣реИ ред

рдХреЛрдИ рдореЗрд░реА рдорджрдж рдХрд░ рд╕рдХрддрд╛ рд╣реИ!
рдореИрдВ рдкрд╣рд▓реЗ рд╕реЗ рд╣реА рдЕрдЬрдЧрд░ 3.7.1 рд╕реНрдерд╛рдкрд┐рдд рдХрд░ рдЪреБрдХрд╛ рд╣реВрдВ, рдЯреЗрдВрд╕рд░-рдлреНрд▓реЛ рд╕реНрдерд╛рдкрд┐рдд рдХрд░рдирд╛ рдЪрд╛рд╣рддрд╛ рд╣реВрдВ, рд▓реЗрдХрд┐рди рдореИрдВ рдирд╣реАрдВ рдХрд░ рд╕рдХрддрд╛
рдХреЛрдИ рднреА рдЬреЛ рдореБрдЭреЗ рдмрддрд╛ рд╕рдХрддрд╛ рд╣реИ рдХрд┐ рдЗрд╕ рд╕рдорд╕реНрдпрд╛ рдХреЛ рдХреИрд╕реЗ рдареАрдХ рдХрд┐рдпрд╛ рдЬрд╛рдП

рдХреЛрдИ рдореЗрд░реА рдорджрдж рдХрд░ рд╕рдХрддрд╛ рд╣реИ!
рдореИрдВ рдкрд╣рд▓реЗ рд╕реЗ рд╣реА рдЕрдЬрдЧрд░ 3.7.1 рд╕реНрдерд╛рдкрд┐рдд рдХрд░ рдЪреБрдХрд╛ рд╣реВрдВ, рдЯреЗрдВрд╕рд░-рдлреНрд▓реЛ рд╕реНрдерд╛рдкрд┐рдд рдХрд░рдирд╛ рдЪрд╛рд╣рддрд╛ рд╣реВрдВ, рд▓реЗрдХрд┐рди рдореИрдВ рдирд╣реАрдВ рдХрд░ рд╕рдХрддрд╛
рдХреЛрдИ рднреА рдЬреЛ рдореБрдЭреЗ рдмрддрд╛ рд╕рдХрддрд╛ рд╣реИ рдХрд┐ рдЗрд╕ рд╕рдорд╕реНрдпрд╛ рдХреЛ рдХреИрд╕реЗ рдареАрдХ рдХрд┐рдпрд╛ рдЬрд╛рдП

рд╣рд╛рдп рдирд╛рд╕рд░, рджреБрд░реНрднрд╛рдЧреНрдп рд╕реЗ Tensorflow рдЕрднреА рддрдХ Python v3.7 рдХреЗ рд╕рд╛рде рдХрд╛рдо рдирд╣реАрдВ рдХрд░рддрд╛ рд╣реИред рдЖрдкрдХреЛ рдЕрдирдЗрдВрд╕реНрдЯреЙрд▓ рдХрд░рдирд╛ рд╣реЛрдЧрд╛, Python рдХреА рд╡реЗрдмрд╕рд╛рдЗрдЯ рдкрд░ рдЬрд╛рдирд╛ рд╣реЛрдЧрд╛ рдФрд░ Python v3.6 рдЗрдВрд╕реНрдЯреЙрд▓ рдХрд░рдирд╛ рд╣реЛрдЧрд╛ред рдЕрдЪреНрдЫреА рдмрд╛рдд рдпрд╣ рд╣реИ рдХрд┐ рдХрдИ рдЕрдВрддрд░ рдирд╣реАрдВ рд╣реИрдВ :) рдмрд╕ рдЕрдкрдиреЗ рд╕рднреА рдкреИрдХреЗрдЬреЛрдВ рдХреЛ рдлрд┐рд░ рд╕реЗ рд╕реНрдерд╛рдкрд┐рдд рдХрд░рдирд╛ рд╕реБрдирд┐рд╢реНрдЪрд┐рдд рдХрд░реЗрдВред рдореИрдВрдиреЗ рд╡реНрдпрдХреНрддрд┐рдЧрдд рд░реВрдк рд╕реЗ рдЕрдкрдиреЗ рд╕рднреА рдкреИрдХреЗрдЬреЛрдВ рдХрд╛ рдПрдХ рд╕реНрдХреНрд░реАрдирд╢реЙрдЯ рд▓рд┐рдпрд╛ (рдпрд╛рдиреА рдХрдорд╛рдВрдб рдкреНрд░реЙрдореНрдкреНрдЯ рдореЗрдВ рдЬрд╛рдПрдВ, рдФрд░ рдкрд╛рдЗрдк рд╕реВрдЪреА рдореЗрдВ рдЯрд╛рдЗрдк рдХрд░реЗрдВ, рдлрд┐рд░ рдПрдХ рд╕реНрдХреНрд░реАрдирд╢реЙрдЯ рд▓реЗрдВ) рдФрд░ рдЙрдиреНрд╣реЗрдВ рд╕рдм рдХреБрдЫ рдлрд┐рд░ рд╕реЗ рд╕реНрдерд╛рдкрд┐рдд рдХрд░рдиреЗ рдХреЗ рд▓рд┐рдП рд╕рдВрджрд░реНрднрд┐рдд рдХрд┐рдпрд╛ред

рдЪрд┐рдпрд░реНрд╕

рдХреЛрдИ рдореЗрд░реА рдорджрдж рдХрд░ рд╕рдХрддрд╛ рд╣реИ!
рдореИрдВ рдкрд╣рд▓реЗ рд╕реЗ рд╣реА рдЕрдЬрдЧрд░ 3.7.1 рд╕реНрдерд╛рдкрд┐рдд рдХрд░ рдЪреБрдХрд╛ рд╣реВрдВ, рдЯреЗрдВрд╕рд░-рдлреНрд▓реЛ рд╕реНрдерд╛рдкрд┐рдд рдХрд░рдирд╛ рдЪрд╛рд╣рддрд╛ рд╣реВрдВ, рд▓реЗрдХрд┐рди рдореИрдВ рдирд╣реАрдВ рдХрд░ рд╕рдХрддрд╛
рдХреЛрдИ рднреА рдЬреЛ рдореБрдЭреЗ рдмрддрд╛ рд╕рдХрддрд╛ рд╣реИ рдХрд┐ рдЗрд╕ рд╕рдорд╕реНрдпрд╛ рдХреЛ рдХреИрд╕реЗ рдареАрдХ рдХрд┐рдпрд╛ рдЬрд╛рдП

рд╣рд╛рдп рдирд╛рд╕рд░, рджреБрд░реНрднрд╛рдЧреНрдп рд╕реЗ Tensorflow рдЕрднреА рддрдХ Python v3.7 рдХреЗ рд╕рд╛рде рдХрд╛рдо рдирд╣реАрдВ рдХрд░рддрд╛ рд╣реИред рдЖрдкрдХреЛ рдЕрдирдЗрдВрд╕реНрдЯреЙрд▓ рдХрд░рдирд╛ рд╣реЛрдЧрд╛, Python рдХреА рд╡реЗрдмрд╕рд╛рдЗрдЯ рдкрд░ рдЬрд╛рдирд╛ рд╣реЛрдЧрд╛ рдФрд░ Python v3.6 рдЗрдВрд╕реНрдЯреЙрд▓ рдХрд░рдирд╛ рд╣реЛрдЧрд╛ред рдЕрдЪреНрдЫреА рдмрд╛рдд рдпрд╣ рд╣реИ рдХрд┐ рдХрдИ рдЕрдВрддрд░ рдирд╣реАрдВ рд╣реИрдВ :) рдмрд╕ рдЕрдкрдиреЗ рд╕рднреА рдкреИрдХреЗрдЬреЛрдВ рдХреЛ рдлрд┐рд░ рд╕реЗ рд╕реНрдерд╛рдкрд┐рдд рдХрд░рдирд╛ рд╕реБрдирд┐рд╢реНрдЪрд┐рдд рдХрд░реЗрдВред рдореИрдВрдиреЗ рд╡реНрдпрдХреНрддрд┐рдЧрдд рд░реВрдк рд╕реЗ рдЕрдкрдиреЗ рд╕рднреА рдкреИрдХреЗрдЬреЛрдВ рдХрд╛ рдПрдХ рд╕реНрдХреНрд░реАрдирд╢реЙрдЯ рд▓рд┐рдпрд╛ (рдпрд╛рдиреА рдХрдорд╛рдВрдб рдкреНрд░реЙрдореНрдкреНрдЯ рдореЗрдВ рдЬрд╛рдПрдВ, рдФрд░ рдкрд╛рдЗрдк рд╕реВрдЪреА рдореЗрдВ рдЯрд╛рдЗрдк рдХрд░реЗрдВ, рдлрд┐рд░ рдПрдХ рд╕реНрдХреНрд░реАрдирд╢реЙрдЯ рд▓реЗрдВ) рдФрд░ рдЙрдиреНрд╣реЗрдВ рд╕рдм рдХреБрдЫ рдлрд┐рд░ рд╕реЗ рд╕реНрдерд╛рдкрд┐рдд рдХрд░рдиреЗ рдХреЗ рд▓рд┐рдП рд╕рдВрджрд░реНрднрд┐рдд рдХрд┐рдпрд╛ред

рдЪрд┐рдпрд░реНрд╕

рдзрдиреНрдпрд╡рд╛рдж

рдЕрдЬрдЧрд░ 3.7.2 рдХреЗ рд▓рд┐рдП рдЯреЗрдВрд╕рд░рдлрд╝реНрд▓реЛ рдХрд╛ рд╕рдорд░реНрдерди рдХрдм рдХрд┐рдпрд╛ рдЬрд╛рдПрдЧрд╛?

рдХреЛрд╢рд┐рд╢ рдХрд░реЛ

python3 -m pip install --upgrad https://storage.googleapis.com/tensorflow/mac/cpu/tensorflow-0.12.0-py3-none-any.whl

$ pip --version
pip 18.1 

$ python --version
Python 3.7.0

$ pip install tensorflow
Collecting tensorflow
  Could not find a version that satisfies the requirement tensorflow (from versions: )
No matching distribution found for tensorflow

$ pip search tensorflow
tensorflow (1.12.0)       - TensorFlow is an open source machine learning framework for everyone.

$ pip install --upgrade https://storage.googleapis.com/tensorflow/mac/cpu/tensorflow-1.12.0-py3-none-any.whl

$ python
Python 3.7.0 (default, Oct 18 2018, 15:08:12)
[Clang 10.0.0 (clang-1000.10.44.2)] on darwin
Type "help", "copyright", "credits" or "license" for more information.
>>> import tensorflow
/lib/python3.7/importlib/_bootstrap.py:219: RuntimeWarning: compiletime version 3.6 of module 'tensorflow.python.framework.fast_tensor_util' does not match runtime version 3.7
  return f(*args, **kwds)
>>> tensorflow.__version__
'1.12.0'

рдЕрдЪреНрдЫрд╛ рдпрд╣ рдореЗрд░реЗ рд▓рд┐рдП рдХрд╛рдо рдирд╣реАрдВ рдХрд░рддрд╛ рд╣реИред рдкрд╛рдЗрдк рдХреЗ рд╕рд╛рде рдбрд╛рдЙрдирд▓реЛрдб рд╕рдорд╛рдкреНрдд рд╣реЛрдиреЗ рдХреЗ рдмрд╛рдж рдпрд╣ рдХрд╣рддрд╛ рд╣реИ рдХрд┐ рдЯреЗрдВрд╕рд░рдлреНрд▓реЛ рдирд╛рдо рдХрд╛ рдХреЛрдИ рдореЙрдбреНрдпреВрд▓ рдирд╣реАрдВ рд╣реИ

рдареАрдХ рд╣реИ рджреЛрд╕реНрддреЛрдВ... рджреЛ рдШрдВрдЯреЗ рдмрд┐рддрд╛рдПред рдЕрднреА рдХреЗ рд▓рд┐рдП рддреНрд╡рд░рд┐рдд рд╕рдорд╛рдзрд╛рди: рдпрд╣ рдЙрддрдирд╛ рд╣реА рд╕рд░рд▓ рд╣реИ... рдЕрднреА рдХреЗ рд▓рд┐рдП python3.6.5_1 рдХрд╛ рдЙрдкрдпреЛрдЧ рдХрд░реЗрдВред рдЕрдкрдиреЗ рдЯреЗрдВрд╕рд░рдлрд╝реНрд▓реЛ-рдЖрдзрд╛рд░рд┐рдд рдПрдкреНрд▓рд┐рдХреЗрд╢рди рдХреЛ рдкреБрд░рд╛рдиреЗ рдкрд╛рдпрдерди рдирд┐рд╖реНрдкрд╛рджрди рдпреЛрдЧреНрдп рдХреЗ рдкреВрд░реНрдг рдкрде рдХреЗ рд╕рд╛рде рд▓реЙрдиреНрдЪ рдХрд░реЗрдВред

тЭп /usr/local/Cellar/python3/3.6.5_1/bin/python3
Python 3.6.5 (default, Jun 17 2018, 12:13:06)
[GCC 4.2.1 Compatible Apple LLVM 9.1.0 (clang-902.0.39.2)] on darwin
Type "help", "copyright", "credits" or "license" for more information.
>>> import tensorflow
>>> tensorflow.__version__
'1.12.0'
>>>

рдмрд╕ рд╕реБрдирд┐рд╢реНрдЪрд┐рдд рдХрд░реЗрдВ рдХрд┐ рдЖрдк рд╣реЛрдордмреНрд░реЗ рдпрд╛ рдХрд┐рд╕реА рдЕрдиреНрдп рддрд░реАрдХреЗ рд╕реЗ рдЗрд╕ рдЕрдЬрдЧрд░ рд╕рдВрд╕реНрдХрд░рдг рдХреЛ рд╕реНрдерд╛рдкрд┐рдд рдХрд░рдиреЗ рдХрд╛ рдкреНрд░рдмрдВрдзрди рдХрд░рддреЗ рд╣реИрдВред рдЕрдм рдмрд╛рдХреА рд╕рдм рдХреБрдЫ рдирд┐рдпрдорд┐рдд рдЕрдк-рдЯреВ-рдбреЗрдЯ рдЕрдЬрдЧрд░ 3.7 рдХрд╛ рдЙрдкрдпреЛрдЧ рдХрд░ рд╕рдХрддрд╛ рд╣реИ, рдЬрдмрдХрд┐ рд╣рдо рдЗрд╕реЗ рдареАрдХ рдХрд░рдиреЗ рдХреЗ рд▓рд┐рдП TensorFlow devs рдХреА рдкреНрд░рддреАрдХреНрд╖рд╛ рдХрд░рддреЗ рд╣реИрдВред

@PiotrWegrzyn ,

рджреЗрдЦреЗрдВ рдХрд┐ рдХреНрдпрд╛ рдирд┐рдореНрдирд▓рд┐рдЦрд┐рдд рдЖрдкрдХреЗ рд▓рд┐рдП рдХрд╛рдо рдХрд░рддрд╛ рд╣реИ:
pip3 install --upgrade https://storage.googleapis.com/tensorflow/mac/cpu/tensorflow-1.12.0-py3-none-any.whl

@ рд╢рд╛рд╣рд░рд┐рд▓96

рдЖрдкрдХреЗ рд╕рдорд╛рдзрд╛рди рдиреЗ рдореЗрд░реЗ рд▓рд┐рдП рдЖрд░реНрдХ рд▓рд┐рдирдХреНрд╕ рдкрд░ рдХрд╛рдо рдХрд┐рдпрд╛, рдзрдиреНрдпрд╡рд╛рдж!

рднрд▓реЗ рд╣реА рдЖрдк рд╕рдлрд▓рддрд╛рдкреВрд░реНрд╡рдХ tensorflow1.12 рдХреЛ рдЕрдЬрдЧрд░ 3.7 рдореЗрдВ рд╕реНрдерд╛рдкрд┐рдд рдХрд░реЗрдВ, рдпрд╣ рдХреБрдЫ рдЕрдирдкреЗрдХреНрд╖рд┐рдд рддреНрд░реБрдЯрд┐ рдкреНрд░рд╛рдкреНрдд рдХрд░реЗрдЧрд╛ рдЬреИрд╕реЗ:
'tf.constant(5)' рдкреНрд░рд╛рдкреНрдд рдХрд░реЗрдВ 'UnboundLocalError: рд╕реНрдерд╛рдиреАрдп рдЪрд░ 'рд╕реНрд╡' рдЕрд╕рд╛рдЗрдирдореЗрдВрдЯ рд╕реЗ рдкрд╣рд▓реЗ рд╕рдВрджрд░реНрднрд┐рдд'

рд╣рдо PIP рдкрд░ Python 3.7 рдХреЗ рд▓рд┐рдП Tensorflow рдХреА рдЙрдореНрдореАрдж рдХрдм рдХрд░ рд╕рдХрддреЗ рд╣реИрдВ? рдЬрдм рдЖрдк рдЗрд╕реЗ рдХрд┐рд╕реА рдЕрдиреНрдп рдкреИрдХреЗрдЬ рдореЗрдВ рдирд┐рд░реНрднрд░рддрд╛ рдХреЗ рд░реВрдк рдореЗрдВ рд╢рд╛рдорд┐рд▓ рдХрд░рддреЗ рд╣реИрдВ рддреЛ рдЕрдиреНрдп рд╡рд┐рдХрд▓реНрдк рдХрд╛рдо рдирд╣реАрдВ рдХрд░рддреЗ рд╣реИрдВред

TensorFlow рдореЗрдВ рдкрд╛рдЗрдк рдкрд░ 3.7 рд╣реЛрдЧрд╛ рдЬрдм рдЗрд╕рдХреА рдирд┐рд░реНрднрд░рддрд╛ рдПрдХ рд╣реЛрдЧреАред рдпрд╣, рд╡рд┐рд╢реЗрд╖ рд░реВрдк рд╕реЗ, рдкреНрд░реЛрдЯреЛрдмрдл рдХрд╛ рдЕрд░реНрде рд╣реИред рдкреНрд░реЛрдЯреЛрдмрдл рдХреЗ рд▓рд┐рдП рдкреИрдЪ рд╣реИрдВ рдЬреЛ 3.7 рдкрд░ рдХрд╛рдо рдХрд░рдирд╛ рд╕рдВрднрд╡ рдмрдирд╛рддреЗ рд╣реИрдВ (рдирд╡рдВрдмрд░ рд╕реЗ, рдореЛрдЯреЗ рддреМрд░ рдкрд░), рд▓реЗрдХрд┐рди рдЕрднреА рддрдХ рдХреЛрдИ рд░рд┐рд▓реАрдЬ рдирд╣реАрдВ рд╣реБрдИ рд╣реИред

Python3.7 рдХрд╛ рд╕рдорд░реНрдерди рдХрд░рдирд╛ рдореБрд╢реНрдХрд┐рд▓ рд▓рдЧрддрд╛ рд╣реИред рдорд╣реАрдиреЛрдВ рд╣реЛ рдЧрдП рд╣реИрдВ рдФрд░ рдЕрднреА рднреА рдХреЛрдИ рдкреНрд░рдЧрддрд┐ рдирд╣реАрдВ рд╣реБрдИ рд╣реИ

+1

+1

+1

рдкрд╛рдпрдерди рд╕рдореВрд╣ рдПрдХ рдорд╛рдореВрд▓реА рд╕рдВрд╕реНрдХрд░рдг рдЯрдХреНрдХрд░ рдХреЗ рд╕рд╛рде рдмреНрд░реЗрдХрд┐рдВрдЧ рднрд╛рд╖рд╛ рдкрд░рд┐рд╡рд░реНрддрди рдЬрд╛рд░реА рдХрд░рдиреЗ рдкрд░ рдЬреЛрд░ рджреЗрддрд╛ рд╣реИред

рдореИрдВрдиреЗ рд╕реЛрдЪрд╛ рдерд╛ рдХрд┐ рдкрд╛рдЗрдерди рдЗрди рдмреНрд░реЗрдХрд┐рдВрдЧ рдкрд░рд┐рд╡рд░реНрддрдиреЛрдВ рдХреЗ рд╕рд╛рде рдХрд┐рдпрд╛ рдЧрдпрд╛ рдерд╛ред рд▓реЗрдХрд┐рди Google async рдкрд░ рдПрдХ рдЪрд░ рдирд╛рдо рдХреЗ рд░реВрдк рдореЗрдВ рдЖрддреЗ рд╣реИрдВ?

рдореИрдВ рд╕рд┐рд░реНрдл рдЗрд╕ рдмрд╛рдд рдкрд░ рдкреНрд░рдХрд╛рд╢ рдбрд╛рд▓рдирд╛ рдЪрд╛рд╣рддрд╛ рдерд╛ рдХрд┐ +1 рдпрд╛ рд╢рд┐рдерд┐рд▓ рд░реВрдк рд╕реЗ рдЬреБрдбрд╝реА рдЯрд┐рдкреНрдкрдгрд┐рдпреЛрдВ рдХреЛ рдкреЛрд╕реНрдЯ рдХрд░рдиреЗ рд╕реЗ рдЗрд╕ рдереНрд░реЗрдб рдХреЛ рджреЗрдЦрдиреЗ рд╡рд╛рд▓реЗ рд▓рдЧрднрдЧ 65 рд▓реЛрдЧреЛрдВ рдХреЗ рдореЗрд▓рдмреЙрдХреНрд╕ рд╕реНрдкреИрдо рд╣реЛ рдЬрд╛рдПрдВрдЧреЗ (рдореИрдВ рдХрдИ рдЖрд╢рд╛рдУрдВ рдХреЗ рд╕рд╛рде рд╢рд░реНрдд рд▓рдЧрд╛рддрд╛ рд╣реВрдВ рдХрд┐ рдЕрдЧрд▓реА рдЕрдзрд┐рд╕реВрдЪрдирд╛ рдпрд╣ рдХрд╣реЗрдЧреА рдХрд┐ рд╣рдореЗрдВ рдкрд╛рдпрдерди 3.7 рдХреЗ рд▓рд┐рдП рдПрдХ рд░рд┐рд▓реАрдЬ рдорд┐рд▓реА рд╣реИ)ред

рдХреЛрдИ рдЯрд┐рдкреНрдкрдгреА рд▓рд┐рдЦреЗ рдмрд┐рдирд╛ рд╕реВрдЪрдирд╛рдУрдВ рдХреА рд╕рджрд╕реНрдпрддрд╛ рд▓реЗрдиреЗ рдХреЗ рд▓рд┐рдП рдкреГрд╖реНрда рдХреЗ рд╢реАрд░реНрд╖-рджрд╛рдИрдВ рдУрд░ "рд╕рджрд╕реНрдпрддрд╛ рд▓реЗрдВ" рдмрдЯрди рдкрд░ рдХреНрд▓рд┐рдХ рдХрд░ рд╕рдХрддреЗ рд╣реИрдВред рдореБрдЭреЗ рд▓рдЧрддрд╛ рд╣реИ рдХрд┐ рд╕рдмреНрд╕реНрдХреНрд░рд╛рдЗрдмрд░реЛрдВ рдХреА рд╕рдВрдЦреНрдпрд╛ рдЗрд╕ рд╕рдорд╕реНрдпрд╛ рдореЗрдВ рдЗрдВрдЯрд░рд╕реНрдЯ рдХрд╛ рдЙрддрдирд╛ рд╣реА рдЕрдЪреНрдЫрд╛ рд╕рдВрдХреЗрдд рд╣реЛрдЧрд╛ рдЬрд┐рддрдирд╛ рдХрд┐ "+1" рд▓рд┐рдЦрдирд╛ рд▓реЗрдХрд┐рди рдмрд╛рдХреА рд╕рднреА рдХреЛ рд╕реВрдЪрд┐рдд рдХрд┐рдП рдмрд┐рдирд╛ред рд╢реБрдХреНрд░рд┐рдпрд╛ :)

рдЗрд╕рдХреЗ рдЕрд▓рд╛рд╡рд╛, рдпрд╣рд╛рдВ рдПрдХ рдФрд░ рд╣рд╛рдЗрд▓рд╛рдЗрдЯ рд╣реИ: async рд╡рд╛рд╕реНрддрд╡ рдореЗрдВ рд╕рд┐рддрдВрдмрд░ 2015 рдореЗрдВ рдЬрд╛рд░реА рдкрд╛рдпрдерди 3.5 рдореЗрдВ рдкреЗрд╢ рдХрд┐рдпрд╛ рдЧрдпрд╛ рдерд╛ - рдЬреЛ рдХрд┐ рддреАрди рд╕рд╛рд▓ рд╕реЗ рдЕрдзрд┐рдХ рд╕рдордп рдкрд╣рд▓реЗ рдерд╛ред рдФрд░ рдЙрдиреНрд╣реЛрдВрдиреЗ рдЪреЗрддрд╛рд╡рдиреА рджреА рдереА:

async рдФрд░ рдкреНрд░рддреАрдХреНрд╖рд╛ рдХреЛ рдЪрд░, рд╡рд░реНрдЧ, рдлрд╝рдВрдХреНрд╢рди рдпрд╛ рдореЙрдбреНрдпреВрд▓ рдирд╛рдореЛрдВ рдХреЗ рд░реВрдк рдореЗрдВ рдЙрдкрдпреЛрдЧ рдХрд░рдиреЗ рдХреА рдЕрдиреБрд╢рдВрд╕рд╛ рдирд╣реАрдВ рдХреА рдЬрд╛рддреА рд╣реИред PEP 492 рджреНрд╡рд╛рд░рд╛ Python 3.5 рдореЗрдВ рдкреЗрд╢ рдХрд┐рдпрд╛ рдЧрдпрд╛, рд╡реЗ Python 3.7 рдореЗрдВ рдЙрдЪрд┐рдд рдХреАрд╡рд░реНрдб рдмрди рдЬрд╛рдПрдВрдЧреЗред

рдЗрд╕рд▓рд┐рдП рдореЗрд░рд╛ рдорд╛рдирдирд╛ тАЛтАЛрд╣реИ рдХрд┐ рд╣рдо рдЗрд╕рдХреЗ рд▓рд┐рдП #25429 . рдкрд░ рдПрдХ рд╡рд╛рд╕реНрддрд╡рд┐рдХ рдЦреБрд▓реЗ рдЯрд┐рдХрдЯ рдХрд╛ рдЕрдиреБрд╕рд░рдг рдХрд░ рд╕рдХрддреЗ рд╣реИрдВ

рдкреНрд░рдгрд╛рд▓реА рдХреА рдЬрд╛рдирдХрд╛рд░реА

  • рдХреНрдпрд╛ рдореИрдВрдиреЗ рдХрд╕реНрдЯрдо рдХреЛрдб рд▓рд┐рдЦрд╛ рд╣реИ (рдЬреИрд╕рд╛ рдХрд┐ TensorFlow рдореЗрдВ рдкреНрд░рджрд╛рди рдХреА рдЧрдИ рд╕реНрдЯреЙрдХ рдЙрджрд╛рд╣рд░рдг рд╕реНрдХреНрд░рд┐рдкреНрдЯ рдХрд╛ рдЙрдкрдпреЛрдЧ рдХрд░рдиреЗ рдХреЗ рд╡рд┐рдкрд░реАрдд) : N/A
  • OS рдкреНрд▓реЗрдЯрдлреЙрд░реНрдо рдФрд░ рд╡рд┐рддрд░рдг (рдЬреИрд╕реЗ, Linux Ubuntu 16.04) : macOS 10.13
  • TensorFlow (рд╕реНрд░реЛрдд рдпрд╛ рдмрд╛рдЗрдирд░реА) рд╕реЗ рд╕реНрдерд╛рдкрд┐рдд : рдмрд╛рдЗрдирд░реА
  • TensorFlow рд╕рдВрд╕реНрдХрд░рдг (рдиреАрдЪреЗ рдХрдорд╛рдВрдб рдХрд╛ рдЙрдкрдпреЛрдЧ рдХрд░реЗрдВ) : 1.8
  • рдкрд╛рдпрдерди рд╕рдВрд╕реНрдХрд░рдг : 3.7
  • рдмреЗрдЬрд╝рд▓ рд╕рдВрд╕реНрдХрд░рдг (рдпрджрд┐ рд╕реНрд░реЛрдд рд╕реЗ рд╕рдВрдХрд▓рд┐рдд рдХрд┐рдпрд╛ рдЬрд╛ рд░рд╣рд╛ рд╣реИ) : рдПрди/рдП
  • рдЬреАрд╕реАрд╕реА/рдХрдВрдкрд╛рдЗрд▓рд░ рд╕рдВрд╕реНрдХрд░рдг (рдпрджрд┐ рд╕реНрд░реЛрдд рд╕реЗ рд╕рдВрдХрд▓рд┐рдд рдХрд┐рдпрд╛ рдЬрд╛ рд░рд╣рд╛ рд╣реИ) : рд▓рд╛рдЧреВ рдирд╣реАрдВ
  • CUDA/cuDNN рд╕рдВрд╕реНрдХрд░рдг : рдПрди/рдП
  • GPU рдореЙрдбрд▓ рдФрд░ рдореЗрдореЛрд░реА : N/A
  • рдкреБрди: рдкреЗрд╢ рдХрд░рдиреЗ рдХреЗ рд▓рд┐рдП рд╕рдЯреАрдХ рдЖрджреЗрд╢ : pip install tensorflow

рд╕рдорд╕реНрдпрд╛ рд╡рд┐рд╕реНрддрд╛рд░ рд╕реЗ рд╕рдордЭрд╛рдЗрдпреЗ

рдкрд╛рдЗрдк рдХреЗ рд╕рд╛рде Python3.7 рдкрд░ TensorFlow рд╕реНрдерд╛рдкрд┐рдд рдХрд░рдирд╛ рд╡рд┐рдлрд▓ рд░рд╣рд╛ред рдХреГрдкрдпрд╛ рдиреАрдЪреЗ рд╡рд┐рдлрд▓рддрд╛ рд▓реЙрдЧ рджреЗрдЦреЗрдВред

рд╕реНрд░реЛрдд рдХреЛрдб / рд▓реЙрдЧ

рдРрд╕рд╛ рд╕рдВрд╕реНрдХрд░рдг рдирд╣реАрдВ рдорд┐рд▓рд╛ рдЬреЛ рдЯреЗрдВрд╕рд░рдлрд╝реНрд▓реЛ рдХреА рдЖрд╡рд╢реНрдпрдХрддрд╛ рдХреЛ рдкреВрд░рд╛ рдХрд░рддрд╛ рд╣реЛ (рд╕рдВрд╕реНрдХрд░рдгреЛрдВ рд╕реЗ:)
рдЯреЗрдВрд╕рд░рдлрд╝реНрд▓реЛ рдХреЗ рд▓рд┐рдП рдХреЛрдИ рдорд┐рд▓рд╛рди рд╡рд┐рддрд░рдг рдирд╣реАрдВ рдорд┐рд▓рд╛

рдпрд╣ рдореЗрд░реЗ рд▓рд┐рдП рдХрд╛рдо рдХрд┐рдпрд╛:

рдЕрдЬрдЧрд░-рдПрдо рдкрд╛рдЗрдк рд╕реНрдерд╛рдкрд┐рдд рдХрд░реЗрдВ https://storage.googleapis.com/tensorflow/mac/cpu/tensorflow-1.12.0-py3-none-any.whl

рд╣реЗ рджреЛрд╕реНрддреЛрдВ, рдореЗрд░рд╛ рдорд╛рдирдирд╛ тАЛтАЛрд╣реИ рдХрд┐ рдпрд╣ рдирд╡реАрдирддрдо рдЕрдЬрдЧрд░/рдкрд╛рдЗрдк рдкрд░ рддрдп рдХрд┐рдпрд╛ рдЧрдпрд╛ рд╣реИ

$ python -V
Python 3.7.2
$pip -V
pip 19.0.3 from /usr/lib/python3.7/site-packages/pip (python 3.7)

рдпрд╣рд╛рдБ рдЗрдВрд╕реНрдЯрд╛рд▓ рдХрд╛ рд▓реЙрдЧ рд╣реИ

$ pip install tensorflow --no-cache-dir
Collecting tensorflow
  Downloading https://files.pythonhosted.org/packages/74/1b/8b39fbe2fc8a7f6c9e19824b45b8a295526da466f75e7c53e00c51105664/tensorflow-1.13.0rc2-cp37-cp37m-manylinux1_x86_64.whl (92.7MB)
    100% |тЦИтЦИтЦИтЦИтЦИтЦИтЦИтЦИтЦИтЦИтЦИтЦИтЦИтЦИтЦИтЦИтЦИтЦИтЦИтЦИтЦИтЦИтЦИтЦИтЦИтЦИтЦИтЦИтЦИтЦИтЦИтЦИ| 92.7MB 9.8MB/s 
Collecting keras-applications>=1.0.6 (from tensorflow)
  Downloading https://files.pythonhosted.org/packages/90/85/64c82949765cfb246bbdaf5aca2d55f400f792655927a017710a78445def/Keras_Applications-1.0.7-py2.py3-none-any.whl (51kB)
    100% |тЦИтЦИтЦИтЦИтЦИтЦИтЦИтЦИтЦИтЦИтЦИтЦИтЦИтЦИтЦИтЦИтЦИтЦИтЦИтЦИтЦИтЦИтЦИтЦИтЦИтЦИтЦИтЦИтЦИтЦИтЦИтЦИ| 61kB 10.3MB/s 
Collecting tensorflow-estimator<1.14.0rc0,>=1.13.0rc0 (from tensorflow)
  Downloading https://files.pythonhosted.org/packages/bb/48/13f49fc3fa0fdf916aa1419013bb8f2ad09674c275b4046d5ee669a46873/tensorflow_estimator-1.13.0-py2.py3-none-any.whl (367kB)
    100% |тЦИтЦИтЦИтЦИтЦИтЦИтЦИтЦИтЦИтЦИтЦИтЦИтЦИтЦИтЦИтЦИтЦИтЦИтЦИтЦИтЦИтЦИтЦИтЦИтЦИтЦИтЦИтЦИтЦИтЦИтЦИтЦИ| 368kB 9.7MB/s 
Requirement already satisfied: protobuf>=3.6.1 in /home/j/.myvirtualenvs/pintpass/lib/python3.7/site-packages (from tensorflow) (3.6.1)
Requirement already satisfied: numpy>=1.13.3 in /home/j/.myvirtualenvs/pintpass/lib/python3.7/site-packages (from tensorflow) (1.15.4)
Collecting absl-py>=0.1.6 (from tensorflow)
  Downloading https://files.pythonhosted.org/packages/31/bc/ab68120d1d89ae23b694a55fe2aece2f91194313b71f9b05a80b32d3c24b/absl-py-0.7.0.tar.gz (96kB)
    100% |тЦИтЦИтЦИтЦИтЦИтЦИтЦИтЦИтЦИтЦИтЦИтЦИтЦИтЦИтЦИтЦИтЦИтЦИтЦИтЦИтЦИтЦИтЦИтЦИтЦИтЦИтЦИтЦИтЦИтЦИтЦИтЦИ| 102kB 14.3MB/s 
Collecting astor>=0.6.0 (from tensorflow)
  Downloading https://files.pythonhosted.org/packages/35/6b/11530768cac581a12952a2aad00e1526b89d242d0b9f59534ef6e6a1752f/astor-0.7.1-py2.py3-none-any.whl
Requirement already satisfied: six>=1.10.0 in /home/j/.myvirtualenvs/pintpass/lib/python3.7/site-packages (from tensorflow) (1.11.0)
Collecting gast>=0.2.0 (from tensorflow)
  Downloading https://files.pythonhosted.org/packages/4e/35/11749bf99b2d4e3cceb4d55ca22590b0d7c2c62b9de38ac4a4a7f4687421/gast-0.2.2.tar.gz
Requirement already satisfied: wheel>=0.26 in /home/j/.myvirtualenvs/pintpass/lib/python3.7/site-packages (from tensorflow) (0.32.3)
Collecting termcolor>=1.1.0 (from tensorflow)
  Downloading https://files.pythonhosted.org/packages/8a/48/a76be51647d0eb9f10e2a4511bf3ffb8cc1e6b14e9e4fab46173aa79f981/termcolor-1.1.0.tar.gz
Collecting tensorboard<1.13.0,>=1.12.0 (from tensorflow)
  Downloading https://files.pythonhosted.org/packages/07/53/8d32ce9471c18f8d99028b7cef2e5b39ea8765bd7ef250ca05b490880971/tensorboard-1.12.2-py3-none-any.whl (3.0MB)
    100% |тЦИтЦИтЦИтЦИтЦИтЦИтЦИтЦИтЦИтЦИтЦИтЦИтЦИтЦИтЦИтЦИтЦИтЦИтЦИтЦИтЦИтЦИтЦИтЦИтЦИтЦИтЦИтЦИтЦИтЦИтЦИтЦИ| 3.1MB 10.5MB/s 
Collecting keras-preprocessing>=1.0.5 (from tensorflow)
  Downloading https://files.pythonhosted.org/packages/c0/bf/0315ef6a9fd3fc2346e85b0ff1f5f83ca17073f2c31ac719ab2e4da0d4a3/Keras_Preprocessing-1.0.9-py2.py3-none-any.whl (59kB)
    100% |тЦИтЦИтЦИтЦИтЦИтЦИтЦИтЦИтЦИтЦИтЦИтЦИтЦИтЦИтЦИтЦИтЦИтЦИтЦИтЦИтЦИтЦИтЦИтЦИтЦИтЦИтЦИтЦИтЦИтЦИтЦИтЦИ| 61kB 18.7MB/s 
Collecting grpcio>=1.8.6 (from tensorflow)
  Downloading https://files.pythonhosted.org/packages/0a/47/d40daf7a46543e4d762ce0507d83ec49a883a6c0fed4d33a9e93652de8a8/grpcio-1.18.0-cp37-cp37m-manylinux1_x86_64.whl (10.6MB)
    100% |тЦИтЦИтЦИтЦИтЦИтЦИтЦИтЦИтЦИтЦИтЦИтЦИтЦИтЦИтЦИтЦИтЦИтЦИтЦИтЦИтЦИтЦИтЦИтЦИтЦИтЦИтЦИтЦИтЦИтЦИтЦИтЦИ| 10.6MB 10.4MB/s 
Collecting h5py (from keras-applications>=1.0.6->tensorflow)
  Downloading https://files.pythonhosted.org/packages/8e/fd/2ca5c4f4ed33ac4178f9c4d551e3946ab480866e3cd67a65a67a4bb35367/h5py-2.9.0-cp37-cp37m-manylinux1_x86_64.whl (2.8MB)
    100% |тЦИтЦИтЦИтЦИтЦИтЦИтЦИтЦИтЦИтЦИтЦИтЦИтЦИтЦИтЦИтЦИтЦИтЦИтЦИтЦИтЦИтЦИтЦИтЦИтЦИтЦИтЦИтЦИтЦИтЦИтЦИтЦИ| 2.8MB 12.5MB/s 
Collecting mock>=2.0.0 (from tensorflow-estimator<1.14.0rc0,>=1.13.0rc0->tensorflow)
  Downloading https://files.pythonhosted.org/packages/e6/35/f187bdf23be87092bd0f1200d43d23076cee4d0dec109f195173fd3ebc79/mock-2.0.0-py2.py3-none-any.whl (56kB)
    100% |тЦИтЦИтЦИтЦИтЦИтЦИтЦИтЦИтЦИтЦИтЦИтЦИтЦИтЦИтЦИтЦИтЦИтЦИтЦИтЦИтЦИтЦИтЦИтЦИтЦИтЦИтЦИтЦИтЦИтЦИтЦИтЦИ| 61kB 9.2MB/s 
Requirement already satisfied: setuptools in /home/j/.myvirtualenvs/pintpass/lib/python3.7/site-packages (from protobuf>=3.6.1->tensorflow) (40.6.3)
Collecting markdown>=2.6.8 (from tensorboard<1.13.0,>=1.12.0->tensorflow)
  Downloading https://files.pythonhosted.org/packages/7a/6b/5600647404ba15545ec37d2f7f58844d690baf2f81f3a60b862e48f29287/Markdown-3.0.1-py2.py3-none-any.whl (89kB)
    100% |тЦИтЦИтЦИтЦИтЦИтЦИтЦИтЦИтЦИтЦИтЦИтЦИтЦИтЦИтЦИтЦИтЦИтЦИтЦИтЦИтЦИтЦИтЦИтЦИтЦИтЦИтЦИтЦИтЦИтЦИтЦИтЦИ| 92kB 18.4MB/s 
Collecting werkzeug>=0.11.10 (from tensorboard<1.13.0,>=1.12.0->tensorflow)
  Downloading https://files.pythonhosted.org/packages/20/c4/12e3e56473e52375aa29c4764e70d1b8f3efa6682bef8d0aae04fe335243/Werkzeug-0.14.1-py2.py3-none-any.whl (322kB)
    100% |тЦИтЦИтЦИтЦИтЦИтЦИтЦИтЦИтЦИтЦИтЦИтЦИтЦИтЦИтЦИтЦИтЦИтЦИтЦИтЦИтЦИтЦИтЦИтЦИтЦИтЦИтЦИтЦИтЦИтЦИтЦИтЦИ| 327kB 15.6MB/s 
Collecting pbr>=0.11 (from mock>=2.0.0->tensorflow-estimator<1.14.0rc0,>=1.13.0rc0->tensorflow)
  Downloading https://files.pythonhosted.org/packages/8c/7f/fed53b379500fd889707d1f6e61c2a35e12f2de87396894aff89b017d1d6/pbr-5.1.2-py2.py3-none-any.whl (107kB)
    100% |тЦИтЦИтЦИтЦИтЦИтЦИтЦИтЦИтЦИтЦИтЦИтЦИтЦИтЦИтЦИтЦИтЦИтЦИтЦИтЦИтЦИтЦИтЦИтЦИтЦИтЦИтЦИтЦИтЦИтЦИтЦИтЦИ| 112kB 3.4MB/s 
Installing collected packages: h5py, keras-applications, absl-py, pbr, mock, tensorflow-estimator, astor, gast, termcolor, markdown, werkzeug, grpcio, tensorboard, keras-preprocessing, tensorflow
  Running setup.py install for absl-py ... done
  Running setup.py install for gast ... done
  Running setup.py install for termcolor ... done
Successfully installed absl-py-0.7.0 astor-0.7.1 gast-0.2.2 grpcio-1.18.0 h5py-2.9.0 keras-applications-1.0.7 keras-preprocessing-1.0.9 markdown-3.0.1 mock-2.0.0 pbr-5.1.2 tensorboard-1.12.2 tensorflow-1.13.0rc2 tensorflow-estimator-1.13.0 termcolor-1.1.0 werkzeug-0.14.1

рдФрд░ рдкрд░рд┐рдгрд╛рдо

$ python
Python 3.7.2 (default, Jan 10 2019, 23:51:51) 
[GCC 8.2.1 20181127] on linux
Type "help", "copyright", "credits" or "license" for more information.
>>> import tensorflow as tf
>>>

рдЕрд╕рдВрдмрдВрдзрд┐рдд..
@ campanelli-resonon рдЕрд░реЗ! рдЬрд╝рд░реВрд░ рдмрд╛рд╣рд░ рдардВрдб рд╣реИ :)

рдореБрдЭреЗ рдпрд╣ рддреНрд░реБрдЯрд┐ рддрдм рдорд┐рд▓реА рдЬрдм рдореИрдВрдиреЗ рдЙрдмрдВрдЯреВ рдФрд░ рдкрд╛рдпрдерди 3.5 рдкрд░ рдЯреЗрдВрд╕рд░рдлрд╝реНрд▓реЛ рд╕реНрдерд╛рдкрд┐рдд рдХрд░рдиреЗ рдХрд╛ рдкреНрд░рдпрд╛рд╕ рдХрд┐рдпрд╛, рдХреГрдкрдпрд╛ рдХреЛрдИ рдорджрдж рдХрд░реЗрдВред рдореИрдВрдиреЗ рдкрд┐рдЫрд▓реЗ рд╣рдлреНрддреЗ рдПрдХ рд╕рдорд╛рди рдкреНрд░рд╢реНрди рдкреЛрд╕реНрдЯ рдХрд┐рдпрд╛ рд╣реИред
image

рдореИрдВ рдУрд░реЗрдХрд▓ рд╡рд░реНрдЪреБрдЕрд▓ рдмреЙрдХреНрд╕ рдкрд░ рдЙрдмрдВрдЯреВ рдЪрд▓рд╛ рд░рд╣рд╛ рд╣реВрдВ- рд╡рд┐рдВрдбреЛрдЬ 10 рдкрд░ рдЪрд▓ рд░рд╣рд╛ рд╣реИ, рдФрд░ рдкрд╛рдпрдерди 3.5, рдореБрдЭреЗ рдКрдкрд░ рднреА рд╡рд╣реА рдЗрдВрд╕реНрдЯреЙрд▓ рдЯреЗрдВрд╕рд░рдлреНрд▓реЛ рддреНрд░реБрдЯрд┐ рд╕рдВрджреЗрд╢ рдорд┐рд▓рд╛ рд╣реИ, рдЬрдм рдореИрдВрдиреЗ рдпреВрдЖрд░рдПрд▓ рдкрде рдХреЛ (linjx рдФрд░ рдореИрдХ) рдореЗрдВ рдмрджрд▓ рджрд┐рдпрд╛ рдерд╛ред

рдорд╛рдлреА: рдЯрд╛рдЗрдкреЛ рдХреЗ рд▓рд┐рдП, рдореЗрд░рд╛ рдорддрд▓рдм рдерд╛ (рд▓рд┐рдирдХреНрд╕, рдФрд░ рдореИрдХ)ред

рдХрд┐рд╕реА рднреА рдЕрджреНрдпрддрди?
рд╕реНрдерд╛рдкрд┐рдд рдЕрдЬрдЧрд░ рдХреЗ рд╕рд╛рде рд╡рд┐рдВрдбреЛ рд╕рд┐рд╕реНрдЯрдо рдкрд░ 3.7.2
рдореИрдВ рдХрдорд╛рдВрдб рдЯрд╛рдЗрдк рдХрд░рддрд╛ рд╣реВрдВ pip install Tensoflow (рдирд╣реАрдВ pip install tensorflow) рдФрд░ рдпрд╣ рдХрд╛рдо рдХрд░ рдЧрдпрд╛ред

рдпрд╣ рддрдп рд╣реИ, рд╣рдо pypi рдореЗрдВ 3.7 рдмрд╛рдпрдиреЗрд░рд┐рдЬрд╝ рдкреНрд░рдХрд╛рд╢рд┐рдд рдХрд░ рд░рд╣реЗ рд╣реИрдВред рдпрджрд┐ рдЖрдкрдХреЛ рд╕рдорд╕реНрдпрд╛рдПрдБ рджрд┐рдЦрд╛рдИ рджреЗ рд░рд╣реА рд╣реИрдВ, рддреЛ рдХреГрдкрдпрд╛ рдПрдХ рдирдпрд╛ рдЕрдВрдХ рдЦреЛрд▓реЗрдВред

рд╣рд╛рдп рдореЗрдЬрд┐рдорд╛рд░реБ

рд╡рд╣ рд╕реНрдерд╛рдкрдирд╛ рд╕рдлрд▓рддрд╛рдкреВрд░реНрд╡рдХ рдкреВрд░реНрдг рд╣реЛ рдЧрдИ, рд▓реЗрдХрд┐рди рдЯреЗрдВрд╕реЛрдлрд╝реНрд▓реЛ рд╕рдорд╛рди рд╣реИрдЯреЗрдВрд╕рд░рдлрд╝реНрд▓реЛ? **рдЖрдкрдХреЗ рд╕рд╣рдпреЛрдЧ рдХреЗ рд▓рд┐рдП рдмрд╣реБрдд рдзрдиреНрдпрд╡рд╛рдж!

рд╕рд▓реАрд╣

рд╢реБрдХреНрд░, 29 рдорд╛рд░реНрдЪ, 2019 рдХреЛ рдкреВрд░реНрд╡рд╛рд╣реНрди 11:32 рдмрдЬреЗ рдорд╛рд░реНрдЯрд┐рди рд╡рд┐рдХ рдиреЛрдЯрд┐рдлрд┐рдХреЗрд╢рди @github.com
рд▓рд┐рдЦрд╛ рдерд╛:

рдпрд╣ рддрдп рд╣реИ, рд╣рдо pypi рдореЗрдВ 3.7 рдмрд╛рдпрдиреЗрд░рд┐рдЬрд╝ рдкреНрд░рдХрд╛рд╢рд┐рдд рдХрд░ рд░рд╣реЗ рд╣реИрдВред рдЕрдЧрд░ рдЖрдк рджреЗрдЦ рд░рд╣реЗ рд╣реИрдВ
рд╕рдорд╕реНрдпрд╛рдПрдВ, рдХреГрдкрдпрд╛ рдПрдХ рдирдпрд╛ рдореБрджреНрджрд╛ рдЦреЛрд▓реЗрдВред

-
рдЖрдк рдЗрд╕реЗ рдкреНрд░рд╛рдкреНрдд рдХрд░ рд░рд╣реЗ рд╣реИрдВ рдХреНрдпреЛрдВрдХрд┐ рдЖрдкрдиреЗ рдЯрд┐рдкреНрдкрдгреА рдХреА рдереАред
рдЗрд╕ рдИрдореЗрд▓ рдХрд╛ рд╕реАрдзреЗ рдЙрддреНрддрд░ рджреЗрдВ, рдЗрд╕реЗ GitHub рдкрд░ рджреЗрдЦреЗрдВ
https://github.com/tensorflow/tensorflow/issues/20444#issuecomment-478061701 ,
рдпрд╛ рдереНрд░реЗрдб рдХреЛ рдореНрдпреВрдЯ рдХрд░реЗрдВ
https://github.com/notifications/unsubscribe-auth/Apb8t7oBHHeBnRivki9_h5XCks1D_fO0jks5vbkAHgaJpZM4U-EUx
.

рд╣рд╛рдп рдореЗрдЬрд┐рдорд╛рд░реБ

рд╡рд╣ рдЗрдВрд╕реНрдЯреЙрд▓ рд╕рдлрд▓рддрд╛рдкреВрд░реНрд╡рдХ рдкреВрд░реНрдг рд╣реЛ рдЧрдпрд╛, рд▓реЗрдХрд┐рди рдЯреЗрдВрд╕реЛрдлрд╝реНрд▓реЛ рдЯреЗрдВрд╕реЛрдлрд╝реНрд▓реЛ рдХреЗ рд╕рдорд╛рди рд╣реИ? рд╕рдорд╛рди рдХрд╛рд░реНрдпрдХреНрд╖рдорддрд╛ рд╣реИ, рдореИрдВрдиреЗ рдкрд╣рд▓реЗ рдЯреЗрдВрд╕реЛрдлрд╝реНрд▓реЛ рдХрд╛ рдЙрдкрдпреЛрдЧ рдирд╣реАрдВ рдХрд┐рдпрд╛ рд╣реИред
рдЖрдкрдХреЗ рд╕рд╣рдпреЛрдЧ рдХреЗ рд▓рд┐рдП рдмрд╣реБрдд рдзрдиреНрдпрд╡рд╛рдж!

рдпрд╣ рддрдп рд╣реИ, рд╣рдо pypi рдореЗрдВ 3.7 рдмрд╛рдпрдиреЗрд░рд┐рдЬрд╝ рдкреНрд░рдХрд╛рд╢рд┐рдд рдХрд░ рд░рд╣реЗ рд╣реИрдВред рдпрджрд┐ рдЖрдкрдХреЛ рд╕рдорд╕реНрдпрд╛рдПрдБ рджрд┐рдЦрд╛рдИ рджреЗ рд░рд╣реА рд╣реИрдВ, рддреЛ рдХреГрдкрдпрд╛ рдПрдХ рдирдпрд╛ рдЕрдВрдХ рдЦреЛрд▓реЗрдВред

рддреЛ рдХреНрдпрд╛ рдЗрд╕рдХрд╛ рдорддрд▓рдм рдпрд╣ рд╣реИ рдХрд┐ рдореЗрд░реЗ рджреНрд╡рд╛рд░рд╛ рдКрдкрд░ рдкреЛрд╕реНрдЯ рдХреА рдЧрдИ рдЯреЗрдВрд╕рд░рдлрд╝реНрд▓реЛ рдЗрдВрд╕реНрдЯреЙрд▓ рддреНрд░реБрдЯрд┐ рдХреЛ рд╣рд▓ рдХрд░рдиреЗ рдХреЗ рд▓рд┐рдП рдореБрдЭреЗ рдЕрдкрдиреЗ рд╡рд░реНрддрдорд╛рди рдкрд╛рдпрдерди рд╕рдВрд╕реНрдХрд░рдг рдХреЛ 3.7 рдореЗрдВ рдмреИрдЪ/рдЕрдкрдЧреНрд░реЗрдб рдХрд░рдиреЗ рдХреА рдЖрд╡рд╢реНрдпрдХрддрд╛ рд╣реИред рдЙрдмрдВрдЯреВ 16.04 рдкрд░ рдореЗрд░рд╛ рд╡рд░реНрддрдорд╛рди рдкрд╛рдпрдерди рд╕рдВрд╕реНрдХрд░рдг "рдкрд╛рдпрдерди 3.5" рд╣реИред рдЬрд╛рдирдХрд╛рд░реА рдХреЗ рд▓рд┐рдП рдзрдиреНрдпрд╡рд╛рдж!

рдХреГрдкрдпрд╛ рдЕрдкрдиреА рд╕рдорд╕реНрдпрд╛ рдХреЗ рд▓рд┐рдП рдЙрдкрдпреБрдХреНрдд рд╕рдорд╕реНрдпрд╛ рдХрд╛ рдкрддрд╛ рд▓рдЧрд╛рдПрдВред рдпрд╣ рдорд╛рдорд▓рд╛ рдХрд░реАрдм 3.7 рдХрд╛ рд╣реИред

рдирдорд╕реНрддреЗ
рдЗрд╕рдиреЗ рдореЗрд░реЗ рд▓рд┐рдП рдкрд╛рдпрдерди 3.7.2 рдкрд░ рдХрд╛рдо рдХрд┐рдпрд╛:
sudo pip3 рд╕реНрдерд╛рдкрд┐рдд рдХрд░реЗрдВ
https://storage.googleapis.com/tensorflow/mac/cpu/tensorflow-1.8.0-py3-none-any.whl
https://storage.googleapis.com/tensorflow/mac/cpu/tensorflow-1.12.0-py3-none-any.whl

рд╕рд╛рджрд░,
рд╕рд▓реАрд╣

рд╢реБрдХреНрд░, 29 рдорд╛рд░реНрдЪ, 2019 рдХреЛ рдкреВрд░реНрд╡рд╛рд╣реНрди 11:22 рдмрдЬреЗ mejimaru [email protected] рдиреЗ рд▓рд┐рдЦрд╛:

рдХрд┐рд╕реА рднреА рдЕрджреНрдпрддрди?
рд╕реНрдерд╛рдкрд┐рдд рдЕрдЬрдЧрд░ рдХреЗ рд╕рд╛рде рд╡рд┐рдВрдбреЛ рд╕рд┐рд╕реНрдЯрдо рдкрд░ 3.7.2
рдореИрдВ рдХрдорд╛рдВрдб рдЯрд╛рдЗрдк рдХрд░рддрд╛ рд╣реВрдВ pip install Tensoflow (рдирд╣реАрдВ pip install tensorflow) рдФрд░
рдЗрд╕рдиреЗ рдХрд╛рдо рдХрд┐рдпрд╛ред

рдкрд╛рдЗрдк рдЯреЗрдиреНрд╕реЛрдлреНрд▓реЛ рд╕реНрдерд╛рдкрд┐рдд рдХрд░реЗрдВ

рдХрд╛рдо рдХрд┐рдпрд╛ред

-
рдЖрдк рдЗрд╕реЗ рдкреНрд░рд╛рдкреНрдд рдХрд░ рд░рд╣реЗ рд╣реИрдВ рдХреНрдпреЛрдВрдХрд┐ рдЖрдкрдиреЗ рдЯрд┐рдкреНрдкрдгреА рдХреА рдереАред
рдЗрд╕ рдИрдореЗрд▓ рдХрд╛ рд╕реАрдзреЗ рдЙрддреНрддрд░ рджреЗрдВ, рдЗрд╕реЗ GitHub рдкрд░ рджреЗрдЦреЗрдВ
https://github.com/tensorflow/tensorflow/issues/20444#issuecomment-478058562 ,
рдпрд╛ рдереНрд░реЗрдб рдХреЛ рдореНрдпреВрдЯ рдХрд░реЗрдВ
https://github.com/notifications/unsubscribe-auth/Apb8t-hlKUU7z8SO0STC3kXCis9BXs7gks5vbj3MgaJpZM4U-EUx
.

рдореИрдВ Python3.7.2 рдкрд░ рддреНрд░реБрдЯрд┐ рдХреЗ рдмрд┐рдирд╛ tensorflow рд╕реНрдерд╛рдкрд┐рдд рдХрд░рдиреЗ рдореЗрдВ рд╕рдХреНрд╖рдо рдерд╛, рд▓реЗрдХрд┐рди рдЬрдм рдореИрдВрдиреЗ Python 3,7.2 рдХреЛ рд▓реЙрдиреНрдЪ рдХрд┐рдпрд╛ рдФрд░ tensorflow рдХреЛ рдЖрдпрд╛рдд рдХрд░рдиреЗ рдХрд╛ рдкреНрд░рдпрд╛рд╕ рдХрд┐рдпрд╛, рддреЛ рдореБрдЭреЗ рдПрдХ рддреНрд░реБрдЯрд┐ рдорд┐рд▓реА "рдХреЛрдИ рдореЙрдбреНрдпреВрд▓ рдЬрд┐рд╕рдХрд╛ рдирд╛рдо tensorflow рдирд╣реАрдВ рд╣реИ"ред рдХреЛрдИ рд╣реЗрдк?

рдореИрдВ рд╕реНрд░реЛрдд рдХреЛрдб рд░реЛрдо рдЬреАрдердм рдХрд╛ рдЙрдкрдпреЛрдЧ рдХрд░рдХреЗ Tensorflow рдХреЛ рд╕реНрдерд╛рдкрд┐рдд рдХрд░рдиреЗ рдХрд╛ рдкреНрд░рдпрд╛рд╕ рдХрд░ рд░рд╣рд╛ рд╣реВрдВ, рдЬрдм рдореИрдВрдиреЗ рдЗрд╕реЗ рдЕрдкрдиреЗ Python3.7 рдкреНрд▓реЗрдЯрдлреЙрд░реНрдо рдкрд░ рд╕реНрдерд╛рдкрд┐рдд рдХрд░рдиреЗ рдХреЗ рд▓рд┐рдП рдЫреЛрдбрд╝ рджрд┐рдпрд╛, рддреЛ рдРрд╕рд╛ рдХрд░рдиреЗ рдХреЗ рд▓рд┐рдП рдореБрдЭреЗ рдЗрд╕ рд▓рд┐рдВрдХ рд╕реЗ "Bazel" рдбрд╛рдЙрдирд▓реЛрдб рдХрд░рдиреЗ рдХреА рдЖрд╡рд╢реНрдпрдХрддрд╛ рдирд╣реАрдВ рд╣реИ: https:// github.com/bazelbuild/bazel/pulls ред рд╕рдорд╕реНрдпрд╛, рдореБрдЭреЗ (wget, рдпрд╛ git clone,...etc) рдХрд╛ рдЙрдкрдпреЛрдЧ рдХрд░рдХреЗ "Bazel" рдХреЛ рдбрд╛рдЙрдирд▓реЛрдб рдФрд░ рдЗрдВрд╕реНрдЯреЙрд▓ рдХрд░рдиреЗ рдХреЗ рд▓рд┐рдП рд▓рд┐рдВрдХ рдорд┐рд▓ рд╕рдХрддрд╛ рд╣реИред
рдХреЛрдИ рдорджрдж рдХреГрдкрдпрд╛ред
рдЗрд╕рдХреЗ рдЕрд▓рд╛рд╡рд╛, рдХреНрдпрд╛ рдЖрдкрдХреЛ рд▓рдЧрддрд╛ рд╣реИ рдХрд┐ рдореБрдЭреЗ 3.7 рдкрд░ рдкреВрд░реА рддрд░рд╣ рд╕реЗ Tensorflow рд╕реНрдерд╛рдкрд┐рдд рдХрд░рдирд╛ рдЫреЛрдбрд╝ рджреЗрдирд╛ рдЪрд╛рд╣рд┐рдП, рдпрджрд┐ рдРрд╕рд╛ рд╣реИ рддреЛ TENSORFLOW рд╕реНрдерд╛рдкрд┐рдд рдХрд░рдиреЗ рдХреЗ рд▓рд┐рдП рд╕рдмрд╕реЗ рд╕реНрдерд┐рд░ рдкрд╛рдпрдерди рдкреНрд▓реЗрдЯрдлрд╝реЙрд░реНрдо рдХреНрдпрд╛ рд╣реИ?!!!!
рдпрд╣ рдмрд╣реБрдд рдирд┐рд░рд╛рд╢рд╛рдЬрдирдХ рд╣реИ!

@ SalihOsman1 рдРрд╕рд╛ рд▓рдЧрддрд╛ рд╣реИ рдХрд┐ рдЖрдкрдиреЗ рдЗрд╕реЗ рд╕рдлрд▓рддрд╛рдкреВрд░реНрд╡рдХ рд╕реНрдерд╛рдкрд┐рдд рдХрд░ рд▓рд┐рдпрд╛ рд╣реИред рдпрджрд┐ рдпрд╣ рдХрд╛рдо рдирд╣реАрдВ рдХрд░рддрд╛ рд╣реИ, рддреЛ рдЬрд╛рдВрдЪреЗрдВ рдХрд┐ рдХреНрдпрд╛ рдЖрдк рд╕рд╣реА рдкрд╛рдпрдерди рдмрд╛рдЗрдирд░реА рдХрд╛ рдЙрдкрдпреЛрдЧ рдХрд░ рд░рд╣реЗ рд╣реИрдВред рдпрджрд┐ рдЖрдкрдХреЛ рд╕рдВрджреЗрд╣ рд╣реИ рдХрд┐ рдЯреЗрдВрд╕рд░рдлрд╝реНрд▓реЛ рдореЗрдВ рдХреЛрдИ рдмрдЧ рд╣реИ, рддреЛ рдХреГрдкрдпрд╛ рд╕рдорд╕реНрдпрд╛ рдЯреЗрдореНрдкрд▓реЗрдЯ рдореЗрдВ рдЕрдиреБрд░реЛрдзрд┐рдд рдЬрд╛рдирдХрд╛рд░реА рдкреНрд░рджрд╛рди рдХрд░рддреЗ рд╣реБрдП рдПрдХ рд╕рдорд╕реНрдпрд╛ рдЦреЛрд▓реЗрдВред рдЖрдкрдХреЗ рд╡рд┐рд╡рд░рдг рдХреЛ рджреЗрдЦрддреЗ рд╣реБрдП, рдореЗрд░рд╛ рдорд╛рдирдирд╛ тАЛтАЛтАЛтАЛрд╣реИ рдХрд┐ рдпрд╣ рдХреБрдЫ рдЖрд╕рд╛рди рд╣реИ, рдЬреИрд╕реЗ рдЧрд▓рдд рдкрд╛рдпрдерди рд╕реНрдерд╛рдкрдирд╛ рдореЗрдВ рд╕реНрдерд╛рдкрд┐рдд рдХрд░рдирд╛ред

рдЖрдкрдХреА рд╕рдорд╕реНрдпрд╛ рдХреЗ рд▓рд┐рдП рдорджрдж рд▓реЗрдиреЗ рдХрд╛ рдпрд╣ рдЧрд▓рдд рдореБрджреНрджрд╛ рд╣реИред

@ SalihOsman1 рдЕрдкрдиреЗ рдЕрдЬрдЧрд░ - рд╕рдВрд╕реНрдХрд░рдг рдХреА рдЬрд╛рдБрдЪ рдХрд░реЗрдВ, рдЖрдкрдХреЛ 64-рдмрд┐рдЯ рдХреЗ рд▓рд┐рдП рдЕрдЬрдЧрд░ рдкреИрдХреЗрдЬ рд╕реНрдерд╛рдкрд┐рдд рдХрд░рдирд╛ рдЪрд╛рд╣рд┐рдПред

рдпрд╣ рдореЗрд░реЗ рд▓рд┐рдП рдХрд╛рдо рд╣реИ
рдмрд╕ рдЗрд╕реЗ рдиреАрдЪреЗ рдЙрдкрдпреЛрдЧ рдХрд░рдиреЗ рдХрд╛ рдкреНрд░рдпрд╛рд╕ рдХрд░реЗрдВ
рдкрд╛рдЗрдк рд╕реНрдерд╛рдкрд┐рдд рдХрд░реЗрдВ https://storage.googleapis.com/tensorflow/mac/cpu/tensorflow-1.8.0-py3-none-any.whl

рдпрд╣ рдореЗрд░реЗ рд▓рд┐рдП рдХрд╛рдо рдХрд┐рдпрд╛ред рдзрдиреНрдпрд╡рд╛рдж

@ SalihOsman1 рдЕрдкрдиреЗ рдЕрдЬрдЧрд░ - рд╕рдВрд╕реНрдХрд░рдг рдХреА рдЬрд╛рдБрдЪ рдХрд░реЗрдВ, рдЖрдкрдХреЛ 64-рдмрд┐рдЯ рдХреЗ рд▓рд┐рдП рдЕрдЬрдЧрд░ рдкреИрдХреЗрдЬ рд╕реНрдерд╛рдкрд┐рдд рдХрд░рдирд╛ рдЪрд╛рд╣рд┐рдПред

рд╣рд╛рдБ рд╕рдорд╕реНрдпрд╛ рд╣рд▓ рд╣реЛ рдЧрдИ ty рджреЛрд╕реНрдд рдореЗрд░рд╛ рдкрд╛рдпрдерди рд╕рдВрд╕реНрдХрд░рдг 32 рдмрд┐рдЯ рдерд╛

рдРрд╕рд╛ рд▓рдЧрддрд╛ рд╣реИ рдХрд┐ рдЬреНрдпрд╛рджрд╛рддрд░ рд▓реЛрдЧ рдкрд╣рдЪрд╛рдирддреЗ рд╣реИрдВ рдХрд┐ рд╕рдорд╕реНрдпрд╛ рдареАрдХ рд╣реЛ рдЧрдИ рд╣реИред рдореИрдВрдиреЗ рдЗрд╕реЗ рдЖрдЬ рд╣реА рд░реИрдкреНрд╕рдмрд┐рдпрди рдмрд╕реНрдЯрд░ рдХреЗ рдПрдХ рдирдП рдЗрдВрд╕реНрдЯрд╛рд▓ рдкрд░ рд▓реЛрдб рдХрд┐рдпрд╛ рд╣реИред рдпрд╣ рдЕрдЬрдЧрд░ 3.7.3 рдХреЗ рд╕рд╛рде рдЖрддрд╛ рд╣реИред рд▓реЗрдХрд┐рди рдкрд╛рдЗрдк 18.1 рд╣реИ рдФрд░ рдпрджрд┐ рдЖрдк рдЯреЗрдВрд╕рд░рдлрд╝реНрд▓реЛ рдХреЗ рд▓рд┐рдП рд╕рд╛рдЗрдЯ рджрд╕реНрддрд╛рд╡реЗрдЬрд╝реАрдХрд░рдг рджреЗрдЦрддреЗ рд╣реИрдВ рддреЛ рд╡реЗ рдЪрд╛рд╣рддреЗ рд╣реИрдВ рдХрд┐ рдкреАрдЖрдИрдкреА>19.0 рд╣реЛред рддреЛ рдореБрдЭреЗ рдХрд░рдирд╛ рдерд╛
python3 -рдПрдо рдкрд╛рдЗрдк рд╕реНрдерд╛рдкрд┐рдд рдХрд░реЗрдВ - рдкрд╛рдЗрдк рдХреЛ рдЕрдкрдЧреНрд░реЗрдб рдХрд░реЗрдВ

рдФрд░ рдлрд┐рд░ рдЬрд╛рдВрдЪреЗрдВ
pip3 --рд╕рдВрд╕реНрдХрд░рдг

рдпрд╣ рд╕реБрдирд┐рд╢реНрдЪрд┐рдд рдХрд░рдиреЗ рдХреЗ рд▓рд┐рдП рдХрд┐ рдпрд╣ 19.2 . рдкрд░ рдерд╛
рдлрд┐рд░ рдореИрдВ рднрд╛рдЧрд╛

sudo pip3 tensorflow рд╕реНрдерд╛рдкрд┐рдд рдХрд░реЗрдВ
(рдПрдХ рдкреБрд╕реНрддрдХрд╛рд▓рдп рдХреЗ рд▓рд┐рдП рд▓рд┐рдЦрдиреЗ рдХреА рдЕрдиреБрдорддрд┐ рдереА) рдФрд░ рдЗрд╕рдиреЗ рдареАрдХ рдХрд╛рдо рдХрд┐рдпрд╛

рдореИрдВ рдЕрдЬрдЧрд░ 3.7 рдХреЗ рд▓рд┐рдП рд╡рд┐рдВрдбреЛрдЬрд╝ рдкрд░ TensorFlow рдХреИрд╕реЗ рд╕реНрдерд╛рдкрд┐рдд рдХрд░реВрдВ? рдореИрдВ рдХрд┐рд╕ рдХрдорд╛рдВрдб рдХрд╛ рдЙрдкрдпреЛрдЧ рдХрд░рддрд╛ рд╣реВрдВ рдпрд╣ pip install tensorflow рдХрд╛ рдЙрдкрдпреЛрдЧ рдХрд░рдХреЗ рдХрд╛рдо рдирд╣реАрдВ рдХрд░ рд░рд╣рд╛ рд╣реИ

рдЕрдкрдиреЗ pip --version рдХреА рдЬрд╛рдБрдЪ рдХрд░реЗрдВ рдпрд╣ рдХрдо рд╕реЗ рдХрдо 19.0 рд╣реЛрдирд╛ рдЪрд╛рд╣рд┐рдП рдЕрдиреНрдпрдерд╛ tensorflow рд╕реНрдерд╛рдкрд┐рдд рдирд╣реАрдВ рд╣реЛрдЧрд╛ред рдЕрдЧрд░ рдпрд╣ рдЕрднреА рднреА рдЙрддреНрддрд░ рд╕реНрдерд╛рдкрд┐рдд рдирд╣реАрдВ рдХрд░рддрд╛ рд╣реИ рддреЛ рдЖрдкрдХреЛ рд╡рд╛рд╕реНрддрд╡ рдореЗрдВ рдХреМрди рд╕реА рддреНрд░реБрдЯрд┐ рдЧрдбрд╝рдмрдбрд╝ рдЙрдореНрд░ рдорд┐рд▓ рд░рд╣реА рд╣реИ

рдЕрдкрдиреЗ pip --version рдХреА рдЬрд╛рдБрдЪ рдХрд░реЗрдВ рдпрд╣ рдХрдо рд╕реЗ рдХрдо 19.0 рд╣реЛрдирд╛ рдЪрд╛рд╣рд┐рдП рдЕрдиреНрдпрдерд╛ tensorflow рд╕реНрдерд╛рдкрд┐рдд рдирд╣реАрдВ рд╣реЛрдЧрд╛ред рдЕрдЧрд░ рдпрд╣ рдЕрднреА рднреА рдЙрддреНрддрд░ рд╕реНрдерд╛рдкрд┐рдд рдирд╣реАрдВ рдХрд░рддрд╛ рд╣реИ рддреЛ рдЖрдкрдХреЛ рд╡рд╛рд╕реНрддрд╡ рдореЗрдВ рдХреМрди рд╕реА рддреНрд░реБрдЯрд┐ рдЧрдбрд╝рдмрдбрд╝ рдЙрдореНрд░ рдорд┐рд▓ рд░рд╣реА рд╣реИ

рдореИрдВрдиреЗ рдкрд╛рдЗрдк 19.2.3 рдХрд╛ рдЙрдкрдпреЛрдЧ рдХрд░рдХреЗ Tensorflow рдХрд╛ рдирд╡реАрдирддрдо рд╕рдВрд╕реНрдХрд░рдг (рдФрд░ рдкреБрд░рд╛рдиреЗ рд╕рдВрд╕реНрдХрд░рдг) рд╕реНрдерд╛рдкрд┐рдд рдХрд┐рдпрд╛ рд╣реИ рдФрд░ рдореИрдВ рдЕрднреА рднреА Tensorflow рдирд╣реАрдВ рдЪрд▓рд╛ рд╕рдХрддрд╛ред рдореИрдВ рдирд┐рдореНрдирд▓рд┐рдЦрд┐рдд рддреНрд░реБрдЯрд┐ рдореЗрдВ рдЪрд▓рддрд╛ рд░рд╣рддрд╛ рд╣реВрдВ:

ImportError: No module named tensorflow

рдШрдВрдЯреЛрдВ рд╢реЛрдз рдХреЗ рдмрд╛рдж рдореБрдЭреЗ рдХреЛрдИ рд╕рдорд╛рдзрд╛рди рдирд╣реАрдВ рдорд┐рд▓рд╛ рд╣реИред рдХреНрдпрд╛ рдХрд┐рд╕реА рдиреЗ рд╣рд╛рд▓ рд╣реА рдореЗрдВ рдЕрдЬрдЧрд░ 2 рдпрд╛ 3 рдХреЗ рд╕рд╛рде рдЯреЗрдВрд╕рд░рдлрд╝реНрд▓реЛ рдХреЛ рд╕рдлрд▓рддрд╛рдкреВрд░реНрд╡рдХ рд╕реНрдерд╛рдкрд┐рдд рдХрд┐рдпрд╛ рд╣реИ?

@ Bredding234 TensorFlow 2.0 (рд░рд┐рд▓реАрдЬрд╝ рдЙрдореНрдореАрджрд╡рд╛рд░ 0) рдореЗрдВ рд╡рд┐рдВрдбреЛрдЬрд╝ рдкрд░ 3.7 рдХреЗ рд▓рд┐рдП рдкрд┐рдкреНрд╕ рд╣реИрдВ, #31431 рджреЗрдЦреЗрдВред рдХреГрдкрдпрд╛ рдЙрд╕ рд╕рдВрд╕реНрдХрд░рдг рдХреЛ рдЖрдЬрд╝рдорд╛рдПрдВ рдФрд░ рд╣рдореЗрдВ рдмрддрд╛рдПрдВ рдХрд┐ рдХреНрдпрд╛ рдпрд╣ рдХрд╛рдо рдирд╣реАрдВ рдХрд░ рд░рд╣рд╛ рд╣реИред

@cskonopka рдЖрдк рдХрд┐рд╕ рдСрдкрд░реЗрдЯрд┐рдВрдЧ рд╕рд┐рд╕реНрдЯрдо рдХрд╛ рдЙрдкрдпреЛрдЧ рдХрд░ рд░рд╣реЗ рд╣реИрдВ? рдХреНрдпрд╛ рдЖрдк рд╡рд░реНрдЪреБрдЕрд▓рдПрдиреНрд╡ рдореЗрдВ рд╣реИрдВ? рдХреГрдкрдпрд╛ рд╕реБрдирд┐рд╢реНрдЪрд┐рдд рдХрд░реЗрдВ рдХрд┐ рд╕реНрдерд╛рдкрд┐рдд рдХрд░рдиреЗ рдХреЗ рд▓рд┐рдП рдЙрдкрдпреЛрдЧ рдХрд┐рдпрд╛ рдЬрд╛рдиреЗ рд╡рд╛рд▓рд╛ рдПрдХ рд╣реА рдкрд╛рдпрдерди рджреБрднрд╛рд╖рд┐рдпрд╛ рдкрд░реАрдХреНрд╖рдг рдХреЗ рд▓рд┐рдП рднреА рдЙрдкрдпреЛрдЧ рдХрд┐рдпрд╛ рдЬрд╛рддрд╛ рд╣реИред рдЙрджрд╛рд╣рд░рдг рдХреЗ рд▓рд┐рдП, рдЖрдк рдХрд░ рд╕рдХрддреЗ рд╣реИрдВ:

python -m pip install --upgrade pip
python -m pip install tensorflow
python -c "import tensorflow"

рдФрд░ рдпрд╣ рд╡рд┐рдлрд▓ рдирд╣реАрдВ рд╣реЛрдирд╛ рдЪрд╛рд╣рд┐рдПред

рдореБрдЭреЗ рдПрдХ рд╣реА рддреНрд░реБрдЯрд┐ рд╣реИред
tferr
рдордЬреЗрджрд╛рд░ рдмрд╛рдд рдпрд╣ рд╣реИ рдХрд┐ рдПрдирд╛рдХреЛрдВрдбрд╛ рд╡рд░реНрдЪреБрдЕрд▓рдПрдиреНрд╡ рдореЗрдВ TensorFlow рдЗрдВрд╕реНрдЯреЙрд▓ рдХрд░рдирд╛ рдареАрдХ рдХрд╛рдо рдХрд░рддрд╛ рд╣реИред

@hannesrollin pip рд╕рдВрд╕реНрдХрд░рдг? рдХреНрдпрд╛ рдЖрдкрдиреЗ https://github.com/tensorflow/tensorflow/issues/20444#issuecomment -524941378 рдореЗрдВ рджрд┐рдП рдЪрд░рдгреЛрдВ рдХрд╛ рдкрд╛рд▓рди рдХрд┐рдпрд╛?

рдпрд╣рд╛рдБ рдПрдХ рд╣реА рдореБрджреНрджрд╛,
рдЕрдЬрдЧрд░ рд╕рдВрд╕реНрдХрд░рдг 3.7
рдкрд╛рдЗрдк рд╕рдВрд╕реНрдХрд░рдг 19.2
рд╡рд┐рдВрдбреЛрдЬрд╝ 10 . рдкрд░

рдкрд╛рдЗрдк рдЯреЗрдВрд╕рд░рдлрд╝реНрд▓реЛ рд╕реНрдерд╛рдкрд┐рдд рдХрд░реЗрдВ
рддреНрд░реБрдЯрд┐: рдРрд╕рд╛ рд╕рдВрд╕реНрдХрд░рдг рдирд╣реАрдВ рдорд┐рд▓рд╛ рдЬреЛ рдЯреЗрдВрд╕рд░рдлрд╝реНрд▓реЛ рдХреА рдЖрд╡рд╢реНрдпрдХрддрд╛ рдХреЛ рдкреВрд░рд╛ рдХрд░рддрд╛ рд╣реЛ (рд╕рдВрд╕реНрдХрд░рдгреЛрдВ рд╕реЗ: рдХреЛрдИ рдирд╣реАрдВ)
рддреНрд░реБрдЯрд┐: рдЯреЗрдВрд╕рд░рдлрд╝реНрд▓реЛ рдХреЗ рд▓рд┐рдП рдХреЛрдИ рдорд┐рд▓рд╛рди рд╡рд┐рддрд░рдг рдирд╣реАрдВ рдорд┐рд▓рд╛

рдпрд╣рд╛рдБ рдПрдХ рд╣реА рдореБрджреНрджрд╛,
рдЕрдЬрдЧрд░ рд╕рдВрд╕реНрдХрд░рдг 3.7
рдкрд╛рдЗрдк рд╕рдВрд╕реНрдХрд░рдг 19.2
рд╡рд┐рдВрдбреЛрдЬрд╝ 10 . рдкрд░

рдкрд╛рдЗрдк рдЯреЗрдВрд╕рд░рдлрд╝реНрд▓реЛ рд╕реНрдерд╛рдкрд┐рдд рдХрд░реЗрдВ
рддреНрд░реБрдЯрд┐: рдРрд╕рд╛ рд╕рдВрд╕реНрдХрд░рдг рдирд╣реАрдВ рдорд┐рд▓рд╛ рдЬреЛ рдЯреЗрдВрд╕рд░рдлрд╝реНрд▓реЛ рдХреА рдЖрд╡рд╢реНрдпрдХрддрд╛ рдХреЛ рдкреВрд░рд╛ рдХрд░рддрд╛ рд╣реЛ (рд╕рдВрд╕реНрдХрд░рдгреЛрдВ рд╕реЗ: рдХреЛрдИ рдирд╣реАрдВ)
рддреНрд░реБрдЯрд┐: рдЯреЗрдВрд╕рд░рдлрд╝реНрд▓реЛ рдХреЗ рд▓рд┐рдП рдХреЛрдИ рдорд┐рд▓рд╛рди рд╡рд┐рддрд░рдг рдирд╣реАрдВ рдорд┐рд▓рд╛

рдХреГрдкрдпрд╛ рд╕реБрдирд┐рд╢реНрдЪрд┐рдд рдХрд░реЗрдВ рдХрд┐ рдЖрдк рдЬрд┐рд╕ рдкрд╛рдпрдерди рджреБрднрд╛рд╖рд┐рдпрд╛ рдХрд╛ рдЙрдкрдпреЛрдЧ рдХрд░ рд░рд╣реЗ рд╣реИрдВ рд╡рд╣ 64 рдмрд┐рдЯ рд╕рдВрд╕реНрдХрд░рдг рд╣реЛрдирд╛ рдЪрд╛рд╣рд┐рдП, рдЗрд╕рд╕реЗ рдореЗрд░реЗ рд▓рд┐рдП рд╕рдорд╕реНрдпрд╛ рд╣рд▓ рд╣реЛ рдЧрдИ рд╣реИ рдЗрд╕реЗ рдЖрдЬрд╝рдорд╛рдПрдВред

рдкреНрд░рдгрд╛рд▓реА рдХреА рдЬрд╛рдирдХрд╛рд░реА

  • рдХреНрдпрд╛ рдореИрдВрдиреЗ рдХрд╕реНрдЯрдо рдХреЛрдб рд▓рд┐рдЦрд╛ рд╣реИ (рдЬреИрд╕рд╛ рдХрд┐ TensorFlow рдореЗрдВ рдкреНрд░рджрд╛рди рдХреА рдЧрдИ рд╕реНрдЯреЙрдХ рдЙрджрд╛рд╣рд░рдг рд╕реНрдХреНрд░рд┐рдкреНрдЯ рдХрд╛ рдЙрдкрдпреЛрдЧ рдХрд░рдиреЗ рдХреЗ рд╡рд┐рдкрд░реАрдд) : N/A
  • OS рдкреНрд▓реЗрдЯрдлреЙрд░реНрдо рдФрд░ рд╡рд┐рддрд░рдг (рдЬреИрд╕реЗ, Linux Ubuntu 16.04) : macOS 10.13
  • TensorFlow (рд╕реНрд░реЛрдд рдпрд╛ рдмрд╛рдЗрдирд░реА) рд╕реЗ рд╕реНрдерд╛рдкрд┐рдд : рдмрд╛рдЗрдирд░реА
  • TensorFlow рд╕рдВрд╕реНрдХрд░рдг (рдиреАрдЪреЗ рдХрдорд╛рдВрдб рдХрд╛ рдЙрдкрдпреЛрдЧ рдХрд░реЗрдВ) : 1.8
  • рдкрд╛рдпрдерди рд╕рдВрд╕реНрдХрд░рдг : 3.7
  • рдмреЗрдЬрд╝рд▓ рд╕рдВрд╕реНрдХрд░рдг (рдпрджрд┐ рд╕реНрд░реЛрдд рд╕реЗ рд╕рдВрдХрд▓рд┐рдд рдХрд┐рдпрд╛ рдЬрд╛ рд░рд╣рд╛ рд╣реИ) : рдПрди/рдП
  • рдЬреАрд╕реАрд╕реА/рдХрдВрдкрд╛рдЗрд▓рд░ рд╕рдВрд╕реНрдХрд░рдг (рдпрджрд┐ рд╕реНрд░реЛрдд рд╕реЗ рд╕рдВрдХрд▓рд┐рдд рдХрд┐рдпрд╛ рдЬрд╛ рд░рд╣рд╛ рд╣реИ) : рд▓рд╛рдЧреВ рдирд╣реАрдВ
  • CUDA/cuDNN рд╕рдВрд╕реНрдХрд░рдг : рдПрди/рдП
  • GPU рдореЙрдбрд▓ рдФрд░ рдореЗрдореЛрд░реА : N/A
  • рдкреБрди: рдкреЗрд╢ рдХрд░рдиреЗ рдХреЗ рд▓рд┐рдП рд╕рдЯреАрдХ рдЖрджреЗрд╢ : pip install tensorflow

рд╕рдорд╕реНрдпрд╛ рд╡рд┐рд╕реНрддрд╛рд░ рд╕реЗ рд╕рдордЭрд╛рдЗрдпреЗ

рдкрд╛рдЗрдк рдХреЗ рд╕рд╛рде Python3.7 рдкрд░ TensorFlow рд╕реНрдерд╛рдкрд┐рдд рдХрд░рдирд╛ рд╡рд┐рдлрд▓ рд░рд╣рд╛ред рдХреГрдкрдпрд╛ рдиреАрдЪреЗ рд╡рд┐рдлрд▓рддрд╛ рд▓реЙрдЧ рджреЗрдЦреЗрдВред

рд╕реНрд░реЛрдд рдХреЛрдб / рд▓реЙрдЧ

рдРрд╕рд╛ рд╕рдВрд╕реНрдХрд░рдг рдирд╣реАрдВ рдорд┐рд▓рд╛ рдЬреЛ рдЯреЗрдВрд╕рд░рдлрд╝реНрд▓реЛ рдХреА рдЖрд╡рд╢реНрдпрдХрддрд╛ рдХреЛ рдкреВрд░рд╛ рдХрд░рддрд╛ рд╣реЛ (рд╕рдВрд╕реНрдХрд░рдгреЛрдВ рд╕реЗ:)
рдЯреЗрдВрд╕рд░рдлрд╝реНрд▓реЛ рдХреЗ рд▓рд┐рдП рдХреЛрдИ рдорд┐рд▓рд╛рди рд╡рд┐рддрд░рдг рдирд╣реАрдВ рдорд┐рд▓рд╛

рдХреГрдкрдпрд╛ рд╕реБрдирд┐рд╢реНрдЪрд┐рдд рдХрд░реЗрдВ рдХрд┐ рдЖрдк рдЬрд┐рд╕ рдкрд╛рдпрдерди рджреБрднрд╛рд╖рд┐рдпрд╛ рдХрд╛ рдЙрдкрдпреЛрдЧ рдХрд░ рд░рд╣реЗ рд╣реИрдВ рд╡рд╣ 64 рдмрд┐рдЯ рд╕рдВрд╕реНрдХрд░рдг рд╣реЛрдирд╛ рдЪрд╛рд╣рд┐рдП, рдЗрд╕рд╕реЗ рдореЗрд░реЗ рд▓рд┐рдП рд╕рдорд╕реНрдпрд╛ рд╣рд▓ рд╣реЛ рдЧрдИ рд╣реИ рдЗрд╕реЗ рдЖрдЬрд╝рдорд╛рдПрдВред

32 рдмрд┐рдЯ рдмрдирд╛рдо 64 рдмрд┐рдЯ рдкрд╛рдпрдерди рджреБрднрд╛рд╖рд┐рдпреЛрдВ рдХреЗ рд▓рд┐рдП, #32315 рдФрд░ #31431 рджреЗрдЦреЗрдВ

рдбрд┐рдмрдЧ рдХрд░рдиреЗ рдХреЗ рд▓рд┐рдП рдХрд┐ рдЖрдк рдЯреЗрдВрд╕рд░рдлрд╝реНрд▓реЛ рдХреНрдпреЛрдВ рд╕реНрдерд╛рдкрд┐рдд рдирд╣реАрдВ рдХрд░ рд╕рдХрддреЗ, pip debug --verbose рдХреЛ рдЖрдкрдХреЗ рдкрд░рд┐рд╡реЗрд╢ рджреНрд╡рд╛рд░рд╛ рд╕рдорд░реНрдерд┐рдд рдЯреИрдЧ рдХреА рд╕реВрдЪреА рдкреНрд░рджрд░реНрд╢рд┐рдд рдХрд░рдиреА рдЪрд╛рд╣рд┐рдПред рдлрд┐рд░ рдбрд╛рдЙрдирд▓реЛрдб рдлрд╝рд╛рдЗрд▓реЗрдВ рджреЗрдЦреЗрдВ (рдпрд╣рд╛рдВ 2.0 рдХреЗ рд▓рд┐рдП, рд▓реЗрдХрд┐рди рдЖрдк рджреВрд╕рд░реЛрдВ рдХреА рднреА рдЬрд╛рдВрдЪ рдХрд░ рд╕рдХрддреЗ рд╣реИрдВ) рдФрд░ рджреЗрдЦреЗрдВ рдХрд┐ рдЖрдкрдХрд╛ рдЯреИрдЧ рд╡рд╣рд╛рдВ рд╣реИ рдпрд╛ рдирд╣реАрдВред рдЕрдЧрд░ рд╣реИ рддреЛ рдпрд╣ рд╣рдорд╛рд░реЗ рд▓рд┐рдП рдПрдХ рдорд╕рд▓рд╛ рд╣реИ, рдирд╣реАрдВ рддреЛ рдпрд╣ рдХреЛрдИ рдЗрд╢реНрдпреВ рд░рд┐рдкреЛрдЬрд┐рдЯрд░реА рдореЗрдВ рдирд╣реАрдВ рдЦреБрд▓рдирд╛ рдЪрд╛рд╣рд┐рдПред

рдореЗрд░реЗ рдкрд╛рд╕ рдПрдХ рдЙрддреНрддрд░ рд╣реИ рдХреНрдпреЛрдВрдХрд┐ рдЕрдЬрдЧрд░ 3.7 рдЯреЗрдВрд╕рд░рдлрд╝реНрд▓реЛ рдХрд╛рдо рдирд╣реАрдВ рдХрд░ рд░рд╣рд╛ рд╣реИ
https://www.youtube.com/watch?v=_kU50QtlI14 рд▓рд┐рдВрдХ рдкрд░ рдЗрди рдЪрд░рдгреЛрдВ рдХрд╛ рдкрд╛рд▓рди рдХрд░рдХреЗ рдЕрдЬрдЧрд░ 3.6 рдЖрдЬрд╝рдорд╛рдПрдВ

рдореЗрд░реЗ рд▓рд┐рдпреЗ рдХрд╛рд░реНрдп рдХрд░рддрд╛ рд╣реИ!

рдореЗрд░рд╛ рдЕрдЬрдЧрд░ 3.8.0 рд╣реИ рдЗрд╕рдХрд╛ рдорддрд▓рдм рд╣реИ рдХрд┐ рдореБрдЭреЗ рдЯреЗрдВрд╕рд░рдлрд╝реНрд▓реЛ рдХрд╛ рдЙрдкрдпреЛрдЧ рдХрд░рдиреЗ рдХреЗ рд▓рд┐рдП рдбрд╛рдЙрдирдЧреНрд░реЗрдб рдХрд░рдирд╛ рд╣реЛрдЧрд╛?

рдореЗрд░рд╛ рдЕрдЬрдЧрд░ 3.8.0 рд╣реИ рдЗрд╕рдХрд╛ рдорддрд▓рдм рд╣реИ рдХрд┐ рдореБрдЭреЗ рдЯреЗрдВрд╕рд░рдлрд╝реНрд▓реЛ рдХрд╛ рдЙрдкрдпреЛрдЧ рдХрд░рдиреЗ рдХреЗ рд▓рд┐рдП рдбрд╛рдЙрдирдЧреНрд░реЗрдб рдХрд░рдирд╛ рд╣реЛрдЧрд╛?

рдореБрдЭреЗ рд▓рдЧрддрд╛ рд╣реИ рдХрд┐ рдпрд╣ рдЕрдм рдХреЗ рд▓рд┐рдП рдЕрдзрд┐рдХ рд╕реБрд░рдХреНрд╖рд┐рдд рд╣реИ , 3.8 рдХреЗ рдмрдЬрд╛рдп рдЕрдЬрдЧрд░ 3.7 рдХрд╛ рдЙрдкрдпреЛрдЧ рдХрд░реЗрдВ рдХреНрдпреЛрдВрдХрд┐ рд╕рднреА рдПрдордПрд▓/рдбреАрдПрд▓ рдкреИрдХреЗрдЬ рдЕрдЬрдЧрд░ 3.8 рдХреЗ рд▓рд┐рдП рдХреЛрдИ рдирдпрд╛ рд╕рдВрд╕реНрдХрд░рдг рдЬрд╛рд░реА рдирд╣реАрдВ рдХрд░рддреЗ рд╣реИрдВред рдореИрдВ рдЬрдирд╡рд░реА 2020 рдХреА рд╢реБрд░реБрдЖрдд рд╕реЗ py3.8 рдХрд╛ рдЙрдкрдпреЛрдЧ рд╢реБрд░реВ рдХрд░рдиреЗ рдХреА рд╕рд▓рд╛рд╣ рджреЗрддрд╛ рд╣реВрдВред
(рдХреБрдЫ рдкреИрдХреЗрдЬреЛрдВ рдХреЛ 3.8 рдХреЗ рддрд╣рдд 3.7 рдХрд╛рдо рдХреЗ рд░реВрдк рдореЗрдВ рдЕрджреНрдпрддрди рдХреА рдЖрд╡рд╢реНрдпрдХрддрд╛ рдирд╣реАрдВ рд╣реЛрдЧреА, рд▓реЗрдХрд┐рди рджреВрд╕рд░рд╛ рддрд░реАрдХрд╛ рдХрд╛рдо рдирд╣реАрдВ рдХрд░рддрд╛ рд╣реИред -> рджреЗрдЦрддреЗ рд╣реИрдВ рдХрд┐ рд╕рдореБрджрд╛рдп рдЕрдЧрд▓реЗ рдХреБрдЫ рдорд╣реАрдиреЛрдВ рдореЗрдВ py3.8 рдХрд╛ рдЬрд╡рд╛рдм рдХреИрд╕реЗ рджреЗрдЧрд╛ )ред

#33374 . рдореЗрдВ Python3.8 рд╕рдорд░реНрдерди рд╣реИ

рдпрд╣ рд╕рдорд╕реНрдпрд╛ 32 рдмрд┐рдЯ рджреБрднрд╛рд╖рд┐рдпрд╛ рдХреЗ рд╕рд╛рде рд╡рд┐рдВрдбреЛрдЬрд╝ рдкрд░ python3.7 рдХреЗ рдмрд╛рд░реЗ рдореЗрдВ рд╣реИред рдХреГрдкрдпрд╛ рдореБрджреНрджреЛрдВ рдХреЛ рднреНрд░рдорд┐рдд рди рдХрд░реЗрдВред

рдореБрдЭреЗ рд▓рдЧрддрд╛ рд╣реИ рдХрд┐ TensorFlow рдЕрднреА рддрдХ 3.8.0 рдЕрдЬрдЧрд░ рдХрд╛ рд╕рдорд░реНрдерди рдирд╣реАрдВ рдХрд░рддрд╛ рд╣реИ рдпрд╛ рдЗрд╕ рд╕рдВрд╕реНрдХрд░рдг рдореЗрдВ рд╕рдорд░реНрдерди рдирд╣реАрдВ рд╣реИред рдореИрдВ рдЗрд╕реА рд╕рдорд╕реНрдпрд╛ рд╕реЗ рдЧреБрдЬрд░рддрд╛ рд╣реВрдВ, рдФрд░ рдореИрдВрдиреЗ рдПрдирд╛рдХреЛрдВрдбрд╛ рдореЗрдВ рдЕрдЬрдЧрд░ 3.5.2 рдХрд╛ рд╡рд╛рддрд╛рд╡рд░рдг рдмрдирд╛рдиреЗ рдХрд╛ рд╕рдорд╛рдзрд╛рди рдХрд┐рдпрд╛, рдФрд░ рдпрд╣ рдореЗрд░реЗ рд▓рд┐рдП рдареАрдХ рдХрд╛рдо рдХрд░рддрд╛ рд╣реИред рдЕрдиреБрдорд╛рди рд▓рдЧрд╛рддреЗ рд╣реБрдП, рдпрджрд┐ рдЖрдк рдЯреЗрдВрд╕рд░рдлрд╝реНрд▓реЛ-рдЬреАрдкреАрдпреВ рдХрд╛ рдЙрдкрдпреЛрдЧ рдХрд░реЗрдВрдЧреЗ рддреЛ рдЖрдк рдЙрд╕реА рд╕рдорд╕реНрдпрд╛ рд╕реЗ рдЧреБрдЬрд░ рд╕рдХрддреЗ рд╣реИрдВ, рдХреНрдпреЛрдВрдХрд┐ CUDA 11 рдХреЗ рд▓рд┐рдП рдЕрднреА рддрдХ рд╕рдорд░реНрдерди рдирд╣реАрдВ рд╣реИ, рдФрд░ рд╡рд░реНрддрдорд╛рди cudnn рд╕рдВрд╕реНрдХрд░рдг рд╕реЗ, рдЗрд╕рд▓рд┐рдП рдореБрдЭреЗ рдПрдХ рдбреВрдмрдиреЗ рд╡рд╛рд▓рд╛ рднреА рдХрд░рдиреЗ рдХреА рдЖрд╡рд╢реНрдпрдХрддрд╛ рд╣реИред

TensorFlow рд╡рд░реНрддрдорд╛рди рдореЗрдВ рдЕрдЬрдЧрд░ 3.7 рдХрд╛ рд╕рдорд░реНрдерди рдирд╣реАрдВ рдХрд░рддрд╛ рд╣реИред рд▓реЗрдХрд┐рди рдореИрдВрдиреЗ рд╡рд░реНрдЪреБрдЕрд▓рдПрдиреНрд╡, рдХреЛрдВрдбрд╛ рдЬреИрд╕реЗ рдкрд╛рдпрдерди рд╡рд╛рддрд╛рд╡рд░рдг рдХрд╛ рдЙрдкрдпреЛрдЧ рдХрд░рдХреЗ рдЗрд╕ рдореБрджреНрджреЗ рдХреЛ рд╣рд▓ рдХрд┐рдпрд╛ рд╣реИ (рд╡рд┐рдВрдбреЛрдЬрд╝ 10 рдкрд░)ред

рд╕рдмрд╕реЗ рдкрд╣рд▓реЗ, рдЖрдкрдХреЛ рдЕрдЬрдЧрд░ 3.5x рдпрд╛ 3.6x >> https://www.python.org/downloads/ рдбрд╛рдЙрдирд▓реЛрдб рдХрд░рдирд╛ рд╣реЛрдЧрд╛
рдФрд░ рдЗрд╕реЗ рдЕрдкрдиреА рдорд╢реАрди рдореЗрдВ рд╕реНрдерд╛рдкрд┐рдд рдХрд░реЗрдВред рддреЛ рдЖрдкрдХреЛ Python 3.6 рдпрд╛ 3.5 рдХреЗ рд╕рд╛рде рд╡рд░реНрдЪреБрдЕрд▓ рд╡рд╛рддрд╛рд╡рд░рдг рдмрдирд╛рдирд╛ рд╣реЛрдЧрд╛ред
(рд╡рд░реНрдЪреБрдЕрд▓рдПрдиреНрд╡-рдкреА {рдкрд╛рдпрдерди рдХрд╛ рдкрде (3.6)} {рдкрд░реНрдпрд╛рд╡рд░рдг рдХрд╛ рдирд╛рдо})ред

рдЕрдВрдд рдореЗрдВ, рдЖрднрд╛рд╕реА рд╡рд╛рддрд╛рд╡рд░рдг рдХреЛ рд╕рдХреНрд░рд┐рдп рдХрд░рдиреЗ рдХреЗ рд╕рд╛рде рдЕрдкрдиреЗ рдЕрдЬрдЧрд░ рд╕рдВрд╕реНрдХрд░рдг рдХреА рдЬрд╛рдБрдЪ рдХрд░реЗрдВред

рдпрд╣рд╛рдБ рдПрдХ рд╣реА рдореБрджреНрджрд╛,
рдЕрдЬрдЧрд░ рд╕рдВрд╕реНрдХрд░рдг 3.7
рдкрд╛рдЗрдк рд╕рдВрд╕реНрдХрд░рдг 19.2
рд╡рд┐рдВрдбреЛрдЬрд╝ 10 . рдкрд░

рдкрд╛рдЗрдк рдЯреЗрдВрд╕рд░рдлрд╝реНрд▓реЛ рд╕реНрдерд╛рдкрд┐рдд рдХрд░реЗрдВ
рддреНрд░реБрдЯрд┐: рдРрд╕рд╛ рд╕рдВрд╕реНрдХрд░рдг рдирд╣реАрдВ рдорд┐рд▓рд╛ рдЬреЛ рдЯреЗрдВрд╕рд░рдлрд╝реНрд▓реЛ рдХреА рдЖрд╡рд╢реНрдпрдХрддрд╛ рдХреЛ рдкреВрд░рд╛ рдХрд░рддрд╛ рд╣реЛ (рд╕рдВрд╕реНрдХрд░рдгреЛрдВ рд╕реЗ: рдХреЛрдИ рдирд╣реАрдВ)
рддреНрд░реБрдЯрд┐: рдЯреЗрдВрд╕рд░рдлрд╝реНрд▓реЛ рдХреЗ рд▓рд┐рдП рдХреЛрдИ рдорд┐рд▓рд╛рди рд╡рд┐рддрд░рдг рдирд╣реАрдВ рдорд┐рд▓рд╛

рдХреГрдкрдпрд╛ рд╕реБрдирд┐рд╢реНрдЪрд┐рдд рдХрд░реЗрдВ рдХрд┐ рдЖрдк рдЬрд┐рд╕ рдкрд╛рдпрдерди рджреБрднрд╛рд╖рд┐рдпрд╛ рдХрд╛ рдЙрдкрдпреЛрдЧ рдХрд░ рд░рд╣реЗ рд╣реИрдВ рд╡рд╣ 64 рдмрд┐рдЯ рд╕рдВрд╕реНрдХрд░рдг рд╣реЛрдирд╛ рдЪрд╛рд╣рд┐рдП, рдЗрд╕рд╕реЗ рдореЗрд░реЗ рд▓рд┐рдП рд╕рдорд╕реНрдпрд╛ рд╣рд▓ рд╣реЛ рдЧрдИ рд╣реИ рдЗрд╕реЗ рдЖрдЬрд╝рдорд╛рдПрдВред

рд╣рд╛рдВ рдореЗрд░реА рддрд░рд╣

рдпрд╣ рдореЗрд░реЗ рд▓рд┐рдП рдХрд╛рдо рд╣реИ
рдмрд╕ рдЗрд╕реЗ рдиреАрдЪреЗ рдЙрдкрдпреЛрдЧ рдХрд░рдиреЗ рдХрд╛ рдкреНрд░рдпрд╛рд╕ рдХрд░реЗрдВ
рдкрд╛рдЗрдк рд╕реНрдерд╛рдкрд┐рдд рдХрд░реЗрдВ https://storage.googleapis.com/tensorflow/mac/cpu/tensorflow-1.8.0-py3-none-any.whl

рдпрд╣ рдореЗрд░реЗ рд▓рд┐рдП рдХрд╛рдо рдХрд┐рдпрд╛ред рдзрдиреНрдпрд╡рд╛рдж

рдпрд╣ рдореЗрд░реЗ рд▓рд┐рдП рднреА рдХрд╛рдо рдХрд┐рдпрд╛ред рдзрдиреНрдпрд╡рд╛рдж

рдХреГрдкрдпрд╛, рдореИрдВ рдЙрдмрдВрдЯреВ рдХрд╛ рдЙрдкрдпреЛрдЧ рдХрд░рддрд╛ рд╣реВрдВ
рдЕрдЬрдЧрд░ 3.8.0
рдкрд┐рдк 19.3.1
рдЬрдм рдореИрдВ рдЯреЗрдВрд╕рд░рдлрд╝реНрд▓реЛ рд╕реНрдерд╛рдкрд┐рдд рдХрд░рддрд╛ рд╣реВрдБ рддреЛ рдореБрдЭреЗ рдпрд╣ рд╕рдорд╕реНрдпрд╛ рдорд┐рд▓рддреА рд╣реИ
"рдРрд╕рд╛ рд╕рдВрд╕реНрдХрд░рдг рдирд╣реАрдВ рдорд┐рд▓рд╛ рдЬреЛ рдЯреЗрдВрд╕рд░рдлрд╝реНрд▓реЛ рдХреА рдЖрд╡рд╢реНрдпрдХрддрд╛ рдХреЛ рдкреВрд░рд╛ рдХрд░рддрд╛ рд╣реЛ (рд╕рдВрд╕реНрдХрд░рдгреЛрдВ рд╕реЗ рдХреЛрдИ рдирд╣реАрдВ)"

@ayness рдХреГрдкрдпрд╛ рдкрд┐рдЫрд▓реЗ рд╕рдВрджреЗрд╢ рдкрдврд╝реЗрдВ

рд╡рд┐рд╢реЗрд╖ рд░реВрдк рд╕реЗ, рдЖрдкрдХрд╛ рдЙрдкрдпреЛрдЧ рдорд╛рдорд▓рд╛ https://github.com/tensorflow/tensorflow/issues/20444#issuecomment -545036560 рдФрд░ #33374 рджреНрд╡рд╛рд░рд╛ рдХрд╡рд░ рдХрд┐рдпрд╛ рдЧрдпрд╛ рд╣реИ

@ ShriLingam23 рдпрд╣ рдореБрджреНрджрд╛ python3.7 рдХреЗ рдмрд╛рд░реЗ рдореЗрдВ рд╣реИред рддреЛ рдЕрдЬрдЧрд░ 3.8 рд╕реЗ 3.7.6 рддрдХ рдбрд╛рдЙрдирдЧреНрд░реЗрдб рдХрд░рдиреЗ рдХреЗ рдмрд╛рд░реЗ рдореЗрдВ рдЖрдкрдХрд╛ рд╕рдорд╛рдзрд╛рди рджрд╛рдпрд░реЗ рд╕реЗ рдмрд╛рд╣рд░ рд╣реИред рдХреГрдкрдпрд╛ рдмрдВрдж рдореБрджреНрджреЛрдВ рдкрд░ рдЯрд┐рдкреНрдкрдгреА рди рдХрд░реЗрдВ рдХреНрдпреЛрдВрдХрд┐ рдЗрд╕рд╕реЗ рдХреЗрд╡рд▓ рдЕрдзрд┐рдХ рднреНрд░рдо рдкреИрджрд╛ рд╣реЛрддрд╛ рд╣реИ

@NIravMeghani рдХреГрдкрдпрд╛ рдЕрд╕рдВрдмрдВрдзрд┐рдд рдореБрджреНрджреЛрдВ рдХреЛ рд╕реНрдкреИрдо рдХрд░рдирд╛ рдмрдВрдж рдХрд░реЗрдВред

TF 1.8 рдкрд╣рд▓реЗ рд╕реЗ рд╣реА рдХреБрдЫ рд╡рд░реНрд╖реЛрдВ рдХреЗ рд▓рд┐рдП рдЬреАрд╡рди рд╕реЗ рдмрд╛рд╣рд░ рд╣реИред рдХреГрдкрдпрд╛ TF 1.15, 2.0 рдпрд╛ 2.1 рдХрд╛ рдЙрдкрдпреЛрдЧ рдХрд░реЗрдВред

TF рдХреЗ рдкрд╛рд╕ рдлрд┐рд▓рд╣рд╛рд▓ рдЕрдЬрдЧрд░ 8 рдХреЗ рд▓рд┐рдП рдХреЛрдИ рд╕рдорд░реНрдерди рдирд╣реАрдВ рд╣реИред рдЖрдк рдЪрд╛рд╣реЗрдВ рддреЛ рд╕реНрд░реЛрдд рд╕реЗ рд╕рдВрдХрд▓рди рдХрд░ рд╕рдХрддреЗ рд╣реИрдВред рд╣рдо рдЬрд▓реНрдж рд╣реА рдЕрдЬрдЧрд░ 8 рд╕рдорд░реНрдерди рдЬрд╛рд░реА рдХрд░рдиреЗ рдкрд░ рдХрд╛рдо рдХрд░ рд░рд╣реЗ рд╣реИрдВред

рднрд╡рд┐рд╖реНрдп рдореЗрдВ, рдХреГрдкрдпрд╛ рдЕрд╕рдВрдмрдВрдзрд┐рдд рдореБрджреНрджреЛрдВ рдкрд░ рдкреАрда рдердкрдердкрд╛рдиреЗ рдХреЗ рдмрдЬрд╛рдп рдПрдХ рдирдпрд╛ рдЕрдВрдХ рдЦреЛрд▓реЗрдВред

рдпрд╣ рдореЗрд░реЗ рд▓рд┐рдП рдХрд╛рдо рд╣реИ
рдмрд╕ рдЗрд╕реЗ рдиреАрдЪреЗ рдЙрдкрдпреЛрдЧ рдХрд░рдиреЗ рдХрд╛ рдкреНрд░рдпрд╛рд╕ рдХрд░реЗрдВ
рдкрд╛рдЗрдк рд╕реНрдерд╛рдкрд┐рдд рдХрд░реЗрдВ https://storage.googleapis.com/tensorflow/mac/cpu/tensorflow-1.8.0-py3-none-any.whl

рдореЗрд░реЗ рд▓рд┐рдП рднреА рдХрд╛рдо рдХрд┐рдпрд╛ред рдзрдиреНрдпрд╡рд╛рджред

@ ravi-ai-cons рдХреГрдкрдпрд╛ "рдореЗрд░реЗ рд▓рд┐рдП рдХрд╛рдо рдХрд┐рдпрд╛, рдзрдиреНрдпрд╡рд╛рдж" рд╕рдВрджреЗрд╢реЛрдВ рдХреЗ рд╕рд╛рде рдмрдВрдж рдореБрджреНрджреЛрдВ рдХреЛ рдЕрдкрдбреЗрдЯ рди рдХрд░реЗрдВ рдХреНрдпреЛрдВрдХрд┐ рдпреЗ рд╕рдВрджреЗрд╢ рдЙрди рд╕рднреА рдХреЛ рдЕрдирд╛рд╡рд╢реНрдпрдХ рдИрдореЗрд▓ рднреЗрдЬрддреЗ рд╣реИрдВ рдЬрд┐рдиреНрд╣реЛрдВрдиреЗ рдЗрд╕ рдореБрджреНрджреЗ рдкрд░ рдЙрддреНрддрд░ рджрд┐рдпрд╛ рдФрд░ рдЗрди рд╕рдВрджреЗрд╢реЛрдВ рдореЗрдВ рдХреЛрдИ рд╡рд╛рд╕реНрддрд╡рд┐рдХ рд╕рд╛рдордЧреНрд░реА рдирд╣реАрдВ рд╣реИред

рдореИрдВрдиреЗ рдЗрд╕ рдореБрджреНрджреЗ рдХреЛ рдЕрдЬрдЧрд░ 3.8 рдкрд░ рдЗрд╕рдХрд╛ рдЙрдкрдпреЛрдЧ рдХрд░рдХреЗ рддрдп рдХрд┐рдпрд╛:
1/ рдореЗрд░реЗ рдорд╛рдорд▓реЗ рдореЗрдВ рдЕрдкрдиреЗ рдЕрдЬрдЧрд░ рдореЗрдВ Scipts рдХреА рдирд┐рд░реНрджреЗрд╢рд┐рдХрд╛ рдкрд░ рдЬрд╛рдПрдВ: C:\Python\Python38-32\Scripts
2/ рд╕реАрдПрдордбреА рдкреНрд░рдХрд╛рд░ рдореЗрдВ: рд╕реАрдбреА рд╕реА:\рдкрд╛рдпрдерди\рдкрд╛рдпрдерди38-32\рд╕реНрдХреНрд░рд┐рдкреНрдЯ
рдлрд┐рд░ рдЯрд╛рдЗрдк рдХрд░реЗрдВ: pip install https://storage.googleapis.com/tensorflow/mac/cpu/tensorflow-1.8.0-py3-none-any.whl

@KaberMed рдпрд╣ рд╕рдорд╕реНрдпрд╛ python3.7 рдХреЗ рдмрд╛рд░реЗ рдореЗрдВ рд╣реИ, рди рдХрд┐ python3.8 рдХреЗ рдмрд╛рд░реЗ рдореЗрдВред рдХреГрдкрдпрд╛ рд╕реНрдкреИрдо рди рдХрд░реЗрдВ

рдореИрдВрдиреЗ рдЗрд╕реЗ рд╕рдлрд▓рддрд╛рдкреВрд░реНрд╡рдХ рд╕реНрдерд╛рдкрд┐рдд рдХрд┐рдпрд╛ pip install https://storage.googleapis.com/tensorflow/mac/cpu/tensorflow-1.8.0-py3-none-any.whl ред рд╣рд╛рд▓рд╛рдБрдХрд┐, рдРрд╕рд╛ рд▓рдЧрддрд╛ рд╣реИ рдХрд┐ рдпрд╣ рдкрд╣рд┐рдпрд╛ Python 3.7 рд╕рдВрдЧрдд рдирд╣реАрдВ рд╣реИред рд╕рдорд╕реНрдпрд╛ рдпрд╣ рд╣реИ рдХрд┐ tensorflow/python/pywrap_tensorflow_internal.py рдореЗрдВ рдЪрд░ рдирд╛рдо рдХреЗ рд░реВрдк рдореЗрдВ async рд╣реИ рд▓реЗрдХрд┐рди async python3.7 рдореЗрдВ рдПрдХ рдХреАрд╡рд░реНрдб рдмрди рдЬрд╛рддрд╛ рд╣реИ, рдЗрд╕рд▓рд┐рдП рдпрд╣ рдЕрдм рдЙрдкрд▓рдмреНрдз рдирд╣реАрдВ рд╣реИред

рд╕рд╛рде рд╣реА, рдореИрдВрдиреЗ рджреЗрдЦрд╛ рдХрд┐ рдпреВрдЖрд░рдПрд▓ рдХреЗрд╡рд▓ рдкрд╛рдпрдерди 3.4, 3.5, 3.6 рдХреЗ рд▓рд┐рдП рд╣реИ рд▓реЗрдХрд┐рди рдореБрдЭреЗ рдХреЛрдИ рдЕрдиреНрдп рдпреВрдЖрд░рдПрд▓ рдирд╣реАрдВ рдорд┐рд▓ рд░рд╣рд╛ рд╣реИред рддреЛ рд╢рд╛рдпрдж рд╣рдореЗрдВ рдЖрдзрд┐рдХрд╛рд░рд┐рдХ рдкрд╛рдпрдерди 3.7-рд╕рдВрдЧрдд рдкреИрдХреЗрдЬ рдХреА рдкреНрд░рддреАрдХреНрд╖рд╛ рдХрд░рдиреЗ рдХреА рдЖрд╡рд╢реНрдпрдХрддрд╛ рд╣реИред

рдореИрдВрдиреЗ рдЗрд╕реЗ рд╕рдлрд▓рддрд╛рдкреВрд░реНрд╡рдХ рд╕реНрдерд╛рдкрд┐рдд рдХрд┐рдпрд╛ pip install https://storage.googleapis.com/tensorflow/mac/cpu/tensorflow-1.8.0-py3-none-any.whl ред рд╣рд╛рд▓рд╛рдБрдХрд┐, рдРрд╕рд╛ рд▓рдЧрддрд╛ рд╣реИ рдХрд┐ рдпрд╣ рдкрд╣рд┐рдпрд╛ Python 3.7 рд╕рдВрдЧрдд рдирд╣реАрдВ рд╣реИред рд╕рдорд╕реНрдпрд╛ рдпрд╣ рд╣реИ рдХрд┐ tensorflow/python/pywrap_tensorflow_internal.py рдореЗрдВ рдЪрд░ рдирд╛рдореЛрдВ рдХреЗ рд░реВрдк рдореЗрдВ async рд╣реИ рд▓реЗрдХрд┐рди async python3.7 рдореЗрдВ рдПрдХ рдХреАрд╡рд░реНрдб рдмрди рдЬрд╛рддрд╛ рд╣реИ, рдЗрд╕рд▓рд┐рдП рдпрд╣ рдЕрдм рдЙрдкрд▓рдмреНрдз рдирд╣реАрдВ рд╣реИред

рд╕рд╛рде рд╣реА, рдореИрдВрдиреЗ рджреЗрдЦрд╛ рдХрд┐ рдпреВрдЖрд░рдПрд▓ рдХреЗрд╡рд▓ рдкрд╛рдпрдерди 3.4, 3.5, 3.6 рдХреЗ рд▓рд┐рдП рд╣реИ рд▓реЗрдХрд┐рди рдореБрдЭреЗ рдХреЛрдИ рдЕрдиреНрдп рдпреВрдЖрд░рдПрд▓ рдирд╣реАрдВ рдорд┐рд▓ рд░рд╣рд╛ рд╣реИред рддреЛ рд╢рд╛рдпрдж рд╣рдореЗрдВ рдЖрдзрд┐рдХрд╛рд░рд┐рдХ рдкрд╛рдпрдерди 3.7-рд╕рдВрдЧрдд рдкреИрдХреЗрдЬ рдХреА рдкреНрд░рддреАрдХреНрд╖рд╛ рдХрд░рдиреЗ рдХреА рдЖрд╡рд╢реНрдпрдХрддрд╛ рд╣реИред

рдЗрд╕рд╕реЗ рдореБрдЭреЗ рдорджрдж рдорд┐рд▓реА, рдзрдиреНрдпрд╡рд╛рдж!
рд╡реАрдПрд╕рд╕реА рдореЗрдВ, рдкрд╛рдпрдерди 3.7.2

https://www.tensorflow.org/install/pip рдХрд╛ рдХрд╣рдирд╛ рд╣реИ рдХрд┐ 64 рдмрд┐рдЯ рдЕрдЬрдЧрд░ рдХреЗ рд▓рд┐рдП рдЕрдЬрдЧрд░ 3.7 рд╕рдорд░реНрдерди рдХреЗ рд▓рд┐рдП рдПрдХ рд╡рд┐рдВрдбреЛрдЬрд╝ рд╕рдВрд╕реНрдХрд░рдг рдореМрдЬреВрдж рд╣реИред
pip install https://storage.googleapis.com/tensorflow/windows/cpu/tensorflow_cpu-2.1.0-cp37-cp37m-win_amd64.whl рдЬреЛ рдкреЗрдЬ рдХрд╣рддрд╛ рд╣реИ рдХрд┐ рд╡рд┐рдВрдбреЛрдЬрд╝ рдХреЗ рд▓рд┐рдП рдкрд╛рдЗрдерди 3.7 рдХрд╛ рдЙрдкрдпреЛрдЧ рдХрд░рдирд╛ рдЪрд╛рд╣рд┐рдПред рдпрджрд┐ рдЖрдкрдХреЛ рдпрд╣ рдХрд╣рддреЗ рд╣реБрдП рддреНрд░реБрдЯрд┐ рдорд┐рд▓рддреА рд╣реИ рдХрд┐ рдЗрд╕ рдкреНрд▓реЗрдЯрдлрд╝реЙрд░реНрдо рдХреЗ рд▓рд┐рдП рдкрд╣рд┐рдпрд╛ рдореМрдЬреВрдж рдирд╣реАрдВ рд╣реИ, рддреЛ рдЗрд╕рдХрд╛ рдорддрд▓рдм рд╣реИ рдХрд┐ рдЖрдк 32 рдмрд┐рдЯ рдЕрдЬрдЧрд░ рдХрд╛ рдЙрдкрдпреЛрдЧ рдХрд░ рд░рд╣реЗ рд╣реИрдВ

рд╡реНрд╣реАрд▓ рдлрд╝рд╛рдЗрд▓ рдХрд╛ рдЙрдкрдпреЛрдЧ рдХрд░рдХреЗ рд╕реНрдерд╛рдкрд┐рдд рдХрд┐рдпрд╛ рдЧрдпрд╛ред

рдХреГрдкрдпрд╛ Java рдХреЗ рд▓рд┐рдП tensorflow рд▓рд╛рдпрдмреНрд░реЗрд░реАрдЬрд╝ рдХрд╛ рдЕрджреНрдпрддрди рд╕рдВрд╕реНрдХрд░рдг рдЬрд╛рд░реА рдХрд░реЗрдВред

рдореИрдВрдиреЗ рдЗрд╕реЗ рд╕рдлрд▓рддрд╛рдкреВрд░реНрд╡рдХ рд╕реНрдерд╛рдкрд┐рдд рдХрд┐рдпрд╛ pip install https://storage.googleapis.com/tensorflow/mac/cpu/tensorflow-1.8.0-py3-none-any.whl ред рд╣рд╛рд▓рд╛рдБрдХрд┐, рдРрд╕рд╛ рд▓рдЧрддрд╛ рд╣реИ рдХрд┐ рдпрд╣ рдкрд╣рд┐рдпрд╛ Python 3.7 рд╕рдВрдЧрдд рдирд╣реАрдВ рд╣реИред рд╕рдорд╕реНрдпрд╛ рдпрд╣ рд╣реИ рдХрд┐ tensorflow/python/pywrap_tensorflow_internal.py рдореЗрдВ рдЪрд░ рдирд╛рдо рдХреЗ рд░реВрдк рдореЗрдВ async рд╣реИ рд▓реЗрдХрд┐рди async python3.7 рдореЗрдВ рдПрдХ рдХреАрд╡рд░реНрдб рдмрди рдЬрд╛рддрд╛ рд╣реИ, рдЗрд╕рд▓рд┐рдП рдпрд╣ рдЕрдм рдЙрдкрд▓рдмреНрдз рдирд╣реАрдВ рд╣реИред
рд╕рд╛рде рд╣реА, рдореИрдВрдиреЗ рджреЗрдЦрд╛ рдХрд┐ рдпреВрдЖрд░рдПрд▓ рдХреЗрд╡рд▓ рдкрд╛рдпрдерди 3.4, 3.5, 3.6 рдХреЗ рд▓рд┐рдП рд╣реИ рд▓реЗрдХрд┐рди рдореБрдЭреЗ рдХреЛрдИ рдЕрдиреНрдп рдпреВрдЖрд░рдПрд▓ рдирд╣реАрдВ рдорд┐рд▓ рд░рд╣рд╛ рд╣реИред рддреЛ рд╢рд╛рдпрдж рд╣рдореЗрдВ рдЖрдзрд┐рдХрд╛рд░рд┐рдХ рдкрд╛рдпрдерди 3.7-рд╕рдВрдЧрдд рдкреИрдХреЗрдЬ рдХреА рдкреНрд░рддреАрдХреНрд╖рд╛ рдХрд░рдиреЗ рдХреА рдЖрд╡рд╢реНрдпрдХрддрд╛ рд╣реИред

рдореИрдВрдиреЗ рдЗрд╕реЗ рд╕рдлрд▓рддрд╛рдкреВрд░реНрд╡рдХ рд╕реНрдерд╛рдкрд┐рдд рдХрд┐рдпрд╛ pip install https://storage.googleapis.com/tensorflow/mac/cpu/tensorflow-1.8.0-py3-none-any.whl ред рд╣рд╛рд▓рд╛рдБрдХрд┐, рдРрд╕рд╛ рд▓рдЧрддрд╛ рд╣реИ рдХрд┐ рдпрд╣ рдкрд╣рд┐рдпрд╛ Python 3.7 рд╕рдВрдЧрдд рдирд╣реАрдВ рд╣реИред рд╕рдорд╕реНрдпрд╛ рдпрд╣ рд╣реИ рдХрд┐ tensorflow/python/pywrap_tensorflow_internal.py рдореЗрдВ рдЪрд░ рдирд╛рдореЛрдВ рдХреЗ рд░реВрдк рдореЗрдВ async рд╣реИ рд▓реЗрдХрд┐рди async python3.7 рдореЗрдВ рдПрдХ рдХреАрд╡рд░реНрдб рдмрди рдЬрд╛рддрд╛ рд╣реИ, рдЗрд╕рд▓рд┐рдП рдпрд╣ рдЕрдм рдЙрдкрд▓рдмреНрдз рдирд╣реАрдВ рд╣реИред
рд╕рд╛рде рд╣реА, рдореИрдВрдиреЗ рджреЗрдЦрд╛ рдХрд┐ рдпреВрдЖрд░рдПрд▓ рдХреЗрд╡рд▓ рдкрд╛рдпрдерди 3.4, 3.5, 3.6 рдХреЗ рд▓рд┐рдП рд╣реИ рд▓реЗрдХрд┐рди рдореБрдЭреЗ рдХреЛрдИ рдЕрдиреНрдп рдпреВрдЖрд░рдПрд▓ рдирд╣реАрдВ рдорд┐рд▓ рд░рд╣рд╛ рд╣реИред рддреЛ рд╢рд╛рдпрдж рд╣рдореЗрдВ рдЖрдзрд┐рдХрд╛рд░рд┐рдХ рдкрд╛рдпрдерди 3.7-рд╕рдВрдЧрдд рдкреИрдХреЗрдЬ рдХреА рдкреНрд░рддреАрдХреНрд╖рд╛ рдХрд░рдиреЗ рдХреА рдЖрд╡рд╢реНрдпрдХрддрд╛ рд╣реИред

рдЗрд╕рд╕реЗ рдореБрдЭреЗ рдорджрдж рдорд┐рд▓реА, рдзрдиреНрдпрд╡рд╛рдж!
рд╡реАрдПрд╕рд╕реА рдореЗрдВ, рдкрд╛рдпрдерди 3.7.2

рдореИрдВрдиреЗ Python3.7 рдкрд░ рд╕рдлрд▓рддрд╛рдкреВрд░реНрд╡рдХ рд╕реНрдерд╛рдкрд┐рдд рдХрд┐рдпрд╛ рд╣реИ рд▓реЗрдХрд┐рди рдпрд╣ рдЗрд╕ рддреНрд░реБрдЯрд┐ рдХреЛ рджрд┐рдЦрд╛рддрд╛ рд╣реИред
рдлрд╝рд╛рдЗрд▓ "C:\Python37-32\lib\site-packages\tensorflow\python\pywrap_tensorflow_internal.py", рдкрдВрдХреНрддрд┐ 114
def TFE_ContextOptionsSetAsync(arg1, async):

рдЗрд╕ рдореБрджреНрджреЗ рдХреЛ рд╣рд▓ рдХрд░рдиреЗ рдХреЗ рд▓рд┐рдП рдХреЛрдИ рд╕рдорд╛рдзрд╛рди?

рдореИрдВрдиреЗ рдкрд╛рдЗрдк рдЗрдВрд╕реНрдЯрд╛рд▓ https://storage.googleapis.com/tensorflow/mac/cpu/tensorflow-1.8.0-py3-none-any.whl рдХрд╛ рдЙрдкрдпреЛрдЧ рдХрд░рдХреЗ Python3.7 рдореЗрдВ рд╕рдлрд▓рддрд╛рдкреВрд░реНрд╡рдХ рдЯреЗрдВрд╕рд░рдлрд╝реНрд▓реЛ рд╕реНрдерд╛рдкрд┐рдд рдХрд┐рдпрд╛ рд╣реИ
рд▓реЗрдХрд┐рди рдЗрд╕рдХрд╛ рдЙрдкрдпреЛрдЧ рдХрд░рддреЗ рд╕рдордп, рдпрд╣ рдЗрд╕ рддреНрд░реБрдЯрд┐ рдХреЛ рджрд┐рдЦрд╛рддрд╛ рд╣реИред
рдлрд╝рд╛рдЗрд▓ "C:\Python37-32\lib\site-packages\tensorflow\python\pywrap_tensorflow_internal.py", рдкрдВрдХреНрддрд┐ 114
def TFE_ContextOptionsSetAsync(arg1, async):

рдЗрд╕ рдореБрджреНрджреЗ рдХреЛ рд╣рд▓ рдХрд░рдиреЗ рдХреЗ рд▓рд┐рдП рдХреЛрдИ рд╕рдорд╛рдзрд╛рди?

рдореИрдВрдиреЗ рдкрд╛рдЗрдк рдЗрдВрд╕реНрдЯрд╛рд▓ https://storage.googleapis.com/tensorflow/mac/cpu/tensorflow-1.8.0-py3-none-any.whl рдХрд╛ рдЙрдкрдпреЛрдЧ рдХрд░рдХреЗ Python3.7 рдореЗрдВ рд╕рдлрд▓рддрд╛рдкреВрд░реНрд╡рдХ рдЯреЗрдВрд╕рд░рдлрд╝реНрд▓реЛ рд╕реНрдерд╛рдкрд┐рдд рдХрд┐рдпрд╛ рд╣реИ
рд▓реЗрдХрд┐рди рдЗрд╕рдХрд╛ рдЙрдкрдпреЛрдЧ рдХрд░рддреЗ рд╕рдордп, рдпрд╣ рдЗрд╕ рддреНрд░реБрдЯрд┐ рдХреЛ рджрд┐рдЦрд╛рддрд╛ рд╣реИред
рдлрд╝рд╛рдЗрд▓ "C:\Python37-32\lib\site-packages\tensorflow\python\pywrap_tensorflow_internal.py", рдкрдВрдХреНрддрд┐ 114
def TFE_ContextOptionsSetAsync(arg1, async):

рдЗрд╕ рдореБрджреНрджреЗ рдХреЛ рд╣рд▓ рдХрд░рдиреЗ рдХреЗ рд▓рд┐рдП рдХреЛрдИ рд╕рдорд╛рдзрд╛рди?

рдХреНрдпрд╛ рдРрд╕рд╛ рд╣реИ рдХрд┐ рдЯреЗрдВрд╕рд░рдлрд╝реНрд▓реЛ -1.8.0 рдкреБрд░рд╛рдирд╛ рд╕рдВрд╕реНрдХрд░рдг рд╣реИ ???

рдореИрдВрдиреЗ рдкрд╛рдЗрдк рдЗрдВрд╕реНрдЯрд╛рд▓ https://storage.googleapis.com/tensorflow/mac/cpu/tensorflow-1.8.0-py3-none-any.whl рдХрд╛ рдЙрдкрдпреЛрдЧ рдХрд░рдХреЗ Python3.7 рдореЗрдВ рд╕рдлрд▓рддрд╛рдкреВрд░реНрд╡рдХ рдЯреЗрдВрд╕рд░рдлрд╝реНрд▓реЛ рд╕реНрдерд╛рдкрд┐рдд рдХрд┐рдпрд╛ рд╣реИ
рд▓реЗрдХрд┐рди рдЗрд╕рдХрд╛ рдЙрдкрдпреЛрдЧ рдХрд░рддреЗ рд╕рдордп, рдпрд╣ рдЗрд╕ рддреНрд░реБрдЯрд┐ рдХреЛ рджрд┐рдЦрд╛рддрд╛ рд╣реИред
рдлрд╝рд╛рдЗрд▓ "C:\Python37-32\lib\site-packages\tensorflow\python\pywrap_tensorflow_internal.py", рдкрдВрдХреНрддрд┐ 114
def TFE_ContextOptionsSetAsync(arg1, async):
рдЗрд╕ рдореБрджреНрджреЗ рдХреЛ рд╣рд▓ рдХрд░рдиреЗ рдХреЗ рд▓рд┐рдП рдХреЛрдИ рд╕рдорд╛рдзрд╛рди?

рдХреНрдпрд╛ рдРрд╕рд╛ рд╣реИ рдХрд┐ рдЯреЗрдВрд╕рд░рдлрд╝реНрд▓реЛ -1.8.0 рдкреБрд░рд╛рдирд╛ рд╕рдВрд╕реНрдХрд░рдг рд╣реИ ???

рд╣рд╛рдБ рдпрд╣ рдЯреЗрдВрд╕рд░рдлрд╝реНрд▓реЛ 1.8.0 рд╣реИред рдореИрдВрдиреЗ async рдХреЛ async1 рд╕реЗ рдмрджрд▓ рджрд┐рдпрд╛ рд╣реИ рдХреНрдпреЛрдВрдХрд┐ async рдЕрдЬрдЧрд░ 3.7 рдореЗрдВ рдХреАрд╡рд░реНрдб рд╣реИ
рддреЛ рдпрд╣ рд╕рдорд╕реНрдпрд╛ рд╣рд▓ рд╣реЛ рдЬрд╛рддреА рд╣реИред

рд▓реЗрдХрд┐рди рдЕрдм рдореБрдЭреЗ рдиреАрдЪреЗ рдЙрд▓реНрд▓рд┐рдЦрд┐рдд рддреНрд░реБрдЯрд┐ рд╣реЛ рд░рд╣реА рд╣реИ:

ImportError: '_pywrap_tensorflow_internal' рдирд╛рдо рдХрд╛ рдХреЛрдИ рдореЙрдбреНрдпреВрд▓ рдирд╣реАрдВ

Mac OSX Catalina --pip рдЗрдВрд╕реНрдЯреЙрд▓ https://storage.googleapis.com/tensorflow/mac/cpu/tensorflow-1.8.0-py3-none-any.whl рдкрд░ рдмрд┐рдирд╛ рдХрд┐рд╕реА рд╕рдорд╕реНрдпрд╛ рдХреЗ рдХрд╛рдо рдХрд┐рдпрд╛ред

рдореЗрд░реЗ рдорд╛рдорд▓реЗ рдореЗрдВ, рд╡рд┐рдВрдбреЛрдЬрд╝ рдорд╢реАрди рдореЗрдВ, рдореЗрд░реЗ рдкрд╛рд╕ рдПрдХ рд╣реА рд╕рдорд╕реНрдпрд╛ рдереАред рдореИрдВ рдЕрдЬрдЧрд░ 3.8 рдХрд╛ рдЙрдкрдпреЛрдЧ рдХрд░ рд░рд╣рд╛ рдерд╛ред рдореИрдВрдиреЗ рдЕрдЬрдЧрд░ = 3.6 . рдХреЗ рд╕рд╛рде рдПрдХ рдирдпрд╛ рдХреЛрдВрдбрд╛ рд╡рд╛рддрд╛рд╡рд░рдг рдмрдирд╛рдпрд╛

conda create --name newenv python=3.6 
pip install tensorflow

рдПрдирд╡реАрдбрд┐рдпрд╛ рдЬреЗрдЯрд╕рди рдкрд░ рдЕрдЬрдЧрд░ 3.6 рдХреЗ рд╕рд╛рде рдпрд╣рд╛рдБ рдПрдХ рд╣реА рддреНрд░реБрдЯрд┐

рдореЗрд░реЗ рдорд╛рдорд▓реЗ рдореЗрдВ, рд╡рд┐рдВрдбреЛрдЬрд╝ рдорд╢реАрди рдореЗрдВ, рдореЗрд░реЗ рдкрд╛рд╕ рдПрдХ рд╣реА рд╕рдорд╕реНрдпрд╛ рдереАред рдореИрдВ рдЕрдЬрдЧрд░ 3.8 рдХрд╛ рдЙрдкрдпреЛрдЧ рдХрд░ рд░рд╣рд╛ рдерд╛ред рдореИрдВрдиреЗ рдЕрдЬрдЧрд░ = 3.6 . рдХреЗ рд╕рд╛рде рдПрдХ рдирдпрд╛ рдХреЛрдВрдбрд╛ рд╡рд╛рддрд╛рд╡рд░рдг рдмрдирд╛рдпрд╛

conda create --name newenv python=3.6 
pip install tensorflow

рдпрд╣ рдореЗрд░реЗ рд▓рд┐рдП рдХрд╛рдо рдХрд┐рдпрд╛

рд╣рд╛рдп рдкреНрд░рдгрдп, рдЖрдкрдХреЗ рдЙрддреНрддрд░ рдХреЗ рд▓рд┐рдП рдмрд╣реБрдд рдмрд╣реБрдд рдзрдиреНрдпрд╡рд╛рджред рдЕрдм рдпрд╣ рдореЗрд░реЗ рд▓рд┐рдП рднреА рдареАрдХ рдХрд╛рдо рдХрд░ рд░рд╣рд╛ рд╣реИ рдЕрдЬрдЧрд░ 3.6 рдХрд╛ рдЙрдкрдпреЛрдЧ рдХрд░рдХреЗ

рдпрд╛рд╣реВ рдореЗрд▓ рд╕реЗ рдПрдбреНрд░реЙрдЗрдб рдкрд░ рднреЗрдЬрд╛ рдЧрдпрд╛

рд╢реБрдХреНрд░, 5 рдЬреВрди 2020 рдХреЛ 14:44 рдмрдЬреЗ, рдкреНрд░рдгрдпрд╕реЛрдиреАрд╕реНрдкреИрди рдиреЛрдЯрд┐рдлрд┐рдХреЗрд╢рди @github.com рдиреЗ рд▓рд┐рдЦрд╛:

рдореЗрд░реЗ рдорд╛рдорд▓реЗ рдореЗрдВ, рд╡рд┐рдВрдбреЛрдЬрд╝ рдорд╢реАрди рдореЗрдВ, рдореЗрд░реЗ рдкрд╛рд╕ рдПрдХ рд╣реА рд╕рдорд╕реНрдпрд╛ рдереАред рдореИрдВ рдЕрдЬрдЧрд░ 3.8 рдХрд╛ рдЙрдкрдпреЛрдЧ рдХрд░ рд░рд╣рд╛ рдерд╛ред рдореИрдВрдиреЗ рдЕрдЬрдЧрд░ = 3.6 . рдХреЗ рд╕рд╛рде рдПрдХ рдирдпрд╛ рдХреЛрдВрдбрд╛ рд╡рд╛рддрд╛рд╡рд░рдг рдмрдирд╛рдпрд╛
conda create --name newenv python=3.6
рдкрд╛рдЗрдк рдЯреЗрдВрд╕рд░рдлрд╝реНрд▓реЛ рд╕реНрдерд╛рдкрд┐рдд рдХрд░реЗрдВ

рдпрд╣ рдореЗрд░реЗ рд▓рд┐рдП рдХрд╛рдо рдХрд┐рдпрд╛

-
рдЖрдк рдЗрд╕реЗ рдкреНрд░рд╛рдкреНрдд рдХрд░ рд░рд╣реЗ рд╣реИрдВ рдХреНрдпреЛрдВрдХрд┐ рдЖрдкрдиреЗ рдЯрд┐рдкреНрдкрдгреА рдХреА рдереАред
рдЗрд╕ рдИрдореЗрд▓ рдХрд╛ рд╕реАрдзреЗ рдЙрддреНрддрд░ рджреЗрдВ, рдЗрд╕реЗ GitHub рдкрд░ рджреЗрдЦреЗрдВ, рдпрд╛ рд╕рджрд╕реНрдпрддрд╛ рд╕рдорд╛рдкреНрдд рдХрд░реЗрдВред

Mac OSX Catalina --pip рдЗрдВрд╕реНрдЯреЙрд▓ https://storage.googleapis.com/tensorflow/mac/cpu/tensorflow-1.8.0-py3-none-any.whl рдкрд░ рдмрд┐рдирд╛ рдХрд┐рд╕реА рд╕рдорд╕реНрдпрд╛ рдХреЗ рдХрд╛рдо рдХрд┐рдпрд╛ред

рдореИрдВрдиреЗ рдЗрд╕реЗ рд╕рдлрд▓рддрд╛рдкреВрд░реНрд╡рдХ рд╕реНрдерд╛рдкрд┐рдд рдХрд┐рдпрд╛ pip install https://storage.googleapis.com/tensorflow/mac/cpu/tensorflow-1.8.0-py3-none-any.whl ред рд╣рд╛рд▓рд╛рдБрдХрд┐, рдРрд╕рд╛ рд▓рдЧрддрд╛ рд╣реИ рдХрд┐ рдпрд╣ рдкрд╣рд┐рдпрд╛ Python 3.7 рд╕рдВрдЧрдд рдирд╣реАрдВ рд╣реИред рд╕рдорд╕реНрдпрд╛ рдпрд╣ рд╣реИ рдХрд┐ tensorflow/python/pywrap_tensorflow_internal.py рдореЗрдВ рдЪрд░ рдирд╛рдо рдХреЗ рд░реВрдк рдореЗрдВ async рд╣реИ рд▓реЗрдХрд┐рди async python3.7 рдореЗрдВ рдПрдХ рдХреАрд╡рд░реНрдб рдмрди рдЬрд╛рддрд╛ рд╣реИ, рдЗрд╕рд▓рд┐рдП рдпрд╣ рдЕрдм рдЙрдкрд▓рдмреНрдз рдирд╣реАрдВ рд╣реИред
рд╕рд╛рде рд╣реА, рдореИрдВрдиреЗ рджреЗрдЦрд╛ рдХрд┐ рдпреВрдЖрд░рдПрд▓ рдХреЗрд╡рд▓ рдкрд╛рдпрдерди 3.4, 3.5, 3.6 рдХреЗ рд▓рд┐рдП рд╣реИ рд▓реЗрдХрд┐рди рдореБрдЭреЗ рдХреЛрдИ рдЕрдиреНрдп рдпреВрдЖрд░рдПрд▓ рдирд╣реАрдВ рдорд┐рд▓ рд░рд╣рд╛ рд╣реИред рддреЛ рд╢рд╛рдпрдж рд╣рдореЗрдВ рдЖрдзрд┐рдХрд╛рд░рд┐рдХ рдкрд╛рдпрдерди 3.7-рд╕рдВрдЧрдд рдкреИрдХреЗрдЬ рдХреА рдкреНрд░рддреАрдХреНрд╖рд╛ рдХрд░рдиреЗ рдХреА рдЖрд╡рд╢реНрдпрдХрддрд╛ рд╣реИред

рдореИрдВрдиреЗ рдЗрд╕реЗ рд╕рдлрд▓рддрд╛рдкреВрд░реНрд╡рдХ рд╕реНрдерд╛рдкрд┐рдд рдХрд┐рдпрд╛ pip install https://storage.googleapis.com/tensorflow/mac/cpu/tensorflow-1.8.0-py3-none-any.whl ред рд╣рд╛рд▓рд╛рдБрдХрд┐, рдРрд╕рд╛ рд▓рдЧрддрд╛ рд╣реИ рдХрд┐ рдпрд╣ рдкрд╣рд┐рдпрд╛ Python 3.7 рд╕рдВрдЧрдд рдирд╣реАрдВ рд╣реИред рд╕рдорд╕реНрдпрд╛ рдпрд╣ рд╣реИ рдХрд┐ tensorflow/python/pywrap_tensorflow_internal.py рдореЗрдВ рдЪрд░ рдирд╛рдореЛрдВ рдХреЗ рд░реВрдк рдореЗрдВ async рд╣реИ рд▓реЗрдХрд┐рди async python3.7 рдореЗрдВ рдПрдХ рдХреАрд╡рд░реНрдб рдмрди рдЬрд╛рддрд╛ рд╣реИ, рдЗрд╕рд▓рд┐рдП рдпрд╣ рдЕрдм рдЙрдкрд▓рдмреНрдз рдирд╣реАрдВ рд╣реИред
рд╕рд╛рде рд╣реА, рдореИрдВрдиреЗ рджреЗрдЦрд╛ рдХрд┐ рдпреВрдЖрд░рдПрд▓ рдХреЗрд╡рд▓ рдкрд╛рдпрдерди 3.4, 3.5, 3.6 рдХреЗ рд▓рд┐рдП рд╣реИ рд▓реЗрдХрд┐рди рдореБрдЭреЗ рдХреЛрдИ рдЕрдиреНрдп рдпреВрдЖрд░рдПрд▓ рдирд╣реАрдВ рдорд┐рд▓ рд░рд╣рд╛ рд╣реИред рддреЛ рд╢рд╛рдпрдж рд╣рдореЗрдВ рдЖрдзрд┐рдХрд╛рд░рд┐рдХ рдкрд╛рдпрдерди 3.7-рд╕рдВрдЧрдд рдкреИрдХреЗрдЬ рдХреА рдкреНрд░рддреАрдХреНрд╖рд╛ рдХрд░рдиреЗ рдХреА рдЖрд╡рд╢реНрдпрдХрддрд╛ рд╣реИред

рдЗрд╕рд╕реЗ рдореБрдЭреЗ рдорджрдж рдорд┐рд▓реА, рдзрдиреНрдпрд╡рд╛рдж!
рд╡реАрдПрд╕рд╕реА рдореЗрдВ, рдкрд╛рдпрдерди 3.7.2

рдпрд╣ рдореЗрд░реЗ рд▓рд┐рдП рдХрд╛рдо рдХрд░рддрд╛ рд╣реИ!

рдпрд╣ рдореЗрд░реЗ рд▓рд┐рдП рдХрд╛рдо рд╣реИ
рдмрд╕ рдЗрд╕реЗ рдиреАрдЪреЗ рдЙрдкрдпреЛрдЧ рдХрд░рдиреЗ рдХрд╛ рдкреНрд░рдпрд╛рд╕ рдХрд░реЗрдВ
рдкрд╛рдЗрдк рд╕реНрдерд╛рдкрд┐рдд рдХрд░реЗрдВ https://storage.googleapis.com/tensorflow/mac/cpu/tensorflow-1.8.0-py3-none-any.whl

рдореЗрд░реЗ рдорд╛рдорд▓реЗ рдореЗрдВ рдпрд╣ рдкрд╣рд┐рдпрд╛ рд╕рд╣реА рдврдВрдЧ рд╕реЗ рдХрд╛рдо рдирд╣реАрдВ рдХрд░рддрд╛ рд╣реИ
рдпрд╣ рддреНрд░реБрдЯрд┐ рд╕реЗ рдиреАрдЪреЗ рдлреЗрдВрдХрддрд╛ рд╣реИ

рдкрд╛рдпрдерди 3.7.9 (рдбрд┐рдлрд╝реЙрд▓реНрдЯ, рдЕрдЧрд╕реНрдд 31 2020, 12:42:55)
[рдЬреАрд╕реАрд╕реА 7.3.0] :: рдПрдирд╛рдХреЛрдВрдбрд╛, рдЗрдВрдХ. рд▓рд┐рдирдХреНрд╕ рдкрд░
рдЕрдзрд┐рдХ рдЬрд╛рдирдХрд╛рд░реА рдХреЗ рд▓рд┐рдП "рд╕рд╣рд╛рдпрддрд╛", "рдХреЙрдкреАрд░рд╛рдЗрдЯ", "рдХреНрд░реЗрдбрд┐рдЯ" рдпрд╛ "рд▓рд╛рдЗрд╕реЗрдВрд╕" рдЯрд╛рдЗрдк рдХрд░реЗрдВред

tf . рдХреЗ рд░реВрдк рдореЗрдВ рдЯреЗрдВрд╕рд░рдлрд╝реНрд▓реЛ рдЖрдпрд╛рдд рдХрд░реЗрдВ
рдЯреНрд░реЗрд╕рдмреИрдХ (рд╕рдмрд╕реЗ рд╣рд╛рд▓рд┐рдпрд╛ рдХреЙрд▓ рдЕрдВрддрд┐рдо):
рдлрд╝рд╛рдЗрд▓ "", рд▓рд╛рдЗрди 1, рдЗрди
рдлрд╝рд╛рдЗрд▓ "/home/ahmed/anaconda3/envs/tf14/lib/python3.7/site-packages/tensorflow/__init__.py", рд▓рд╛рдЗрди 24, рдореЗрдВ
tensorflow.python рд╕реЗ рдЖрдпрд╛рдд pywrap_tensorflow # рдкрд╛рдЗрд▓рд┐рдВрдЯ: рдЕрдХреНрд╖рдо = рдЕрдкреНрд░рдпреБрдХреНрдд-рдЖрдпрд╛рдд
рдлрд╝рд╛рдЗрд▓ "/home/ahmed/anaconda3/envs/tf14/lib/python3.7/site-packages/tensorflow/python/__init__.py", рд▓рд╛рдЗрди 49, рдореЗрдВ
tensorflow.python рд╕реЗ рдЖрдпрд╛рдд pywrap_tensorflow
рдлрд╝рд╛рдЗрд▓ "/home/ahmed/anaconda3/envs/tf14/lib/python3.7/site-packages/tensorflow/python/pywrap_tensorflow.py", рд▓рд╛рдЗрди 58, рдореЗрдВ
tensorflow.python.pywrap_tensorflow_internal рдЖрдпрд╛рдд рд╕реЗ *
рдлрд╝рд╛рдЗрд▓ "/home/ahmed/anaconda3/envs/tf14/lib/python3.7/site-packages/tensorflow/python/pywrap_tensorflow_internal.py", рдкрдВрдХреНрддрд┐ 114
def TFE_ContextOptionsSetAsync(arg1, async):
^
рд╕рд┐рдВрдЯреИрдХреНрд╕ рддреНрд░реБрдЯреА: рдЕрдорд╛рдиреНрдп рд╕рд┐рдВрдЯреИрдХреНрд╕

рдирдорд╕реНрддреЗ! рдХреБрдЫ рдореГрдд рдЕрдВрдд рдереЗ, рд▓реЗрдХрд┐рди рдЗрд╕рдиреЗ рдореЗрд░реЗ рд▓рд┐рдП рдЕрдЬрдЧрд░ 3.6.9/3.8 рдкрд░ рдХрд╛рдо рдХрд┐рдпрд╛ (рдирд╡реАрдирддрдо рд╕рдВрд╕реНрдХрд░рдг рдореЗрдВ рд╕реЗрдЯрдкреВрд▓ рдХреЛ рдЕрдкрдбреЗрдЯ рдХрд░рдирд╛, рдлрд┐рд░ рдЯреЗрдВрд╕рд░рдлрд╝реНрд▓реЛ рд╕реНрдерд╛рдкрд┐рдд рдХрд░рдирд╛):

рдЪрд░рдг 1: pip install -U pip
рдЪрд░рдг 2: pip install -U setuptools
рдЪрд░рдг 3: pip install tensorflow

рд╕реНрд░реЛрдд: Liquidweb.com/kb/how-to-install-tensorflow-on-ubuntu-18-04/

рдХреНрдпрд╛ рдпрд╣ рдкреГрд╖реНрда рдЙрдкрдпреЛрдЧреА рдерд╛?
0 / 5 - 0 рд░реЗрдЯрд┐рдВрдЧреНрд╕

рд╕рдВрдмрдВрдзрд┐рдд рдореБрджреНрджреЛрдВ

ppwwyyxx picture ppwwyyxx  ┬╖  3рдЯрд┐рдкреНрдкрдгрд┐рдпрд╛рдБ

fobus42 picture fobus42  ┬╖  3рдЯрд┐рдкреНрдкрдгрд┐рдпрд╛рдБ

3rduncle picture 3rduncle  ┬╖  3рдЯрд┐рдкреНрдкрдгрд┐рдпрд╛рдБ

wfs picture wfs  ┬╖  3рдЯрд┐рдкреНрдкрдгрд┐рдпрд╛рдБ

ilblackdragon picture ilblackdragon  ┬╖  3рдЯрд┐рдкреНрдкрдгрд┐рдпрд╛рдБ